Exam Master 2

¡Supera tus tareas y exámenes ahora con Quizwiz!

Breast abscess Phyllodes tumor Juvenile fibroadenoma Giant fibroadenoma Malignancy Juvenile fibroadenoma

A 16-year-old girl presents to her primary care physician after having discovered a breast mass while bathing. She reports no symptoms of any kind; the mass was discovered while taking a shower. There is no history of chest trauma. Exam reveals Tanner stage IV breast development, appropriate to age, and a 2 cm mass in the upper outer quadrant of the right breast. The mass is rubbery in character, mobile, with distinctly palpable borders, non-tender, not fixed to adjacent tissue, and without change in surface anatomy of the breast. What is the most likely diagnosis?

Measurement of urine volume Antibiotics Oxytocin Uterine massage Urodynamic studies Measurement of urine volume

6 hours after delivery, a 25-year-old primigravida presents with fullness and pain in the lower abdomen. Her pregnancy was normal; labor was induced with misoprostol and progressed normally during the first stage; episiotomy was performed 2 hours after second stage had started; and the third stage of labor was normal. The newborn weighed 3800 g, and APGAR score was 9, 9, and 10 in 5 minute intervals. Postpartum visually estimated blood loss is about 550 mL (normally ≥500 mL in the first 24 hours after delivery). She did not void after the delivery. On examination, her vitals are stable and her bladder is overdistended. What is the next step in the management of this patient in addition to catheterization?

CT scan of the brain and a skeletal survey Parental reassurance and discharge with outpatient follow-up Administration of IV fluid and acetaminophen Hematology evaluation for the presence of a bleeding diathesis Ophthalmology consultation CT scan of the brain and a skeletal survey

A 1-year-old boy presents with a history of vomiting, lethargy, and respiratory distress of sudden onset; there is no prior illness or injury. The parents state that the boy is "always irritable" and he keeps them up all night. The parents have missed several well-baby visits; they both work and are unable to keep the appointments. Physical examination reveals red bruises on the boy's arms that appear in the shape of fingers, along with a full fontanel. Ophthalmologic exam demonstrates the presence of retinal hemorrhage. What is the most appropriate next step in the management of this patient?

Dextroamphetamine Imipramine Selegiline Atomoxetine Fluoxetine Dextroamphetamine

A 10-year-old boy has a history of problems at school and at home. Teachers report he rarely can focus on one task for longer than a few minutes, and they describe his behavior as chaotic. His mother states that he never gets tired of running, talking, and playing around the house; she usually has to repeat instructions over and over because he seems to not listen. She also reports that he failed at school and is now repeating the fourth grade. A psychostimulant, an indirect-acting adrenergic receptor agonist that centrally releases dopamine (DA), serotonin (5-HT), and norepinephrine (NE) to the synaptic cleft, is prescribed. What drug is most likely the medication prescribed for this patient?

Streptococcus pyogenes (group A beta-hemolytic) Streptococcus agalactiae (group B) Enterococcus faecalis Peptostreptococcus Streptococcus pneumoniae Streptococcus pyogenes (group A beta-hemolytic) --> celllulitis

A 10-year-old boy is brought to the clinic by his mother. She noted that his face is swollen, and he told her that his urine was cloudy and reddish. He has a history of falling and abrading the skin of his right thigh 2 weeks ago. The next day, the skin became red, hot, and tender; the infection was treated with a topical antibiotic ointment. The cellulitis gradually healed. What organism is the most likely cause of this patient's disease?

He has ADHD. He has a factitious disorder. He has a condition that clusters in families. He must start a regimen of levodopa. He has Sydenham chorea He has a condition that clusters in families.

A 10-year-old boy is referred to you for being fidgety at school, even though he gets good grades. Prior history is unremarkable, and there has been no recent illness. The boy's father had a similar history as a child. While speaking with the patient, you notice that he clears his throat several times per minute. Examination is otherwise normal, except for rapid non-rhythmic jerking movements of the face, neck, and shoulders while at rest. What is true regarding this patient's condition?

Post-traumatic stress disordee School phobia/refusal Depression Separation anxiety disorder Agoraphobia Separation anxiety disorder

A 12-year-old girl presents with symptoms of anxiety. Her parents report she has always been anxious, but her anxiety levels have escalated during this school year. She frequently resists going to school in the morning. Once there, her symptoms often escalate enough to involve the school nurse. They are usually alleviated by telephone contact with her mother. Symptoms also occur on weekends when her mother has to work. The patient has a normal group of friends, but she prefers to be with them at her home rather than elsewhere. She has never slept away from home. She spends a considerable amount of time alone in her room, but she will often come out to "check up" on her mother. What diagnosis best fits the patient's symptoms?

Abdominal ultrasound Upper GI barium studies CT scan abdomen Upper GI endoscopy Stool microscopy Upper GI endoscopy

A 10-year-old boy presents with a 2-month history of intermittent burning pain in the epigastrium. Pain is felt more during the night and between meals; it is partly relieved by eating food or by taking antacids. Pain usually lasts 30-60 minutes and is accompanied by nausea and vomiting. He often has a feeling of bloating and burping. He remains asymptomatic for several days between. There is no history of taking analgesics or anti-inflammatory drugs. Physical examination shows epigastric tenderness. The rest of the examination is essentially normal. Stool examination for occult blood is positive. What is the investigation of choice for establishing the diagnosis?

Facial and right arm twitching lasting 1 minute, then loss of consciousness Blank stare and impaired awareness for 10-20 seconds Eye deviation with facial twitching that lasts 1-2 minutes Loss of consciousness, then stiffening of the body and extremities Brief periods of whole body going limp, then a brief loss of consciousnes Eye deviation with facial twitching that lasts 1-2 minutes

A 12-month-old girl presents with her parents after a 3-day history of intermittent episodes of strange behavior. A neurologist is consulted because the parents are concerned she is having some form of seizure activity. Her health history includes 2 episodes of otitis media, but she is otherwise healthy. Her initial vital signs and physical exam by the emergency room staff are all normal. What description of the infant's strange behavior would lead the neurologist to suspect a diagnosis of simple partial seizures?

Tetanus Diphtheria Guillain-Barré syndrome Botulism Myasthenia gravis Guillain-Barré syndrome (Ascending paralysis)

A 12-year-old girl presents with a 3-day history of progressive weakness and paresthesias in lower legs. Yesterday, she developed weakness in both upper extremities. She is unable to walk without assistance, so her mother brought her in. She had a sore throat 2 weeks ago. PMH is significant for measles and mumps. Because of religious beliefs, she has not had any immunizations. Physical examination reveals a well-developed well-nourished girl. She is awake, alert, and in no acute distress. Oral temperature 98.7°F, blood pressure 140/80 mm Hg, heart rate 84/min and regular, respirations 22/min and unlabored. Speech is moderately dysarthric. She can smile weakly but cannot raise her eyebrows against resistance. Pupillary responses are normal. There is mild upper extremity and severe lower extremity weakness, greater distally than proximally. Reflexes are hypoactive-to-absent. Sensation is intact, except for mildly impaired position and vibratory sensation in both feet. What is the most likely diagnosis?

Nephrotic syndrome

A 12-year-old African American boy presents with a 1-month history of poor appetite and has complained of overall not feeling well. When questioned, the boy cannot delineate any specific symptoms except that he feels "puffy." He denies pain, eating disorder, rash, depression, drug use, and fevers. The family denies recent travel. His PMH is unremarkable with no recent or chronic illnesses. He has had no surgeries, and he takes no medications. His family history includes grandparents on both sides with hypertension, with one of these grandparents also having died from some type of kidney problem. His ROS is entirely negative except for the above symptoms and some noted change in urine, which he describes as frothy. He denies dysuria, gross hematuria, polyuria, and nocturia in the ROS.On physical exam, his vitals are: Tem: 97.9°F; Resp 14; HR 90; BP 120/74 mm Hg right arm sitting. It is noted weight is up 2 pounds from his charted weight 3 months ago. HEENT, neck/thyroid, lungs, cardiac, abdominal, musculoskeletal, neurological and derm exams are unremarkable. Examination of extremities reveals bilateral 1-2+ edema in the upper and lower extremities, with 2+ pulses.Basic laboratory studies were ordered and results are available at the time of visit Given the proteinuria, a urine protein-to-creatinine ratio is obtained, showing 3.6 mg/mg. What is the most likely diagnosis?

Rest, ice, compression, and elevation Referral to physical therapy Ibuprofen 400 mg every 4 hours Application of knee brace Urgent referral to orthopedic surgery Urgent referral to orthopedic surgery (Slipped capital femoral epiphysis from the femoral head )

A 12-year-old boy presents with a 3-month history of intermittent moderate right knee pain and mild right hip pain increasing in frequency and severity over the past week. There is no history of trauma or associated symptoms. He has begun to experience stiffness in the right hip and has developed a limp. Examination reveals normal vital signs, height 64", weight 182 lb (>95th percentile). He walks with an antalgic gait with the right foot externally rotated. There is pain on passive range of motion of the right hip, but not the right knee. Internal rotation of the right hip is limited. External rotation of the right hip with the knee flexed produces external rotation of the right lower leg. Radiographs of the right knee are normal, but radiographs of the right hip reveal a moderate increase of the angle of the femoral head to shaft. What is the best next step to treat the suspected diagnosis in this patient?

Wilson disease

A 12-year-old boy presents with fatigue and jaundice. His past medical history is not significant for recent illness, fever, infectious exposures, medication, alcohol, or drug use. He denies gastrointestinal (GI) symptoms and a history of GI disease. On physical examination, he appears ill; the liver edge is palpable and slightly tender. Skin and sclera are icteric, and there is corneal discoloration. On further eye examination using a slit lamp, brown-yellow rings encircling the iris in the rim of the cornea are noted bilaterally. AST and ALT are elevated, and a serum ceruloplasmin level is reported as low and confirms the diagnosis. What is the name of this disease process?

MRI of the right ankle Hard casting for 6 weeks Early mobilization Hydrocodone 5 mg Rigid brace Early mobilization

A 12-year-old boy who presented to the emergency department 3 days ago with a right ankle injury. He "rolled over" his ankle playing football. Radiographs were negative. He has been treating with rest, icing, compression, and elevation (RICE). Past medical history is unremarkable, without prior ankle injuries. Today, he has mild pain and swelling of the outer aspect of the right ankle near the lateral malleolus with mild tenderness on motion, especially inversion, with a mild limitation of motion due to pain. He can almost fully bear weight on the ankle. There is no joint instability. What is your next step in managing this injury?

Pseudomonas aeruginosa

A 12-year-old girl presents to the emergency room with worsening foot pain. 2 weeks ago, she stepped on a nail while wearing rubber-soled tennis shoes. The area was thoroughly cleaned, but she has developed worsening pain, redness, and drainage from the area. X-ray shows periosteal changes at the site of the wound. A wound culture is obtained. What organism is most likely to grow on culture for this specific mechanism of injury?

Tenderness Edema Horizontal lie Positive Prehn sign Loss of cremasteric reflex Loss of cremasteric reflex

A 12-year-old male patient presents with acute onset of 3 hours of severe pain in the right testis rated 8/10, associated with nausea and scrotal swelling. He never had such pain in his life, and he denies any problem in urination. He has never been operated on, and he denies any history of trauma. On exam, he is in visible distress. Temperature 37°C, heart rate 95 bpm, blood pressure 120/70 mm Hg. Genital examination reveals enlargement and edema of the entire scrotum. The right testicle is erythematous and tender to palpation; it appears to sit higher and lies horizontally in the scrotal sac relative to the left side. The cremasteric reflex is absent ipsilaterally, and there is no relief of pain upon elevation of the scrotum (Prehn sign). Labs show hemoglobin 14.5 g/dL, WBC 13,000/mm3, platelets 210,000/mm3, sodium 140 mmol/dL, potassium 3.8 mmol/dL, chloride 95 mmol/dL, urea 25 mg/dL, creatinine 0.9 mg/dL. What clinical feature helps the most to differentiate this patient's condition from other causes of scrotal pain?

Echolalia

A 13-year-old boy diagnosed with autism spectrum disorder repeats phrases in a parrot-like fashion; he repeats whatever he hears, but comprehension is absent. What condition is this patient exhibiting?

IgA nephritis Nephritic glomerular disease Nephrotic glomerular disease Orthostatic proteinuria Urinary tract infection Nephritic glomerular disease (low proteinuria and puffy face)

A 13-year-old boy presents for an annual examination; he reports no problems with his health. Upon physical examination, his body temperature is 98.3°F, his blood pressure is 150/100 mm Hg, and he shows a slight periorbital puffiness. He explains that recently he has been staying up late to read and has noticed "puffy eyes" in the morning. Urinalysis indicates light brown urine, low-level proteinuria, and no bacteriuria. What is the most likely diagnosis?

Oral ibuprofen (prominent tibial tubercle)

A 13-year-old boy presents with a 5-month history of intermittent right knee pain. He notes that his pain is poorly localized over the anterior knee, but it is provoked by running, jumping, squatting, and climbing or descending stairs. He has had to curtail recreational games during recess and at home. He denies any history of trauma, falls, injury, fever, chills, swelling, skin changes, hip pain, or ankle pain. Physical exam reveals a boy with an appropriate build. The right knee is notable for point tenderness over an enlarged and prominent tibial tubercle. There is associated focal swelling. An in-office lateral x-ray shows anterior soft tissue swelling and elevation of the tibial tubercle. What is the most appropriate pharmacotherapeutic agent for this patient?

Plasmacytoma Ewing's sarcoma Multiple myeloma Endochondroma Osteochondroma Ewing's sarcoma (onion-skin invasive lesion)

A 13-year-old boy presents with pain in his right leg, present for about 2 months but worsening over time. He has developed a low-grade fever. He denies any known injury to the area. On examination, there is tenderness and mild swelling near the right fibula. X-ray reveals an invasive lesion involving the right fibula, with a periosteal onion-skin reaction. What is the most likely diagnosis?

Surgery referral for breast tissue removal Breast ultrasound to assess for malignancy Serum testosterone, estradiol, FSH, and LH levels Point of care urine drug screen Nutrition consult for obesity management Serum testosterone, estradiol, FSH, and LH levels

A 13-year-old boy presents with right breast development over the last 5-6 months. He is on the swim team, so this has caused him much consternation. The breast swelling is slightly tender and without any drainage. He has been healthy, denies taking any medications, denies any substance use or trauma, and is doing well at school. On exam, height is 63" (160 cm) and weight is 115 lb (53 kg), which are both 75th percentile, BP is 94/68, pulse 68 beats/min. There is slightly tender 7-8 cm of right breast elevation and swelling extending from the areola that is non-erythematous. The left side is normal. There is no axillary lymphadenopathy. The testes are descended bilaterally and measure 3 cm in size. Pubic hair shows sparse growth of long downy hair at the base of the penis. There is no axillary or facial hair. He has scant acne. His mother is very anxious and would like further testing. What is the most appropriate initial evaluation to rule out any underlying pathology?

Bulimia nervosa

A 13-year-old girl has a history of being found by her mother consuming large amounts of high-calorie food on several occasions and then vomiting what she ate. For the past 2 days, the patient has experienced pain in her throat, which was diagnosed as esophagitis. Upon physical exam, the patient is dehydrated, has abnormal electrolyte imbalances, and appears to have eroding front teeth. What is the most likely explanation of these findings?

Frequent vomiting Diabetes mellitus Myasthenia gravis Ethylene glycol ingestion Salicylate ingestion Diabetes mellitus

A 13-year-old girl is drowsy and unable to answer questions. Her mother says that she has been extremely thirsty lately and urinates frequently. Her father notes that the patient has also been fatigued. There is a fruity odor to the patient's breath. Blood gases are drawn and reveal the following: pH - 7.3 CO2- 32 HCO - 17 What is the most likely cause of the patient's condition?

Kyphosis

A 13-year-old girl presents for her school physical. On examination, you notice the posterior curvature of her thoracic spine to be very prominent and bulging backward. What type of deformity of the spine does she have?

Intussusception

A 2-year-old boy presents with acute abdominal pain. The boy has passed stool with blood and mucus and has vomited. He has had intermittent severe abdominal pain, which has caused inconsolable crying and drawing up of his legs in episodes of 15-20 minutes. On examination, the abdomen is tender diffusely with guarding and bowel sounds are absent. He has a fever of 100.5°F. The boy is taken to surgery. Refer to the image. What is the most likely diagnosis?

Osgood-Schlatter disease

A 14-year old girl presents due to right leg pain. She is a competitive gymnast and works out in the gym 6 days a week. Upon physical exam, there is knee pain, swelling, and tenderness at the tibial tubercle. The patient's muscles are very tight in front and behind her thigh. All laboratory findings are within normal limits. What is the most likely diagnosis?

Needle aspiration to rule out tumor Reassurance that this is normal Referral to a breast surgeon for biopsy Referral to social services 2-week course of oral steroids Reassurance that this is normal - understand that it is bilaterally - normal, not cancer

A 14-year-old boy presents with bilaterally tender and swollen breasts. He has become increasingly self-conscious about the condition, resulting in his avoiding physical education classes and swim team practices. He has always had good general health with no history of medical illness, hospitalizations, or medications. He does not smoke or drink alcohol, and there is no history of illicit drug use or use of any activity-enhancing products. Family history is positive for a sister and his mother being diagnosed with breast cancer. The remainder of the review of systems is non-contributory. On physical examination, he appears normal in growth and development; there are tender soft masses in the lower quadrants of both breasts, approximately 3 centimeters in diameter. The remainder of the physical examination is unremarkable. What is the most appropriate next step in managing this patient?

Cri du chat syndrome Down syndrome (trisomy 21) Fragile X syndrome Klinefelter syndrome (XXY) Turner syndrome (XO) Fragile X syndrome

A 14-year-old boy is brought to your medical office by his mother for a physical examination. According to the mother, the child was diagnosed with intellectual disability (intellectual developmental disorder) 2 years ago. They have just relocated from another state, and he requires a physician's clearance to start at a new special education school. She states that he has been in good physical health since birth. His past medical history includes a few mild headaches and upper respiratory tract infections, but no chronic conditions, hospitalizations, operations, or medications. The mother has brought his vaccination records with her, and they show all immunizations are current. Family history is positive for a maternal grandmother that developed dementia at age 55 and a maternal uncle with autism. Pertinent findings on the physical examination include an unusually narrow face, a prominent forehead, large protruding ears, a prominent jaw, and unusually large testes. What is the most likely diagnosis?

Pubertal growth spurt Increase in size of testes Increase in body mass Increase in bone accretion Elongation of penis Increase in size of testes

A 14-year-old boy presents due to embarrassment after an incident in school: while undressing before the class in physical education, the other boys laughed at him because of his "underdevelopment." His personal and family history is non-contributory. What is a definitive sign of the onset of puberty in this patient?

Trichinella spiralis

A 14-year-old boy presents with a 1-week history of acute watery diarrhea with vague abdominal discomfort and vomiting. He has developed fever, malaise, facial and periorbital edema, and myalgias. He is experiencing pain and swelling of the calf muscles. Blood examination shows moderate eosinophilia. The parents report that the boy had recently spent a month with his grandparents in Romania, where he learned to hunt and ate wild boar and other meats regularly. What is the most likely etiological agent responsible for this patient's illness?

Follicle-stimulating hormone (FSH)

A 14-year-old girl presents to her family practice physician assistant with her mother. She reports no issues, has no problems at school, participates in school sports activities, and is not sexually active. Her past medical history is non-contributory. Her mother is worried because she still looks prepubertal and has not gotten her period (the mother had menarche at age 12). Height is 5 ft, weight is 79 lb (BMI 15.46; 3rd percentile), Tanner stage is 1 for breast and pubic hair development. The rest of physical examination is normal, including pelvic exam. Laboratory results are all within reference ranges (CBC, ESR, LFT, basic metabolic panel, and urine HCG). What hormone should be evaluated in this patient?

Intestinal obstruction Perianal fistula Toic megacolon Colon carcinoma Hemolytic uremic syndrome Toxic megacolon - Uler Colitis

A 14-year-old patient presents with a 2-week history of 4-5 loose bowel movements daily with blood and mucus, tenesmus, cramping abdominal pain, and a low-grade fever. These episodes have been occurring intermittently for 6 months. Physical examination shows mild pallor, temperature 99°F, pulse 88/min, blood pressure 100/70 mm Hg. Laboratory analysis shows an elevated ESR and CRP. Perinuclear anti-neutrophil cytoplasmic autoantibodies (pANCA) are positive, and anti-Saccharomyces cerevisiae antibodies (ASCA) are negative. Stool examination for ova and parasites and culture for pathogens are negative. Colonoscopy shows hyperemic, edematous, friable, and ulcerated rectal and colonic mucosa; there is no normal mucosa in between. Upper intestinal endoscopy shows normal mucosa. At this stage, what complication is this patient is at an increased risk of developing?

Cerebrospinal fluid examination CT scan of the head Electroencephalogram Cranial ultrasonography MRI scan of the head CT scan of the head

A 15-month-old toddler presents with sudden onset of generalized tonic and clonic convulsions for the last 30 minutes. Parents report that the patient was irritable in the minutes preceding the events, but there is no history of trauma, fever, or vomiting before the onset of convulsions. This is the first episode of seizure, and there is no history of convulsions in the family but the father has a history of cerebral aneurysm requiring surgical clipping. The patient's birth history, neonatal period, and developmental milestones are normal. After initial stabilization by securing the airway, oxygenation, and controlling the acute convulsion, what is the best investigation for this patient?

Testicular torsion

A 15-year-old boy presents at 11:30 AM due to left scrotal pain and swelling; it started when he woke up at 7 AM. He recalls no trauma. When questioned, he says that he has never had intercourse. He has been feeling nauseated, and he vomited once. Physical examination demonstrates a well-nourished well-developed boy, appearing moderately uncomfortable. Vital signs are normal except for an oral temperature of 37.9°C. Pain assessment score (Wong-Baker scale) is 6/10. Tanner Stage III puberty. The left testicle is approximately 1.5 times the size of the right testicle and high-riding compared to the right. The skin is diffusely erythematous. It is difficult to palpate the scrotum due to tenderness. Cremasteric reflex is absent. Penis is circumcised and appears normal. Scrotal ultrasonography with Doppler ultrasound demonstrates decreased blood flow to the testis. Remaining physical examination is normal. What is the most likely diagnosis?

Drug screen and medication history review Liver function tests and abdominal CT scan Mammogram of both breasts Physical examination including genitalia Thyroid function tests Physical examination including genitalia

A 15-year-old boy presents for a high school basketball physical. After his mother leaves the room, he asks if you can "take a look at something." He then lifts his shirt and tells you that he is developing "man breasts." He is very embarrassed. There is firm mobile tender tissue palpable under both nipples. This tissue is approximately 75 mm in size on the right side and 1 cm on the left. There is no nipple discharge on either side. The patient's height is 66", weight is 125 lb (BMI 20.2). What is the most important next step in the diagnostic workup?

Aspiration of the left knee joint Bone biopsy Routine symptomatic treatment Rheumatoid factor Serial knee X-rays every 4 week Routine symptomatic treatmen (tibial tuberosity - the bump on the shin = Lauryn has this )

A 15-year-old boy presents for a routine physical. He reveals a 1-month history of mildly painful swelling of the anterior superior left shin, unaccompanied by fever, erythema, or joint problems. He plays football for his high school and has been assisting his father a project that entails considerable kneeling that worsened the pain in the affected area. Examination is unremarkable except for mild slightly tender swelling of the left anterior shin approximately 5 cm below the knee. Radiographs of the left knee showed mild irregularity of the tibial tubercle. What treatment recommendation is appropriate?

BRCA1 BRCA2 MSH2 APC RET APC

A 15-year-old boy presents with abdominal pain and rectal bleeding. His family history is significant for the premature deaths of his mother and maternal grandmother from metastatic colon cancer. Both died before age 35; in both, autopsy findings included hundreds of colon polyps along with multiple primary colon cancers. Endoscopy of the boy also demonstrates extensive colonic polyp disease. What gene mutation is likely causing the patient's symptoms?

Appendicitis Cholangitis Clostridioides difficile infection Erythromycin-induced hepatic injury Gentamicin-induced liver injury Erythromycin-induced hepatic injury

A 15-year-old boy was prescribed an antibiotic for acne. While being treated, he developed nausea, vomiting, fever, and jaundice. Liver enzymes are elevated, including AST, ALT, and alkaline phosphatase. What is the most likely cause of his symptoms?

Elevated thyroid-stimulating hormone (TSH) Elevated white blood cell count (WBC) Elevated erythrocyte sedimentation rate (ESR) Reduced luteinizing hormone (LH) Reduced blood urea nitrogen (BUN) Reduced luteinizing hormone (LH) - anorexia nervosa - decrease in horomones and will not have periods

A 15-year-old girl has been dieting for 6 months and has lost over 30 pounds. She tells you that she still feels fat. She is afraid to eat for fear of becoming obese. Her last menstrual period was 3 months ago. On physical exam, the patient is cachectic and slightly pale. Her heart rate is 50 beats/minute, her blood pressure is 90/60 mm Hg, and her temperature is 95.5°F. Her weight is 92 lb and her height is 5'6". What laboratory value is most likely present?

Send stool studies and refer for colonoscopy

A 15-year-old girl presents with a 1-year history of intermittent abdominal pain with nausea and occasional bloody diarrhea. She denies fever and weight loss; there is no travel history. Past medical history is significant only for migraines. She takes a multivitamin. Her vital signs are within normal limits. She has mild diffuse abdominal tenderness to palpation and guaiac-positive stool. Her exam is otherwise normal. Hemoglobin 9.7, hematocrit 28%, WBC 12,000/uL. What is the most appropriate next step in her management?

Metronidazole

A 15-year-old girl presents with a 3-day history excessive vaginal discharge without itching or burning. She denies ever being sexually active. Her last menstrual period was 10 days ago. On examination you find a thin, white, homogeneous discharge that has a distinct amine odor when potassium hydroxide is added. On saline wet mount, epithelial cells are covered with bacteria. What is the most appropriate management of this patient?

Recent acetaminophen use Playing soccer regularly History of cold sores History of frequent ear infections as a child History of tympanostomy tube placement History of cold sores

A 15-year-old girl presents with loss of consciousness. She is accompanied by her mother, who states that the patient initially fell ill several days ago with a headache, muscle aches, and fever. The patient developed a severe headache today, accompanied by double vision, difficulty speaking, confusion, and eventual loss of consciousness. She has not taken any medications aside from acetaminophen (Tylenol) for her fever. Her mother states that her daughter is usually active and had been playing soccer regularly until she became ill; the patient has been fairly healthy aside from occasional cold sores. Past medical history is significant for frequent ear infections as a toddler that were treated with tympanostomy tube placement at age 2. Brain imaging reveals edema of the temporal lobe. What component of the patient's history is most likely contributing to her current illness?

45,XO

A 15-year-old girl presents with short stature and primary amenorrhea; her neck is short and broad, her palate is high-arched, and her genitalia are infantile. Her parents recall several episodes of otitis media during childhood. Her serum FSH is elevated and estradiol is low. What would a chromosomal analysis most likely reveal?

Fluoxetine Duloxetine Atomoxetine Clonidine Methylphenidate Fluoxetine

A 15-year-old male patient presents for follow-up of previously diagnosed autism spectrum disorder. The patient is accompanied by his mother. Symptoms do not interfere too much with school or relationships. He attends public school, goes to daily cognitive/behavioral sessions, and attends weekly counseling sessions. The mother has noted a significant increase in anxiety symptoms that have become detrimentally disruptive to the patient's daily activities. What is the most appropriate initial choice of pharmacologic treatment to help decrease the effects of this patient's anxiety?

Urine analysis Echocardiogram Chest X-ray (AP/lateral) Sickle cell solubility test Blood culture Sickle cell solubility test

A 16-month-old African American boy presents with a 2-day history of irritability and refusal to bear weight. His mother denies any recent history of fever, vomiting, diarrhea, rash, or trauma. Family history is significant for a maternal uncle who had a stroke and died when he was 35. On exam, vital signs are stable. Patient appears smaller than stated age and is irritable but consolable. He is in no apparent distress, but he refuses to bear weight or play. The only significant findings on exam are swollen hands and feet. CBC reveals WBC 18,000 mm3, with 40% neutrophils, 30% lymphocytes, and 1% monocytes. Hb is 8 g/dL and platelets are 400,000 mm3. What test would be helpful in making a diagnosis?

Iron supplements can increase endurance in moderate doses. All menstruating adolescents should take an iron supplement. She does not need an iron supplement. Iron supplements have been banned by regulatory agencies. Iron supplementation have no adverse effects She does not need an iron supplement.

A 16-year old girl on her high school's swim team focuses her exercise regimen on endurance rather than building muscle mass. While her menses are regular and moderate in quantity, she is concerned that she may become iron deficient and that this will negatively affect her endurance and athletic performance. She asked if she should begin an iron supplement. Past medical history was unremarkable, and there was no history of tobacco, alcohol, or recreational drug use. Growth and development have been normal, and immunizations are current. Vital signs are normal. Examination is unremarkable, consistent with Tanner stage V. Hematocrit, hemoglobin, and ferritin were normal. What would you recommend?

Admit patient, obtain cultures, and begin IV ceftriaxone and vancomycin

A 16-year-old African American girl with hemoglobin S disease has experienced several complications, including multiple acute splenic sequestration crises that resulted in a splenectomy when she was 14. She no longer receives antibiotic prophylaxis. Her last polyvalent pneumococcal vaccine was 2 years ago. She presents to the emergency department with a 2-day history of fever up to 104.3°F without localizing symptoms. She notes malaise, abdominal discomfort, and one episode of vomiting. Examination reveals a temperature of 103.2°F; pulse 110; blood pressure 116/76 mm Hg; and respirations 21. Hydration is normal, and no localizing findings are present. What is the next step in the management of this patient?

Nafcillin (Unipen) 2 g IV q 4-6 hours for 6 weeks Vancomycin (Vancocin) 1 g IV q 12 hours for 6 weeks Oxacillin (Bactocill) 2 g IV q 4-6 hours for 2 weeks Ciprofloxacin (Cipro) 400 mg IV q 12 hours for 2 weeks Linezolid (Zyvox) 600 mg IV q 12 hours for 2 weeks Vancomycin (Vancocin) 1 g IV q 12 hours for 6 weeks

A 16-year-old boy is currently an inpatient 7 days after a motor vehicle accident that resulted in multiple lower extremity fractures. The fractures were surgically corrected recently. He is currently immobilized below the waist, but casts will not be placed until the majority of his edema has resolved. He is stable, the edema is resolving, and he is recovering well, with the exception of new onset of increased edema and erythema over his left thigh. X-rays show evidence of acute osteomyelitis. The patient does not have any allergies. A culture from the infected bone has revealed methicillin-resistant Staphylococcus aureus. What treatment regimen is the best choice to treat his infection following surgical debridement?

Look at the fingernails while fingers are extended for position in relationship with each other. Look for finger malposition when the fingers are flexed into the palm. Look for folds in the skin over the metacarpal head. Check for motion of the PIP and DIP joints. Look for palmar edema of the affected digit metacarpal Look for finger malposition when the fingers are flexed into the palm

A 16-year-old boy presents following the striking of a wooden door with a closed fist an hour ago when he was angry at his mother. He is neurovascularly intact, and the skin is closed. There is an obvious deformity with a loss of small finger metacarpal knuckle. Radiographs reveal an oblique mid-shaft fracture of the fifth metacarpal with a palmar angulation of 45°. What physical exam technique must be performed to check for rotation of the fracture?

Enterovirus Varicella virus Syphilis Pneumococcus Meningococcus Meningococcus

A 16-year-old boy presents with acute onset of stiff neck, fever, headache, and vomiting. On exam, he appears lethargic, has limited range of motion of his neck, and a petechial rash is noted. Fever is 103°F. The patient's medical history is non-contributory. What is the most likely etiology of the suspected diagnosis?

Atropine Diphenylhydantoin Methylene blue Pyridoxine Vitamin K Pyridoxine

A 16-year-old female patient presents with a 4-hour history of loss of consciousness. The patient's older sister says the patient has been depressed and was found with an empty bottle of tuberculosis medication. The patient has begun having generalized tonic-clonic seizures. Examination reveals a developed teenage female patient responsive only to painful stimuli; there are intermittent tonic-clonic movements of the extremities. Besides the general care of an unconscious patient with seizures, what should be given as a specific antidote?

Primary dysmenorrhea Endometriosis Irritable bowel syndrome Imperforate hymen Ovarian cysts Primary dysmenorrhea

A 16-year-old girl has had recent onset of painful menstrual periods. Menarche was at age 12. Her periods were irregular for about 8-10 months. They have been fairly regular since then, with occasional mild crampy pains on the first 2 days. She rates the pains now as being 8-9 on a scale of 10. Cramps will start about a half a day before the onset of her periods, worsen the following day, and then gradually subside over the next day. She also describes having a headache, looser stools, mild nausea, and low back and thigh pain. Bleeding is moderate on the first 2 days. She is otherwise healthy without history of abdominal surgery. Privately she denies any sexual activity. She is an average student and has missed 1-2 days of school with each period for the past 3 months. She also denies any fever or dysuria. There is also no family history of gynecological problems. On exam, she has diffuse midline lower abdominal tenderness with some mild soreness to lower back and thighs, bowel sounds are normal, and there is no rebound tenderness. An external genital exam is normal. What is the most likely diagnosis?

Vitamin B12 Vitamin C Vitamin D Folic acid Iron Vitamin D (fat soluble) Fat soluble AEDK

A 16-year-old girl presents for continuing concerns about her weight. She has a family history of diabetes and has tried to lose weight before without success. Her BMI (body mass index) is at the 99th percentile for age, and she is given a trial of orlistat. Within 2 weeks, she returns to her pediatrician with bowel movement changes, flatulence, oily discharge, and mild abdominal pain. She lost 5 lb and would like to continue the treatment if the side effects could be eliminated. In addition to reducing fat consumption, what supplementation will she most likely require?

Anisocytosis

A 20-month-old boy presents for a routine visit. He is eating well and drinking at least a bottle of cow's milk daily, which he has done since age 8 months. His history and physical are normal except for mild pallor. Urinalysis is normal. ResultNormal ValuesHemoglobin9.510.5-14.0 g/dLRDW5238.5-49.0%MCV7276.0-90.0 flPlatelets280,000150,000-450,000/μLWBC6.75000-14,500/μL What additional finding might you expect in this child?

Psychogenic non-epileptic seizure (pseudoseizure) Transient ischemic attack Syncope Panic attack Tonic-clonic seizure Psychogenic non-epileptic seizure (pseudoseizure) Asynchronous movements increased with restraint, preceding stressor, and normal objective testing are characteristic of PNES

A 16-year-old girl presents to the emergency room for evaluation of continuous seizures. The patient has a history of idiopathic epilepsy and takes valproic acid. Two previous EEGs have been normal. This spell occurred at the dinner table after a fight with her mom. Physical exam reveals a well-developed well-nourished young woman lying on a gurney; there is continuous but asynchronous motor jerking of all four extremities. Restraints are in place for patient safety, which causes increased intensity of movements. The head intermittently turns from side to side. Eyes are closed tightly and cannot be passively opened. Deep tendon reflexes cannot be assessed. The response to plantar stimulation is withdrawal bilaterally. Exam otherwise normal. Stat EEG and labs are normal, including CBC with differential, UA on catch specimen, BMP, and arterial blood gas. Chest x-ray is normal. Postictal serum prolactin and creative kinase levels are normal. What is the most likely diagnoses?

Reduced gonadotropin-releasing hormone from hypothalamus

A 16-year-old girl presents with concerns over her "growth." She feels ashamed and left out because she is the only girl in her class who has not reached menarche. Her mother reached menarche at age 11, and her younger sister at 10. She is sexually active with her boyfriend of 1 year; she has always used contraception. On examination, her vital signs are normal. Physical examination reveals rudimentary breast buds at Tanner stage 2 and an absence of pubic and axillary hair. A pelvic sonogram shows a normal-appearing vagina and uterus. An MRI of the brain shows a normal-appearing pituitary gland and hypothalamus, with agenesis of the olfactory bulb. What is the pathophysiology of this condition?

Intramuscular methylprednisolone Oral hydrocodone Oral prednisone Oral acetaminophen Topical diclofenac gel Oral acetaminophen (hamstrings)

A 16-year-old male high school junior (baseball shortstop) presents with left hamstring soreness for 4 days. He felt a slight pull during infield practice. He was late that day and skipped his usual warmup and stretching routine. Despite rest, the soreness has persisted. He wants medication for the discomfort, but he mentions that ibuprofen and naproxen have caused gastric upset in the past. Past medical history is otherwise unremarkable, growth and development are unremarkable, and immunizations are current. Vital signs are unremarkable. Examination reveals mild tenderness to palpation of the left hamstrings, aggravated by extension at the knee. What is the most appropriate choice of drug therapy?

Pubertal delay Chromosomal abnormality Pregnancy Hypothalamic amenorrhea Gonadal dysgenesis Pregnancy

A 16-year-old otherwise healthy girl presents with cessation of her menstrual cycle. She does not take any medications or use tobacco or drugs. She has a boyfriend, but she denies intercourse. Her family history is negative for gynecologic or fertility problems, autoimmune diseases, and endocrinopathies; her mother's and female relatives' menarche presented at age 12-14. There are no syndromic features on examination. Her height, weight, and BMI are within normal range, and her vital signs are normal. Her skin is clear (no acne), and there are no signs of hirsutism. There is no thyromegaly. Her breasts are developed, and pubic and axillary hair is present. Her abdomen is benign; her external genitalia maturated; there is no clitoromegaly; vaginal mucosa is moist and pink; saline-moistened applicator swab reveals normal vaginal length. Single finger examination demonstrates the presence of uterus. Neurological examination is non-focal; sense of smell is preserved; visual fields are normal by confrontation; fundoscopic examination shows no papilledema. What will you consider first in further evaluation of amenorrhea in this girl?

Breast abscess Cystosarcoma phyllodes Fat necrosis Fibroadenoma Fibrocystic disease Fibroadenoma This presentation suggests fibroadenoma, the most common benign solid tumor containing glandular and fibrous tissue

A 16-year-old patient presents with a painless mass in the left breast. The lesion does not change in size during the menstrual cycle. Physical examination reveals a non-tender mass on the upper outer part of the left breast measuring about 2 × 3 cm. Aspiration is performed, but no fluid is aspirated. An excisional biopsy is contemplated. What is the most likely diagnosis?

Cerebral contusion Dysthymia Complex partial seizures Postconcussive syndrome Subdural hemorrhage Postconcussive syndrome

A 16-year-old previously healthy boy fell on a cemented surface while skateboarding 3 weeks ago. He lost consciousness for 2 minutes. He was not wearing a helmet. A CT scan showed no abnormalities. He is now back at school and has difficulty concentrating in class and has frequent spells of dizziness and headaches. His neurologic exam is normal. What is the most likely cause of his symptoms?

Cervical ectopic pregnancy Choriocarcinoma Pseudocyesis Tubal ectopic pregnancy Early intrauterine gestation Tubal ectopic pregnancy

A 16-year-old sexually active girl is seen for a 2-month history of amenorrhea. She denies unprotected sex but relies on her partner to use a condom. She has vomited in the early morning twice this week and had vaginal spotting for 3 days accompanied by cramping lower abdominal pain that became sharp. Onset of menses was at age 12, with normal regular periods since then. No history of sexually transmitted disease. Normal vital signs. Slight right and left lower quadrant abdominal tenderness without guarding and rebound is present. Cervix is closed. No blood is seen in the vaginal vault. The uterus is not palpable. Serum β-HCG: 5200 mIU/mL. Vaginal spotting has increased, and abdominal pain has become more frequent. Repeat examination 3 days later is unchanged. The uterus is still not palpable. Repeat serum β-HCG is 6800 mIU/mL. Transvaginal ultrasound fails to reveal an intrauterine pregnancy or gestational sac. What is the most likely diagnosis?

Releases dopamine and norepinephrine stores Enhances effect of GABA Interferes with sodium and potassium transport Blocks the effect of serotonin Blocks binding of acetylcholine to nicotinic receptors Releases dopamine and norepinephrine stores (drugs makes you happy)

A 16-year-old slightly overweight girl presents because she wants to lose weight. She read online that drugs for ADD can help with weight loss. She is unsure of the name of the drug she read about but wonders if she can use them as diet pills instead. Since she was told that drugs like meth have similar properties, she also wants to know if these are safe to take for weight loss. She is told to avoid them because they can become habit-forming and their effectiveness is short-lived. What is the mechanism of action of the drug she is referring to?

Perform thymectomy. Add corticosteroids. Perform plasmapheresis. Withdraw pyridostigmine. Add neostigmine Withdraw pyridostigmine (myasthenia gravis)

A 17-year-old boy develops progressively abnormal muscle fatigability. He is diagnosed as having myasthenia gravis and is admitted to a hospital. In the course of his treatment with pyridostigmine, he develops increased weakness, nausea, vomiting, sweating, and bradycardia. What is the best management for this patient?

Heart palpitationsA 5-year-old boy presents with his mother with a history of fever, hemorrhages, and repeated bacterial infections. On exam, lymphadenopathy and hepatosplenomegaly are present. Blood work shows a white blood cell count of 50,000/µL. Question What is the most likely diagnosis? History of lymphoma On doxycycline for 6 months Craving to eat ice Systemic lupus erythematosus Craving to eat ice

A 17-year-old boy presents for a follow-up regarding fatigue and dizziness. After his initial presentation, he had some general blood work drawn, including a complete blood count (CBC) showing a low hemoglobin and hematocrit. MCV is low. What detail would support your suspected diagnosis if it were present in the patient history?

Endometriosis Ectopic pregnancy Von Willebrand disease Disseminated intravascular coagulation Hemophilia A Von Willebrand disease

A 17-year-old female patient presents with heavy periods. Past medical history is unremarkable. There is no history of surgeries. She has had heavy monthly menses since menarche at age 13. She admits to using more than 20 tampons on each of the heaviest 3 days of the menstrual cycle. Last menstrual cycle started 1 week ago. There are no recent medications. Exam is unremarkable, with no vaginal discharge. Pregnancy test is negative. Cervical cultures are negative. Thyroid studies are unremarkable. Hb 10 g/dL, WBC 9000/mcL, platelet count 250 × 103/mcL. Peripheral blood smear is consistent with microcytic hypochromic anemia. PT is normal, aPTT is prolonged, and bleeding time is prolonged. What is the most likely diagnosis?

"You can continue to play if you are pain-free."

A 17-year-old football player (punter) was tackled and forcefully slammed onto his right dominant shoulder 2 hours ago during a game. He had immediate pain, but it has subsided; he now experiences the same level of pain when he carries a heavy object. He wants to continue punting. He has full active and passive range of motion, but some pain (4/10) with abduction. There is no obvious deformity, and the skin over the shoulder is intact and not tented. There is no crepitus on palpation while moving the arm. Most of his tenderness seems to be on the superolateral right shoulder. What recommendation should be made to this patient?

Decreased c-reactive protein Increased b-natriuretic protein Increased erythrocyte sedimentation rate Decreased alkaline phosphatase Increased uric acid level Increased erythrocyte sedimentation rate (arthritis = inflammation)

A 17-year-old male college freshman presents with fatigue, back pain, and stiffness. The pain has been present for the past several months, but it appears to be worsening. The back symptoms are worse at night and first thing in the morning; they improve somewhat during the day. He reports the pain improves with exercise. He is having difficulty staying productive at school because he is always tired. What laboratory finding is most probable?

Tenderness over the greater tuberosity Tenderness over the acromioclavicular joint Crepitus with motion Decreased radial pulse Paresthesia over the elbow Tenderness over the acromioclavicular joint (AC separation)

A 17-year-old male high school football player presents after being tackled and slammed onto his right dominant shoulder forcefully 2 hours ago during a game. He had immediate pain but was able to continue playing. He has full active and passive range of motion, but some pain (4/10) with abduction. There is no obvious deformity, and the skin over the shoulder is intact and not tented. What finding do you expect on physical exam?

Ultrasound CT without contrast Plain film radiograph MRI without contrast MR arthrography Plain film radiograph (initial radiographic)

A 17-year-old male high school football player presents after being tackled and slammed onto his right dominant shoulder forcefully 2 hours ago during a game. He had immediate pain but was able to continue playing. He has full active and passive range of motion, but some pain (6/10) with abduction. There is no obvious deformity, and the skin over the shoulder is intact and not tented. What initial radiographic procedure should be ordered?

Clavicle fracture Sternoclavicular dislocation Adhesive capsulitis Glenohumeral dislocation Acromioclavicular separation Acromioclavicular separation

A 17-year-old male high school football player was tackled and forcefully slammed onto his right dominant shoulder 2 hours ago during a game. He had immediate pain but was able to continue playing. He has full active and passive range of motion, but there is pain (4/10) with abduction. There is no crepitus on palpation while moving the arm. Most of the tenderness seems to be on the superolateral right shoulder. What is the most likely diagnosis?

Narcissistic Dependent Borderline Histrionic Paranoid Borderline

A 19-year-old female student is referred for psychiatric assessment after an attempted suicide 2 days ago. On questioning, the patient informs that the precipitant to her attempted suicide was a recent breakup with her boyfriend. Her history is significant for multiple suicide attempts since the age of 12. On further questioning, the patient informs that she has a chronic feeling of emptiness and sometimes feels disconnected from reality. She also gets into fights with her boyfriends, as she has difficulty controlling her anger. The patient admits that she has had highly problematic relationships with men and that she falls quickly in and out of love. Examination reveals several cuts on the wrists and arms. What personality disorder does this patient have?

Tenderness to palpation directly over the acromioclavicular joint Loss of surface contour of the shoulder Full passive range of motion Ecchymosis over the clavicle extending to the shoulder Full active motion of the shoulder Loss of surface contour of the shoulder

A 19-year-old male patient presents with pain in the right dominant shoulder, which was injured when he was blocked while shooting a basketball. Physical examination includes significant pain and resistance to passive arm movement. The arm is slightly abducted and externally rotated to minimize pain. What finding is expected on physical exam?

Central sleep apnea Insomnia Medication side effect Narcolepsy Obstructive sleep apnea Narcolepsy

A 19-year-old man presents for evaluation of excessive daytime sleepiness. Despite getting a full night's sleep, his coworkers have repeatedly noticed him suddenly dozing off at his desk, during meetings, and sometimes mid-conversation. Upon further questioning, the patient admits that he occasionally experiences nighttime episodes of feeling unable to speak or move. The remainder of the history and physical exam are unremarkable. Aside from the occasional ibuprofen for shoulder pain, he does not take any medications. The patient's symptoms are most consistent with what disorder?

Rotator cuff tear Shoulder separation Glenohumeral dislocation Fractured clavicle Sternoclavicular subluxation Glenohumeral dislocation

A 19-year-old man presents with pain and deformity of his right dominant shoulder after a sudden jerking movement to that shoulder from a wrestling competitor approximately 1 hour ago. He was unable to continue wrestling and has pain with any movement of the right shoulder. On exam, you see a loss of normal shoulder contour anteriorly. There is no focal joint or bony tenderness. What is the most likely diagnosis?

Anterior/posterior, scapular lateral, and axillary radiographs

A 19-year-old man presents with pain and deformity of his right dominant shoulder after a sudden jerking movement to the same shoulder from a wrestling competitor approximately 1 hour ago. He states he felt a clunking sensation when it happened. He was unable to continue wrestling and has pain with movement of the right shoulder. What initial imaging should be the next step?

Histamine 1 (H1) Serotonin (5-HT2A) Dopamine (D2) Muscarinic cholinergic (M1) Alpha-adrenergic (α-1) Dopamine (D2)

A 19-year-old man with a family history of schizophrenia is receiving medical attention for his first presentation of psychosis. After ruling out organic causes and substance abuse as etiologies of his symptomatology, antipsychotic therapy with haloperidol is initiated. Within 48 hours, the patient begins to experience involuntary spasmodic contractions of the muscles in his face and neck. Inhibition of what neurotransmitter/receptor is causing the patient's symptoms?

Pyelonephritis Salpingitis Cystitis Gastroenteritis Diverticulitis Pyelonephritis --> fever, and flank pain

A 19-year-old woman presents with a 4-day history of fever accompanied by chills, nausea, vomiting, and back pain. She denies any chest pain, cough, or urinary symptoms. On further questioning, she said she had 2 episodes of diarrhea yesterday with vague abdominal discomfort. She has no other significant past medical history, is on no medications, and has no allergies. Family history is significant for hypertension in father and arthritis in mother. She is single, has no children, does not smoke or drink, and works part time as a waitress. On exam, she has a temperature of 102.4°F, pulse of 110/min, BP 110/60 mm Hg, and SPO2 of 92%. Mucous membranes are dry, and sclera is clear. Lungs are clear, and heart sounds are normal. There is mild left flank tenderness and tenderness in the left costovertebral angle. Labs: Hb 12g/dL, WBC 17,000/uL, bands were 18% and platelets 350,000/uL. Chest X-ray and EKG are normal. Urinalysis shows 35 WBC, 6 RBC, and no casts. What is the most likely diagnosis?

Cetirizine Clopidogrel Hydrocodone Sumatriptan Topiramate Topiramate

A 19-year-old woman presents with a multi-year history of episodic throbbing headaches. They have intensified, and she now misses classes and work 4-6 times monthly. They last 2-3 days, accompanied by nausea, vomiting, and light sensitivity. She denies residual symptoms after headaches resolve. She denies neurologic symptoms such as vision or taste changes, gait disturbances, and memory loss. She has tried multiple over-the-counter pain medications without relief. Her mother and maternal aunt experienced similar headaches. She reports some increased stressors and less sleep since starting college. Past medical history is unremarkable, with no other known medical conditions, surgeries, chronic medications, or drug allergies. She has never been sexually active and reports regular menses. She denies tobacco, alcohol, and recreational drugs. Physical exam is unremarkable. What medication would be most effective in preventing this patient's recurrent symptoms?

Evaluate vaginal discharge under microscopy. Perform a Pap smear. Instruct patient how to douche. Perform a vaginal ultrasound. Perform a colposcopy Evaluate vaginal discharge under microscopy.

A 19-year-old woman presents with vaginal pruritus, dysuria, and dyspareunia. On exam, a profuse frothy yellowish-green vaginal discharge with reddened vaginal mucosa is noted. What is the next step in formulating a diagnosis?

Sleep terrors

A 2-year-old boy presents because of his mother's concerns about his sleep. She notes that he is easy to put to sleep, and he has a regular bedtime of 7:30 PM. Most nights for the past 3 weeks, about 90 minutes after being put to sleep, he begins to thrash violently in bed, sometimes letting out bloodcurdling screams. He has his eyes open and seems to be talking, but he does not respond to either parent when spoken to. He sweats a lot. The episodes last about 15 minutes, and he then goes back to sleep; he seems fine in the morning. He naps for about an hour in the morning and an hour in the afternoon, but these episodes do not occur with naps. What is the most likely diagnosis?

Inflammation on temporal artery biopsy Normal magnetic resonance imaging (MRI) Slow wave pattern on electroencephalogram (EEG) Positive toxicology for cocaine Thickened sinus mucosa on computed tomography (CT) Normal magnetic resonance imaging (MRI) (Mirgraine - normal imagining)

A 19-year-old woman presents with worsening headaches. She reports a multi-year history of episodic throbbing headaches. They have intensified, and she now misses classes and work periodically as a result. The headaches occur about 4-6 times per month recently, up from 1-2 per month when she first started experiencing them. Her headaches last 2-3 days and are accompanied by nausea, vomiting, and light sensitivity. After the headache resolves, she denies any residual symptoms. She denies neurologic symptoms, such as vision or taste changes, gait disturbances, and memory loss. She has tried multiple over-the-counter pain medications without relief. Her mother and maternal aunt experienced similar headaches. She reports some increased stressors and less sleep since recently starting college. Her past medical history is unremarkable, with no known medical conditions (except for the headaches), no surgeries, no chronic medications, and no drug allergies. She has never been sexually active and reports regular menses. She denies the use of tobacco, alcohol, and recreational drugs. On physical exam, the patient appears comfortable and reports no headache at this time. Her entire exam, including neurological, is normal. What test result best supports the most likely diagnosis?

Folate deficiency Vitamin K deficiency Breastfeeding Infection Normal process Normal process

A 2-month-old infant who is breastfeeding presents with low hemoglobin levels. The infant was born at home and the mother received no prenatal care; she did not, and does not, take any medications. Family history is unremarkable. On examination, the infant appears healthy. What is the underlying mechanism of low hemoglobin levels in this infant?

Hydrocele

A 2-week-old male infant presents with his father for evaluation of enlarged scrotum. The father states that the scrotum was a little larger in the first few days after birth than it is now, but it has not reduced in size enough to make him feel comfortable that it is normal. Physical examination reveals normally developed penis with abnormally large scrotum that transilluminates on the right side when light is shined on it. What is the most likely diagnosis?

Radiation and surgery is the most optimal treatment plan. Combination chemotherapy is appropriate after surgery. Observation only is appropriate for this patient. Chemotherapy only is appropriate for this patient. Surgery and no further intervention is best Combination chemotherapy is appropriate after surgery

A 2-year-old boy has a solid mass of tumor on the lower pole of his right kidney. Histopathological studies show whorls of mesenchymal cells and clusters of disorganized epithelial cells rather than renal corpuscles and renal tubules, which confirm your suspected diagnosis. Question What is true regarding treatment and/or prognosis of this diagnosis?

Decrease milk consumption

A 2-year-old boy presents with failure to gain weight. He is the same weight he was at his 18-month well check. His mother says that she offers him three meals per day and three snacks, but he usually just picks at them. He does not seem hungry. He does enjoy chicken nuggets, Goldfish crackers, and cookies. She gives him a cup of juice each day and around 40 oz of whole milk. He was a full-term delivery and has no significant past medical history. He was fed breast milk exclusively until age 1. He was then switched to whole milk and solids were introduced. Other than his failure to gain weight, his physical exam is normal. He is gaining adequately in height and head circumference. He is reaching age-appropriate milestones and appears well, without any sign of systemic disease. How should you advise the mother?

Migraine

An 18-year-old woman presents with a history of recurrent headaches that mainly occur above her right eye. She describes them as "throbbing" in quality. During these headaches, she gets bouts of nausea and vomiting. After a nap, she typically feels better. The headaches only last a few hours, and she experiences them roughly 2 times a month. What is the most likely diagnosis?

Bleeding time Complete blood count (CBC) Complete metabolic panel (CMP) Factor V Leiden Pap smear Factor V Leiden - coagulation issue

A 20-year-old Lebanese woman wants to start birth control. She has never been sexually active, and her wedding is in 2 months. She feels well and reports no issues. She thinks she wants "the pill." Her fiancé is also a virgin, and they are not interested in condoms or other barrier contraception. She wants to delay childbearing for at least 2 years. Past medical history includes occasional OTC ibuprofen for menstrual cramps and headaches, penicillin allergy, previous tonsillectomy, menarche age 12, and regular menses with mild-moderate dysmenorrhea. Her older sister had a blood clot in her lung after giving birth. Her paternal grandfather has diabetes and hypertension, maternal grandmother had a stroke, mother had a DVT in a leg, and maternal grandfather had prostate cancer. She denies tobacco, alcohol, and recreational drug use. Vitals: 123 lb, 5'4", BMI 21.1, pulse 88, BP 134/86, temperature 97.9°F. What test should be ordered before prescribing birth control pills?

Antibiotic H2-receptor antagonist Multivitamin H1-receptor antagonist NSAID NSAID

A 20-year-old female college student presents due to a 7-day history of daily heartburn. She has never experienced heartburn as bad as she does currently. She is treating it with over-the-counter histamine-2 receptor antagonist, famotidine, 1 tablet daily. The patient denies any other significant past medical history and is currently taking a daily multivitamin and an antihistamine for seasonal allergies. She recently admits to having increased episodes of headaches that she believes are due to stress. For this reason, she has been taking ibuprofen 600 mg every 8 hours. She states that she has been taking this consistently every 8 hours for the last 10 days; she hopes that her headaches will subside after finals are finished. She also states that she was given amoxicillin 2 weeks ago for a middle ear infection, which resolved without any further intervention. She denies any difficulty swallowing, weight loss, night sweats, chest pain, black tarry stool, use of tobacco or alcohol, or coughing up blood. Physical examination is unremarkable. What medication is most likely causing her increased gastrointestinal symptoms?

Antibiotics Anticholinergic Proton pump inhibitor H1-receptor antagonist Acetaminophen Acetaminophen

A 20-year-old female college student presents due to a 7-day history of daily heartburn. She has never had this as bad as she does currently. The patient denies any other significant past medical history and is currently taking only a multivitamin daily. She admits to recently having increased episodes of headaches that she believes are due to stress. For this reason, she has been taking ibuprofen 600 mg every 8 hours. She states that she has been taking this consistently every 8 hours for the last 10 days. She hopes that after finals are over her headaches will subside. She also states she was given a 10-day course of amoxicillin 2 weeks ago for a middle ear infection, which resolved without any further intervention. She denies any difficulty swallowing, weight loss, night sweats, chest pain, black tarry stools, use of tobacco/alcohol, or coughing up blood. Physical examination is unremarkable for any abnormalities. What medication would you suggest for the patient to begin for her headaches at this time?

Cluster headache Giant cell arteritis Intracranial mass Migraine headache Tension headache Tension headache (back of the head and leaves)

A 20-year-old female college student presents to the student health center with a 1-week history of daily headaches. She has no significant past medical history. Upon further questioning, she admits to headaches occasionally over the past 2 years, but it is lasting longer this time. She classifies the pain as a 4/10 that is generalized, but is worse in the back of the head. The headaches are not debilitating or throbbing, and she describes it as an annoying pain. Acetaminophen helps somewhat but does not make the headache go away completely. On physical exam, there are no neurologic deficits, vision is 20/20 uncorrected, and vital signs are within normal limits. She does note tenderness upon palpation of the muscles of the head, neck, and shoulders. What is the most likely diagnosis?

Neisseria meningitidis (sounds like meningitis)

A 20-year-old male college student presents with fever, chills, malaise, headache, photophobia, confusion, and numerous petechiae on his extremities and trunk. On examination, he has positive Brudzinski and Kernig's signs. A CT scan of the head is within reference limits. A lumbar puncture reveals increased leukocytes, particularly polymorphonuclear neutrophils, increased protein, and decreased glucose levels. Gram staining of the CSF reveals Gram-negative cocci in pairs. What is the most likely causative agent for the infection in this patient?

Strict bed rest at home Intravenous phenytoin Immediate induction of labor Immediate cesarean section Injection of betamethasone Injection of betamethasone

A 20-year-old primigravida presents at 30 weeks gestation with a 2-day history of a headache, decreased urine output, and facial puffiness. On examination, vitals are as follows: pulse 90/min, blood pressure 164/116 mm Hg and 166/114 mm Hg (taken 6 hours apart), RR 20/min. There is generalized edema and exaggerated deep tendon reflexes with presence of clonus. Abdominal examination reveals a fundal height corresponding to 30 weeks gestation and the presence of good fetal heart sounds (FHS). Urine dipstick for protein reveals 3+proteinuria (300 mg/dL) on 2 occasions. What is the next step in the management of this patient?

High-fiber diet and stool softener

A 20-year-old woman presents with 2 weeks of anorectal pain. She notes streaks of blood on her stool and toilet paper. She reports "a tearing pain during each bowel movement." She dreads having a bowel movement, and she attempts to hold it as long as she can. She broke her leg in a skiing accident 4 weeks ago and was prescribed oxycodone/acetaminophen (Percocet) for the first few days due to her pain. An anoscope reveals an acute anal fissure. What should be the next step in treatment?

Rapid influenza test N-acetylcysteine Oral rehydration solution Urine hCG Urine drug test Urine drug test

A 20-year-old woman presents with a rapid onset of nausea, headaches, fast breathing, and dilated pupils. Her sister thinks that the patient is exhausted from staying up several nights to study for final exams. The patient refused food several times, arguing that she did not have time to spend on anything but her studying; she even became aggressive when her roommates insisted on her going out to dinner with them. Upon physical examination, the patient seems tired and has a dry mouth; her body temperature is 99.8°F and her blood pressure is 135/85. The patient states that she took 2 acetaminophen pills the night before and in the morning to get rid of her headaches. What is the next step in the management of this patient?

Vitamin K deficiency

A 20-year-old woman presents with intermittent nose bleeds for the past 2 weeks. She also reports that her menstrual periods have increased in number in the past 2 months. She recently underwent surgery for small bowel resection and eats only one meal a day. Laboratory investigations reveal prolonged prothrombin time, prolonged activated partial thromboplastin time, and a normal platelet count. What is the most likely diagnosis?

Antisocial personality disorder (longer than 18)

A 21-year-old male high school dropout recently set a fire in his old school classroom because he was dared to do so by friends. He has been sent to the provider for evaluation. The provider interviewed him and finds that he had several problems related to truancy and fighting in school; he has been found with liquor in his locker at school, and he always seems to feel that someone else is responsible for his misbehavior. When he was 10, the patient burned down a barn; last year, he and some friends threw rocks at passing cars on the expressway. Past medical history is non-contributory. He denies any recreational drug use. What is the most likely diagnosis?

Bicipital tendinitis Acromioclavicular arthritis Rotator cuff tendinitis Complete rotator cuff tear Adhesive capsulitis Rotator cuff tendinitis (tenderness just under the acromion, limited abduction of the left shoulder)

A 21-year-old male patient presents with a 3-month history of pain in the left shoulder. He is a left-handed pitcher for the college baseball team. The pain began when he started spring training a few months ago, and it has gradually worsened. He is now having difficulty lifting the left arm above the head. He denies any injury to the shoulder, and he states that ibuprofen provides some relief. On physical exam, the patient has tenderness to palpation of the lateral left shoulder just under the acromion, limited abduction of the left shoulder, a negative drop-arm sign, and a negative crossover test. What condition best explains this patient's symptoms?

Refer for physical therapy Manage pain with opiate analgesics Prescribe a muscle relaxant Obtain MRI of the shoulder Refer for orthopedic surgical therapy Refer for physical therapy

A 21-year-old male presents with a 3-week history of pain in his left shoulder. The pain began when he started spring training and has gradually worsened. In addition, the patient is now having difficulty lifting the arm above his head. He denies injury to the shoulder itself; he states that ibuprofen provides some relief. On physical exam, the patient has tenderness to palpation of the lateral left shoulder just under the acromion, limited abduction of the left shoulder, and a negative drop-arm sign and crossover test. What is the next appropriate step in the management of this patient?

A positive crossover test Tenderness to palpation along the anterior aspect of the left humerus Tenderness to palpation just under the acromion A positive Speed's test Palpable crepitus during range-of-motion maneuvers Tenderness to palpation just under the acromion

A 21-year-old man presents with a 3-month history of pain in his left shoulder. He is a left-handed pitcher for his college baseball team and states that the pain began at spring training and has gradually worsened since that time. In addition to the pain, the patient is now having difficulty lifting his left arm above his head. He denies any injury to the shoulder itself and states that ibuprofen provides some relief. You suspect rotator-cuff tendinitis. What physical exam finding would you expect to find in this patient?

Major depressive episode Manic episode Mixed episode Bipolar I disorder Cyclothymic disorder Bipolar I disorder

A 21-year-old woman is markedly thin, and she describes being worried she is going to fail her college finals and not graduate. She presents because she cannot eat or sleep; she feels like she would be better off dead. She was doing well in classes until after a spring break trip to Europe. When queried about previous psychiatric treatment, she tells you that she went "a little nuts" when she was a freshman; it was her first semester and the first time she had lived so far from home. She stayed up for a week trying to write a novel, and during that time she bought two computers and a whole new wardrobe with her father's credit card. What is the most likely diagnosis?

Metformin (Glucophage

A 21-year-old woman presents with urinary frequency. Her BMI is 41. A urinalysis is positive for glucose. Her random blood sugar is 257 mg/dL, hemoglobin A1c is 8.5%, and C-peptide is 1.5 ng/mL. She is diagnosed with type 2 diabetes; diet and exercise are recommended. What is the drug of choice for managing her diabetes?

Eye pain and conjunctival injection Hematuria and dysuria Positive straight leg raise test Subcutaneous nodule formations Warm and swollen hip joints Eye pain and conjunctival injection (ankylosing spondylitis (AS))

A 22-year-old Asian man with no significant past medical history presents with a dull pain. It is insidious in onset and has been felt deep in the lower lumbar gluteal region for 5 months. It is accompanied by low-back morning stiffness that lasts up to a few hours. The stiffness improves with activity and returns with inactivity. For 2 months, the pain has ascended; it has become more persistent and bilateral. He notes that nocturnal exacerbation of pain forces him to rise and move around. He admits to bilateral hip and ankle pain, anorexia, malaise, and intermittent low-grade fever. Physical exam reveals a stooped forward-flexed position, limitation of anterior and lateral flexion, limited extension of the lumbar spine, positive Schober test, restricted ranges of motion of the hips, and reduced chest expansion. Lower back and hip x-ray shows bilateral SI joint erosion with narrowing of the joints. What additional clinical manifestation is most likely in this patient?

Hypothalamus Spinal cord Peripheral nerves Cerebellum Cranial nerves Hypothalamus

A 22-year-old female college student presents because she does not eat properly and has missed several menstrual cycles. Her sorority sisters are certain she is not pregnant because she rarely, if ever, leaves their sorority house except to attend classes; she has not dated in more than 6 months. On examination, she is underweight. She walks unaided and her speech is clear and distinct. She has adequate vision, normal-appearing facial expressions, and adequate hearing. On her college entrance physical examination, her height was 5'7" and her weight was 130 lb. Her weight 1 year later is now 103 lb. Question What area of her nervous system is involved with her disorder?

Dissociative identity disorder (distinct personalities)

A 22-year-old woman presents for psychiatric follow-up. During the visit, she begins to reveal what appear to be distinct personalities. Identities appear to repeatedly take control of the patient's behavior and affect. You know she is an incest survivor. She denies any drug or alcohol use. You have tentatively diagnosed her with a post-traumatic stress disorder, but what other condition might she have?

Azithromycin 1 g PO once and ceftriaxone 500 mg IM once Treat for both gonorrhoeae and chlamydia

A 22-year-old female patient presents with increased vaginal discharge. They are sexually active with two male partners, and the patient uses birth control pills for contraception. Last menstrual period was 12 days ago, and there has been an increased whitish vaginal discharge for 1 week. Physical exam reveals a soft non-tender abdomen. On pelvic exam, there is a light-yellow cervical discharge with erythema of the cervical os. There is no cervical motion tenderness and no adnexal masses or tenderness. Wet mount of the vaginal discharge reveals epithelial cells and WBCs; no yeast or protozoa. Gram stain of the vaginal discharge reveals many leukocytes. Urine pregnancy test is negative. What is the best treatment for this patient?

Lenalidomide and prednisone Doxorubicin, bleomycin, vinblastine, and dacarbazine Radiation therapy Stem cell transplant Observation Doxorubicin, bleomycin, vinblastine, and dacarbazine

A 22-year-old man presents to his physician with swollen lymph nodes in the right axilla. He notes that he develops pain in the area after drinking alcohol. He has been feeling fatigued for the last few weeks and has lost weight without trying. Examination confirms lymphadenopathy in the right axilla. Biopsy of the region shows the presence of Reed Sternberg cells. What is the treatment of choice at this time?

Ultrasound

A 22-year-old man presents with a right groin bulge. During physical assessment, a single sac is found protruding just lateral to the epigastric vessels. Although the clinical scenario is highly suggestive of a hernia, what initial imaging study would be the best choice to support this diagnosis?

Pott's disease Bacterial meningitis Viral syndrome Tetanus Lymphoma Viral syndrome

A 22-year-old man presents with a temp of 101°F, a headache, stiff neck, and some limitation of flexion of his neck for the past 2 days. The patient also has generalized myalgia for the past 3 days. The patient was advised rest, adequate hydration, and treatment with antipyretics. The recommended treatment caused the resolution of symptoms. What is the most likely diagnosis?

Postpartum depression Postpartum psychosis Normal puerperium Dysthymia Postpartum blues Postpartum blues (mild condition)

A 22-year-old postpartum woman presents with a loss of interest in eating, increased sleepiness, and fatigue at her first-week checkup. It was her first pregnancy. She has been happily married for 2 years; she has a stable job and good family support. Out of her hearing range, her husband reports that she seems more irritable, snapping at him for trivial things. She broke down sobbing when asked about her parents, who passed away in a car accident 1 year earlier. What is the most likely diagnosis?

Continue current exercise regimen

A 22-year-old primiparous woman at 10 weeks gestation presents to the office. She works in an office, but she maintains a regimen of swimming and moderate weightlifting 3-4 times per week. She has had no vaginal spotting, and the pregnancy is progressing normally. She would like to continue with her exercise routine but is concerned about reports from friends that exercise has an adverse effect on pregnancy. Past medical history is otherwise unremarkable. Vital signs are normal, and examination is consistent with a 10-week gestation. How should this patient be advised?

Posttraumatic stress disorder

A 22-year-old woman has a history of being involved in a bank robbery 8 months ago. Since then she noticed she has been having loss of memory, flashbacks, disruptive sleep and nightmares recalling the event, irritable moods, and difficulty concentrating. She denies any palpitations, tachycardia, or any chest pain. She denies any repetitive behaviors or feeling uncomfortable in social settings. She also denies any need for order or symmetry, unwanted intrusive thoughts, or scrupulosity. The patient does not mention any concerns about excessive worrying. Upon physical exam, the patient appears disheveled, with poor hygiene and flattened affect. She does not present with any hair loss or baldness. Upon lab results, everything is within normal limits. What is the most likely diagnosis?

Increase her daily dose of methimazole. Initiate a prescription of a small dose of thyroxine. Tell her to present ASAP for total and differential blood count. Prescribe antiviral antibiotics. Prescribe saltwater gargles and acetaminophen Tell her to present ASAP for total and differential blood count

A 22-year-old woman is started on methimazole for the treatment of her hyperthyroidism. She has been diagnosed with Graves' disease and has been symptomatic for the last 6 months. She is otherwise healthy and does not take any other medications. You receive a call 2 weeks later from the patient; she tells you that she has had a mild sore throat for the past 2 days and has been feeling like she is coming down with the flu. She states that the symptoms of her hyperthyroidism are a little bit better. What should you do?

Alprazolam Calcium carbonate Fluoxetine Leuprolide Norgestimate plus ethinyl estradiol Fluoxetine - SSRI - She is having PMS and PMDD becuase of the effect of socails - depression --> SSRI

A 22-year-old woman presents due to feeling depressed, withdrawn, and irritable from 3 days prior to her menses until the day after her flow begins. She frequently misses her college classes, and she is concerned about her symptoms' potential impact on her academic performance. She is otherwise in a good state of health, and she has no history of chronic medical or psychiatric disorders. She is not taking any medication. She does not use tobacco, drink alcohol, or use illicit drugs. Menarche was at age 13, and her menses are currently regular each month. She denies cramping, bloating, and other associated physical symptoms. She is sexually active with one partner, and she uses condoms for contraception. Following prospective documentation and confirmation of her symptom pattern, what would be the single best first-line treatment for this condition?

Dashboard knee injury in a car collision Fever and prior Neisseria gonorrhoeae infection Frequent squatting and climbing stairs Sudden landing playing basketball Lateral blow to knee Sudden landing playing basketbal (Lachman and anterior drawer tests are positive - ACL sports )

A 22-year-old woman presents for evaluation of knee pain. The patient appears in moderate distress and is unable to fully bear weight as she moves to the exam table. Her physical exam reveals a swollen and tender left knee. She is tender at the lateral femoral condyle, lateral tibial plateau, and tibiofemoral joint line. Lachman and anterior drawer tests are positive. Posterior drawer, Apley grind, patellar grind, and McMurray tests are negative. There are no sag or apprehension signs. Valgus and varus stress tests are normal. An X-ray shows no fractures. What inciting event is most likely in this patient's history?

Hypercortisolism

A 22-year-old woman presents with an 8-month history of amenorrhea. Further questions elicit additional pertinent positives of backaches, headaches, and acne. Physical examination reveals a female patient with a moon-shaped face, multiple purple striae, and significant central obesity (body mass index of 36). Considering the given history and physical exam discoveries above, what is the main cause of these findings?

Migraine headache

A 22-year-old woman presents with headache. The pain is located on the right side of her head and is described as throbbing. The headache has lasted for approximately 10 hours and has been unrelieved by acetaminophen and ibuprofen. The pain worsens with movement, exposure to light, and loud noises. Neurological examination is within normal limits. What is the most likely diagnosis?

Methylphenidate Haloperidol Sertraline Topiramate Fluoxetine Topiramate (Borderline personalty disorder --> topiramate is used for angry)

A 23-year-old male patient presents with an unspecified personality disorder. His group and individual therapy sessions are going well, but he wants to gain even more improvement with his diagnosis. He feels that his mood, self-image, and personal relationships have improved overall, but he specifically states he wants to have better anger management. When he becomes angry, for whatever reason, he still becomes extremely irritated, hostile, and even aggressive. What pharmaceutical therapy is most beneficial for this patient?

Complete blood count Glucose tolerance test Serum creatinine measurement Urine albumin measurement Urine culture Urine albumin measurement

A 23-year-old man diagnosed with type 1 diabetes 5 years ago presents for a regular examination. Upon physical examination, temperature is 98.1°F, blood pressure is 120/80 mm Hg, and there are no signs of edema. He explains that recently he has been noticing that his urine seems a little darker and foamy, but he has no discomfort while urinating. Blood tests indicate an A1c level of 6.7%. What test should be ordered?

Magnetic resonance imaging (MRI) (Brain: spine, loss of right side)

A 23-year-old man presents with a 2-week history of fever and back pain that radiates from the midline to one side. The patient also has a 1-week history of right lower limb weakness and urinary incontinence. The patient gives history of trauma to the back following a motor accident 1 month ago. Examination by touch and percussion demonstrates localized pain in the spine; you also note neurological losses on the right side of the body that include decreased sensation and weakness. The patient's temperature is 101°F, and his skin is moist. What investigative procedure has the greatest diagnostic accuracy for this condition?

Intravenous naloxone Intravenous nitroprusside Intravenous diazepam Intravenous N-acetylcysteine Intravenous flumazenil Intravenous naloxone

A 23-year-old man with no known significant past medical history is brought in by emergency medical services in an unconscious state. He was reported by friends to be out partying, carrying a prescription bottle. His father has a known history of severe spinal stenosis, for which he takes prescription opioid analgesics. His physical exam reveals slow and shallow respirations, bradycardia, hypotension, cyanosis, and miosis of both pupils. He is comatose, has diminished bowel sounds and distension with dullness to percussion over the suprapubic abdominal area, and has flaccid musculature. What is the preferred intervention for this patient?

Overactivation of renin-angiotensin system Overproduction of B-HCG Placental ischemia Renal artery stenosis Essential hypertension Placental ischemia (classic feature of pre-eclampsia: blood pressure, edema and proteinuria)

A 23-year-old primigravida patient is referred to the obstetrician by a community nurse. The nurse noted two blood pressure readings of 150/90 mm Hg and 154/90 mm Hg taken 15 minutes apart. Physical examination reveals a uterus at roughly 24 weeks gestation and grade 2 pitting edema. Urine analysis is 1+ positive for albumin. Sonogram at 8 weeks gestation showed a single live intra-uterine gestation. The patient has had regular antenatal checkups and has no past history of hypertension or diabetes. The patient's parents and grandparents have hypertension. The patient is unaware of whether her mother or grandmother had similar problems during pregnancy. What is the most likely initiating event for this patient's condition?

Tension headache

A 23-year-old woman presents due to a headache that appears to be bilateral, and she describes it as a "tight" feeling around her head. She started physician assistant school 2 months ago and has been really stressed out. Upon physical exam, no abnormal findings are found. Additionally, all laboratory findings are within normal limits. What is the most likely diagnosis?

Panic disorder

A 23-year-old woman presents due to palpitations, numbness, shortness of breath, and sweating. She reports that these episodes have been occurring once or twice a week for the past several months and that she cannot discern any consistent pattern or trigger. Although the symptoms occur seemingly at random and independent of social situations, she reports that she has begun to limit her social activities for fear of having an episode while she is away from home. Physical exam and laboratory findings are within normal limits. What is the most likely diagnosis?

Fluoxetine (Prozac) 60 mg daily Diazepam (Valium) 2 mg twice daily Carbamazepine (Tegretol) 200 mg twice daily Donepezil (Aricept) 5 mg daily Methylphenidate HCl (Concerta) 18 mg daily Fluoxetine (Prozac) 60 mg daily

A 23-year-old woman presents to her psychiatrist's office for a follow up regarding her 2-year history of bulimia nervosa. Until this time, she has been undergoing cognitive behavioral therapy (CBT) several times per week. She has shown great improvement, but she would like to consider additional measures to control her disorder. What medication would be an appropriate adjunctive treatment for the patient's bulimia nervosa while she continues to undergo CBT?

Systemic lupus erythematosus Anti-double-stranded DNA antibodies Positive Anti-nuclear antibodyPositive Anti-histone antibodyPositive Malar Rash and swollen joints

A 23-year-old woman presents with a rash and swollen joints. She had been healthy previously, and the only medication she takes is acetaminophen. A review of systems includes recent fevers and a 5 lb weight loss in the past month; she is also experiencing photosensitivity and hematuria. She denies oral ulcers, nasal congestion/discharge, ear pain, pleuritic symptoms, chest pain, neural symptoms, bruising, and bleeding. On physical exam, her temperature is 101°F; her blood pressure is 130/85 mm Hg. She has a malar rash as well as diffusely swollen, warm, and tender joints. Her cardiopulmonary exam is normal. She has no costovertebral angle tenderness. Trace bipedal edema is noted. Diagnostic studies reveal: What is the most likely diagnosis?

Antibiotics Steroids Surgery Watchful waiting Immobilization Surgery (compartment syndrome) (there is a difference in pressure)

A 23-year-old woman presents with an extremely painful right arm. She also reports numbness and tingling of her right hand and fingers. She burned her arm on the stove 1 week ago and was prescribed a silver-containing cream. Physical exam is remarkable for swelling and tenderness of the right forearm. She also has decreased sensation in her right hand and weakness of the right thenar muscles. Tissue pressure in the arm is 55 mm Hg (normal <8 mm Hg). What is the treatment for this patient's symptoms?

Schedule renal ultrasound. Order urine culture and arrange follow-up in 24 hours. Obtain intravenous pyelogram. Treat with oral trimethoprim/sulfamethoxazole. Admit for intravenous antibiotics Admit for intravenous antibiotics

A 23-year-old woman presents with dysuria and left flank pain. She is 27 weeks pregnant. Physical examination reveals a temperature of 38°C (100.4°F); there is tenderness with percussion over the left costovertebral angle. Urinalysis shows 15-20 WBC/hpf and 15-20 bacteria/hpf. What is the most appropriate management plan?

RBC membrane

A 23-year-old woman presents with fatigue and the recent onset of a yellowing of her skin. Her physical examination is remarkable for the presence of splenomegaly. Laboratory results are as follows: Where is the primary pathology in this patient's condition?

Myasthenia gravis

A 23-year-old woman presents with increasing fatigability and muscle weakness. On further questioning, she reveals that she has experienced difficulty swallowing over the past month, and there have been episodes of double vision as the day progresses. Several of her professors have expressed concern because she looks sleepy during the day. A CT of the chest reveals an anterior mediastinal mass. What is the most likely diagnosis?

Flat or stilted intonation Rote memorization of words or phrases Inability to participate in conversation No use of two-word phrases 25% of speech unintelligible to strangers No use of two-word phrases

A 24-month-old boy is brought in by his mother. Although he has been fairly healthy, she is concerned about his speech and language development. The mother says the boy's 4-year-old sister was talking in three- to four-word sentences and asking and answering questions with an extensive vocabulary when she was 2 years old. What finding would warrant a referral for delay in speech and language?

Hemolysis, elevated liver tests, and low platelets syndrome (HELLP)

A 24-year-old African American G2P1 presents to her obstetrician at 34 weeks gestation with weight gain, fatigue, diffuse "swelling," and headache. On physical examination, you note periorbital edema and 3+ pitting edema in the lower extremities peripherally. Physical examination also demonstrates tenderness to palpation in the right upper quadrant. The patient has increased reflexes bilaterally. Vital signs are as follows: Temperature is 99.2°F. Pulse is 89 beats/minute. Respirations are 18. Blood pressure is 174/99 mm Hg. Urinalysis reveals no erythrocytes per high power field and no casts, but there is a large amount of protein in the urine. Chart increased AST, ALT, Alk phosphatase What is the most likely diagnosis?

Ceftriaxone 500 mg IM and doxycycline 100 mg BID for 14 days

A 24-year-old female patient weighing 50 kg presents with lower abdominal pain, nausea, and vomiting that has persisted for about 12 hours. She rates the pain at a 4/10 last night when it began, but she currently rates it a 9/10. She states she had her menses 1 week ago and noted an irregular flow and excessive vaginal discharge since it ceased. She is sexually active and only occasionally uses barrier contraception during sexual encounters. She takes a daily oral contraceptive pill. Considering the most likely diagnosis, what treatment regimen is most appropriate?

Begin aggressive physical therapy. Initiate antibiotics. Initiate bisphosphonate therapy. Initiate high-dose corticosteroids. Utilize crutches for any ambulation Utilize crutches for any ambulation. (Avascular necrosis (AVN)) - crescent sign

A 24-year-old man presents due to increasing pain in his right groin and buttock and difficulty walking. The pain has been present and worsening for about 1 month. He further reports that he had quite significantly injured himself also near his right hip in a fall while snowboarding about 3 months ago. He did not seek care immediately and had difficulty ambulating for a week, then saw a chiropractor to "put his lower back back in." The pain and gait problems eventually improved. Now he is concerned because the pain has returned with no new history of trauma. He is otherwise healthy and takes no regular medicines. He denies any chronic health conditions and has an unremarkable family history for musculoskeletal and rheumatologic conditions. On physical exam, the patient was noted to walk into the exam room with a slight limp. Examination of the hip, buttock, and groin region reveals no edema, erythema, or ecchymosis. The exam does not produce one particular point of tenderness with palpation, but pain is elicited with right hip motions, particularly internal rotation. Diagnostic testing revealed a "crescent sign" and confirmed the suspected condition. The patient will be scheduled for definitive treatment within a week. What approach is best to prevent further complications and maintain his current health until definitive treatment occurs?

Immunosuppressant (local or intralesional corticosteroids)

A 24-year-old man presents with a painful ulcer on the left leg and systemic symptoms. The lesion started 1 week ago as a small pustule that developed at the site of a mosquito bite and rapidly developed into a painful ulcer. The patient describes the pain as "stabbing" and debilitating. He develops symmetrical joint pain, muscle pain, fever, and malaise. Past medical history is significant for ulcerative colitis, currently in remission. On examination, there is a deep exudative ulceration with a well-defined violet border and a worn erythematous indurated edge. There are signs of pathergy. What should be recommended in addition to meticulous wound care?

Left epidural hematoma Right epidural hematoma Left subdural hematoma Right subdural hematoma Subarachnoid hemorrhage Left epidural hematoma (right hemiparesis and left fixed dilated pupil)

A 24-year-old man presents with head trauma received in a motor vehicle accident (MVA) 30 minutes ago. Paramedics relate that the patient experienced a loss of consciousness for 1 minute with a complete recovery. At the subsequent evaluation, the physical exam reveals a patient with a Glasgow Coma Scale (GCS) of 12, right hemiparesis, and a left fixed dilated pupil. What is the most probable finding in the brain CT scan?

Thumb spica splint and referral to an orthopedic surgeon

A 24-year-old man presents with radial-sided wrist pain following a fall on his outstretched hand 3 days ago. He did not seek immediate medical attention due to a lack of swelling. Despite self-treatment with ice and analgesics, his wrist still hurts, especially with ulnar deviation and while trying to open a jar or grip the steering wheel of his car. On examination, his tenderness seems to be in the anatomic snuffbox of the wrist. Radiographs are normal in all views. What is the proper treatment for this patient at this time?

Arthroscopy Bone biopsy Nuclear bone scan Plain radiography (X-ray) Ultrasound Plain radiography (X-ray)

A 24-year-old man with no significant PMH presents due to increasing pain in his right groin and buttock and difficulty walking. The pain has been present and worsening for about 1 month. He further reports that he had significantly injured himself (also near his right hip) in a fall while snowboarding about 3 months ago. He did not seek immediate care and had difficulty ambulating for a week, then saw a chiropractor to "put his lower back back in." The pain and gait problems eventually improved. Now he is concerned because the pain has returned with no new history of trauma. He denies any family history for musculoskeletal and rheumatologic conditions. On physical exam, the patient was noted to walk with a slight limp. Examination of the hip, buttock, and groin region reveals no edema, erythema, or ecchymosis. The exam does not produce one particular point of tenderness with palpation, passive range of motion is limited and painful, especially with forced internal rotation. A straight-leg raise against resistance elicits pain. What diagnostic study would confirm this patient's condition?

Polycystic ovarian syndrome

A 24-year-old obese woman arrives at an infertility clinic accompanied by her husband. The couple has been married for 3 years, and they have been unsuccessful at conceiving a child. History and examination of the husband is unremarkable. She attained menarche at age 13 and gives history of irregular cycles for the past 5 years. There is no history of pelvic pain or discomfort. The patient's last menstrual period was 2 months ago. On physical exam, the woman has hirsutism and acne, and her pelvic examination is unremarkable. Investigations reveal elevated serum levels of dehydroepiandrosterone sulfate (DHEAS) and androstenedione. What is the most likely cause of infertility in this patient?

Decreased serum aldosterone Increased serum atrial natriuretic peptide Increased intravascular volume Increased serum ADH concentration Urine osmolality less than serum osmolality Increased serum ADH concentration (patient is dehydation, need to increase ADH)

A 24-year-old patient presents with severe diarrhea for 3 days. They feel dizzy upon standing, the tongue is dry, and the eyes appear glazed. Serum sodium concentration is 130 mEq/L. What finding is most likely?

Intramuscular injection of B12

A 24-year-old woman is evaluated in the endocrinology clinic. She is on 100 mcg of thyroxine per day for treatment of Hashimoto's thyroiditis and receives oral steroids for autoimmune adrenal insufficiency. In addition, she complains of fatigue, numbness, and tingling in her feet. When routine labs were drawn, the following values were noted: Hb: 9.9 g/dL MCV: 102 Iron indices: normal Folate: normal B12: low Parietal cell antibodies: high What would prevent the development of megaloblastic anemia in this patient?

Antiplatelet IgG

A 24-year-old woman notices that she has a bruising tendency. She frequently has numerous small bruises and purple blotches on her skin. She hates having dental work because of the associated bleeding. On physical exam, you note that she has numerous petechiae. Her lab results are shown in the chart. Most likely to be characterized by what?

Ectopic pregnancy Pelvic inflammatory disease Appendicitis Bacterial vaginosis Endometriosis Pelvic inflammatory disease (discharge, nausea and vomiting )

A 24-year-old woman presents with a 12-hour history of lower abdominal pain, nausea, and vomiting. She rates the pain at a 4 last night when it initially began, but she currently puts it at a 9; she states that the pain seems to be worsening with each passing hour. She states she had her menses 1 week prior, noting an irregular flow and excessive vaginal discharge since it ceased. She is single and admits to not being monogamous; she only occasionally uses barrier contraception during sexual encounters. She takes a daily oral contraceptive pill. What is the most likely diagnosis?

History of tricyclic antidepressant use Past history of appendectomy Family history of diabetes Menstrual history Personal history of alcohol intake History of tricyclic antidepressant use

A 24-year-old woman presents with a 3-month history of excreting fluid from her left breast. When her symptoms first started, she had noted clear fluid staining her bra, which recurred infrequently. If she compressed the left breast tissue around her nipple area, she could easily express clear fluid. Occasionally, with difficulty, she was also able to express the same fluid from her right breast. Medical and surgical history is positive for depressive disorder; it has treated with tricyclic antidepressants for the past 4 months; she had an appendectomy 2 years ago. Her menstrual history is regular, with dysmenorrhea present. She drinks alcohol occasionally. Family history is positive for diabetes. What history has the most significance in regard to this patient's condition?

Vitamin A deficiency

A 25-year-old male patient presents with a 1-month history of an inability to see well while walking home from work at night. He reports that he has been getting sore throats more frequently. He is a recent immigrant from China; he lives alone and eats mainly a rice-based diet. On examination, pericorneal and corneal opacities are noted. What is the most likely diagnosis?

Ankylosing spondylitis

A 25-year-old male patient presents with back pain and stiffness. There has been an increasing presence of stiffness and general fatigue with the pain. The pain is much worse first thing in the morning, rated 7/10. Radiation occasionally occurs into the buttocks, and the symptoms improve with activity. Physical examination is remarkable for forward stooping of the thoracic and cervical spine and decreased lateral flexion in the lumbar spine. What is the most likely diagnosis?

Agoraphobia Binge eating disorder Substance use disorder Reactive attachment disorder Schizophrenia Substance use disorder

A 25-year-old man has a long history of criminal behavior, and he has been in and out of prison for assaultive behavior, theft, armed robbery, and sexual assault. He shows no remorse for his behavior and states that he is the real victim. He also shows little emotion regarding his family history, and he prefers to brag about his sexual exploits. The patient denies a history of mood disorders or schizophrenia. What disorder commonly co-occurs with this patient's symptoms?

Schizophrenia Schizoaffective disorder Schizophreniform disorder Brief psychotic disorder Medication-induced psychotic disorder Medication-induced psychotic disorder (There is no history of schizophrenia and there is no time frame)

A 25-year-old man is referred to you for evaluation. He has no history of psychiatric disturbance, and there is no family history of psychiatric illness. During the evaluation, he states that he has seen people following him and he has been having difficulty concentrating. He believes that he is in danger and that the people following him are FBI agents. When asked about the onset of the symptoms, he states that they began about 1 month ago and they have persisted ever since. The patient is not currently taking any psychiatric drugs, but he has been taking antihistamines to treat allergies for about 1 month. What is the mostly likely diagnosis?

H2-receptor antagonists Proton-pump inhibitor Antiemetics Antibiotics H2-receptor antagonists

A 25-year-old man presents for evaluation of intermittent chest pain. He reports a burning sensation in his chest 1-2 days per week after eating. The symptoms are worse with large meals, eating late at night, and excessive alcohol consumption. He denies difficulty swallowing, weight loss, night sweats, chest pain, use of tobacco, or coughing up blood. His vital signs and physical examination are unremarkable. What is the most appropriate treatment for this patient's current symptoms?

Testicular torsion

A 25-year-old man presents to the local emergency department due to severe testicular pain. The pain began abruptly about 2 hours ago and has gotten progressively worse. He is currently rating it as a 9/10. This is only affecting the left testicle. He has never had an episode like this before. He states he has become nauseated during the presentation. Question What is the most likely diagnosis, categorizing this patient as a surgical emergency?

TNF inhibitors NSAIDs Sulfasalazine Etanercept Corticosteroid NSAIDs (back pain, buttock area, and actually lessen with activity)

A 25-year-old man presents with back pain and stiffness. He states he has had longstanding issues with back pain. He denies any trauma to his back. He has noticed associated increasing stiffness and general fatigue. He feels that these issues have gradually worsened over the last several months and are more persistent recently. He notes that the pain is much worse first thing in the morning, rating it a 6-7/10. Radiation occasionally occurs into the buttock areas and the patient feels the symptoms actually lessen with activity. Physical examination shows marked forward stooping of the thoracic and cervical spine with the lower spine showing the presence of a substantial reduction in lateral flexion. What would be the first-line pharmaceutical treatment to consider in this patient?

Monoamine oxidase inhibitor Selective serotonin reuptake inhibitor Tricyclic antidepressant Lithium carbonate Supportive psychotherapy Supportive psychotherapy

A 25-year-old primigravida woman gave birth to a healthy male infant at 40 weeks gestation by normal spontaneous vaginal delivery (NSVD). She breastfeeds on demand and was doing well until day 4 postpartum. At that time, she developed insomnia, fatigue, and feelings of sadness and depression, which have been present for the last 3 weeks. She cries easily and feels guilty that she does not enjoy her baby as much as she had expected. She has not yet resumed any predelivery social activities and is often ready for bed when her spouse returns from work to assume care for the baby. Because she feels so tired, she wishes she had never begun breastfeeding. What is the best initial choice of treatment for this patient?

Chlamydia trachomatis

A 25-year-old sexually active male patient notices burning and pain while urinating and some urethral discharge. The results of a Gram stain and culture on a sample of the discharge rule out the diagnosis of gonorrhea. What is the most common etiology of this patient's symptoms?

Alprazolam Clonazepam Paroxetine Imipramine Clomipramine Paroxetine

A 25-year-old woman has a 2-month history of "episodes" that occur more than 3 times a week. She describes the episodes as a pounding heartbeat, breaking out in a sweat, and difficulty catching her breath. Occasionally, she will feel dizzy and faint with a "tingling" sensation throughout her body. Based on the suspected diagnosis, what is the first-line drug used in management of this disorder?

Plasma fractionated metanephrine

A 25-year-old woman is admitted to the hospital with a 3-week history of headaches accompanied by palpitations and sweating. The episodes occur several times daily and last for 15 minutes. Heart rate is 125 beats/min and blood pressure is 220/100 mm Hg. Physical examination is remarkable for diaphoresis. What test would be most useful in establishing the cause of these symptoms?

Bipolar disorder

A 25-year-old woman is brought in by her sister. The woman has had increasingly frequent incidences of bizarre behavior, and her family is concerned. The most notable episodes occurred within the past week. She seemed sad and distant for a couple of days, then she left abruptly and returned home after being gone for 2 days. During that time, the woman told her sister that she had driven for 3 hours for no particular reason and spent much of her time partying and spending money. The normally intelligent woman was unable to remember where she had gone, and she could not explain why she left in the first place. After running out of money and sleeping in her car for 2 nights, she called her sister, who came and got her. Her sister found her dirty and speaking quickly about nothing in particular. She was brought home, and she now looks quite calm and seems a bit solemn. On examination, the woman's vital signs are within normal limits. Lab work reveals nothing abnormal. The patient did not display any unusual behavior. She was quite pleasant and cooperative. Her score on the mini-mental state exam was 29, but she does not seem to recall much about what happened a few days ago. What is the most likely diagnosis?

Hyperhidrosis Insomnia Heat intolerance Diarrhea Decreased axillary hair Decreased axillary hair

A 25-year-old woman presents because she feels weak and tired all the time. She also reports that she has been amenorrheic for the past 3 months. Her past history is significant for transsphenoidal surgery followed by salvage radiation therapy for a pituitary macroadenoma 5 years earlier. On examination, she is not pale, but she appears fatigued. Her BP is 100/50 mm Hg, and a pregnancy test is negative. What other sign or symptom would you expect from this patient?

Bluish appearance to cervix - she might be pregenat

A 26-year-old woman presents to her gynecologist's office with a 4-month history of amenorrhea. She has had some breast tenderness, but she denies nausea, vomiting, fatigue, and abdominal pain. She was sexually active until about 6 weeks ago when she broke up with her boyfriend, but she states that they used condoms. What appearance of her cervix during an internal examination would support your suspicion?

Metronidazole by mouth

A 25-year-old woman presents for an annual gynecological visit. Menses onset was at age 12; duration of menses is 6 days, every 30 days. She is nulliparous with no history of abnormal pap smears or diagnosed STIs. She does not smoke and has been in a monogamous relationship with 1 partner for the past year. The patient mentions a whitish-gray vaginal discharge that increases after intercourse, accompanied by a distinct musty odor; she denies pain from this discharge. Physical examination and a positive result of a whiff test support the suspected diagnosis. What is the most appropriate therapy?

Bacterial vaginosis

A 25-year-old woman presents for her annual gynecological visit. You review her history and note menses onset at age 12 and duration of menses is typically around 6 days and occurs every 30 days. She is G0P0 and has no history of abnormal pap smears or diagnosed STIs. The patient is a non-smoker, is single, and is in a monogamous relationship with one partner for the past year. A pertinent positive the patient mentions is a whitish-gray vaginal discharge that increases after intercourse and is accompanied by a distinct musty odor; she denies any pain from this discharge. Physical examination and a positive result of a whiff test support your suspected diagnosis. What is the most likely diagnosis?

Progestin-only oral contraception pill

A 25-year-old woman presents to discuss her available contraception options. You review her history and note menses onset at age 12, duration of menses typically around 6 days, occurring every 30 days. She is G0P0, and she has no history of abnormal pap smears or diagnosed STIs. The patient is a non-smoker in a monogamous relationship with one partner for the past year. All other medical history is non-contributory with the exception of the patient having a deep venous thrombosis at age 19 and a pulmonary embolism at age 21. Other than anticoagulation therapy for the appropriate amount of time, no other hematological evaluation was pursued after these events. Given the patient's history, what form of contraception would you select?

Abdominal guarding Adnexa approximately 2.5 cm in size Anteverted uterus Cervical motion tenderness Discomfort with speculum insertion Cervical motion tenderness

A 25-year-old woman presents with pelvic pain for the last 3-4 menstrual cycles, the most recent episode with increasing symptoms for approximately 2 weeks. She has tried over-the-counter pain relievers without relief. She admits deep dyspareunia, and she has been with her current sexual partner for about 6 months. She does not use barrier contraception every time with her current and previous partner, although she is on an oral contraceptive pill. She has no chronic medical conditions. There are no allergies to any medications. She denies urinary issues. She admits to nausea, but no vomiting or diarrhea. She reports a negative home pregnancy test this morning. Her last pelvic exam was over 1 year ago. LMP was 1 week ago. What physical exam findings would best narrow your diagnosis?

Breastfeed, but temporarily stop if nipples crack or bleed

A 25-year-old woman who is 36 weeks pregnant presents for her regular obstetrics care examination. She was diagnosed with hepatitis C a couple years ago. No viral RNA was detected during her pregnancy, which progressed smoothly and without major discomfort. She has no history of intravenous drug use or blood transfusions. She is HIV negative. She plans on breastfeeding, but she is concerned about transmitting the hepatitis C virus to the newborn. How do you advise this patient regarding breastfeeding her infant?

Escherichia coli

A 26-year-old African American woman presents 2 weeks after an uncomplicated vaginal delivery complaining of signs and symptoms of a urinary tract infection (UTI). Which pathogen is the most likely cause of this patient's UTI?

Escherichia coli - UTI most common and the patient is immunocompromised

A 26-year-old G1P0 pregnant woman with type 1 diabetes presents to her obstetrician for her 20-week appointment. Over the past day, she has had to urinate more frequently; she has also developed a burning sensation with urination. Urinalysis reveals the following: What organism is most likely to be grown from this patient?

Apply a long leg spica cast. Perform emergent surgical fasciotomy. Recommend a topical application of ice. Refer the patient for a lower extremity sonogram. Switch to a parental form of analgesia Perform emergent surgical fasciotomy

A 26-year-old man with no significant past medical history presents to the ED after a car crash 2 hours ago. He was seated in the front passenger space and seatbelt-restrained. He states that his right lower extremity struck the dashboard during the impact. The pain in the right lower extremity was initially tolerable, but it has rapidly intensified over the past 30 minutes. The orally-administered morphine is not helping to reduce his pain anymore; he notes a burning dysesthesia in the area. Physical examination reveals severe tenderness upon passive ranges of motion, swelling, firmness, tenderness to palpation, and squeezing of the affected area. The skin is warm, and distal pulses are intact bilaterally. What is the most appropriate intervention for this patient at this time?

Azithromycin Azithromycin + ceftriaxone Doxycycline + ceftriaxone Fluconazole Levofloxacin + doxycycline Azithromycin

A 26-year-old patient at 29 weeks gestation presents with a 1-week history of vaginal discharge and dysuria. On genital exam, she has mucopurulent cervicitis. Cervical swab results are positive for Chlamydia trachomatis infection and negative for Neisseria gonorrhoeae. What is the most appropriate course of treatment?

Increase levothyroxine dose

A 26-year-old woman has been trying to conceive over the last few months. She is married and has one child. She has a history of one miscarriage. Her past medical history is significant for hypothyroidism, which is treated with levothyroxine (Synthroid) 125 mcg PO per day. Her last thyroid-stimulating hormone (TSH) level performed 3 months ago was normal. Today, she presents with a 6-week history of amenorrhea. A urine pregnancy test is positive. Her TSH level today is elevated. She is estimated 6 weeks pregnant. What is the most likely next step in management?

Insulin

A 26-year-old woman is in the 24th week of her first pregnancy. She is in fairly good shape, and the pregnancy is progressing well, but fasting blood glucose in the office shows 146 mg/dL. It is repeated the next day, and the value is 142 mg/dL. An oral glucose tolerance test comes back as elevated. She is diagnosed as having gestational diabetes and is advised to meet with her obstetrician. In addition to recommending diet, what is the initial recommended regimen for this patient if blood glucose is not controlled by diet?

Alcohol consumption Eclampsia Hyperthyroidism Multiparity Nutritional deficiency Nutritional deficiency (hydatidiform moles)

A 26-year-old woman presents with 20 weeks of amenorrhea. This is the first time she has been to the doctor in 5 years, and she believes she is pregnant. You perform a complete physical examination and cannot auscultate the fetal heart rate. The ultrasound reports a "snowstorm" pattern with placental and fetal remnants missing. The patient tells you she has history of miscarriage "that had something to do with high blood pressure." The doctor explains that she is not pregnant and has a disease related to the proliferation of trophoblasts. What is a risk factor in the development of this disease?

CA125

A 26-year-old woman presents with 8 weeks of gastric reflux that does not improve with medication along with bloating, constipation, vaginal bleeding, and weight loss. Past medical history is significant for father with hypertension and coronary artery disease, mother with breast cancer, aunt with hypothyroidism, and sister with breast cancer. Complete physical examination is significant for an adnexal mass. CBC is significant for hemoglobin 11.5 g/dL and hematocrit of 38%. Urinalysis is negative. What would be the most appropriate tumor marker to order in this patient?

Increasing caffeine intake Limiting aerobic exercises Applying warm compresses locally Drinking a glass of wine at night Limiting sexual activity Applying warm compresses locally

A 26-year-old woman presents with dysmenorrhea that has become more bothersome over the last 3 months. She has difficulty taking ibuprofen (Motrin) due to a stomach ulcer. What is the most appropriate non-pharmacological measure that would relieve her symptoms?

Chronic peripheral vascular disease, leading to poor arterial blood flow Demyelination of the neuron sheath, leading to pain and paresthesias Immobility of the affected limb, leading to venous stasis and thrombosis Increased dopamine release, leading to activation of the reward centers Increased pressure within the tissue space, leading to venous obstruction Increased pressure within the tissue space, leading to venous obstruction

A 27-year-old Caucasian man returns to the emergency department with unbearable left lower leg pain approximately 6 hours after initial discharge. While playing lacrosse, the patient sustained a closed mid-shaft tibial fracture. After casting and an anti-inflammatory, his pain was noted to be mild (2/10) at time of discharge. He reports his pain is increasing dramatically (it is now rated 9/10) and is unresponsive to his prescribed narcotic, acetaminophen, icing, and elevating his leg. He also describes a feeling of tingling and numbness throughout the lower left extremity. The patient arrived on crutches and appears in obvious pain. He is afebrile; he has a pulse of 105. The cast is intact and the remainder of the left leg and foot is examined. The patient is tender to palpation of the left foot and ankle, with swollen firm tissue. The skin appears shiny. Pulses are normal, but capillary refill and sensation are decreased on the left foot compared to the right. With passive movement of the patient's left toes, he cries out in pain. There is no ecchymosis or visible skin lesions on the left foot or ankle. What explains the physiologic basis for this patient's severe pain?

Deep vein thrombosis Endometritis Infected retained placenta Mastitis Septic pelvic thrombophlebitis Endometritis (Prolonged rupture of membranes, cesarean delivery, prolonged labor, and multiple cervical examinations are all risk factors for postpartum endometritis)

A 27-year-old G1P1 woman has recently given birth by cesarean section to a 36-week male newborn. She did well throughout her pregnancy until 34 weeks gestation; she presented with fever, abdominal pain, and wetness. She was diagnosed with preterm premature rupture of membranes (PPROM) and chorioamnionitis. She was treated with steroids and antibiotics, stabilized, and then delivered by C-section with no complications. On postpartum day 7, she presents with sore breasts from breastfeeding and a sore abdomen. She admits to an odorous vaginal discharge, but she denies any associated vaginal bleeding. On physical examination, she has moderate lochia alba, and her temperature is 101.2°F. What is the most likely diagnosis?

Start omeprazole

A 27-year-old male patient presents with a burning sensation in the chest, dry cough, hoarseness, and a sensation of a lump in the throat. He experiences belching followed by a sour liquid taste in the mouth. Episodes appear to be precipitated by stressful events (exams, job interviews). The rest of the history, family history, and physical are non-contributory. EKG, chest x-ray, and CBC are unremarkable. What is the most appropriate next step in management?

Cannabinoids Nicotine Amphetamine Cocaine Opiates Cannabinoids

A 27-year-old male patient underwent pre-employment urine drug testing after being hired for a full-time hospital position. The patient admits to using recreational drugs in the past, but he claims he has not used anything for at least 2 weeks. The urine test returns positive. What substance is this patient most likely using?

Take X-ray of the tibia and fibula. Perform MRI of the right leg. Remove cast and check compartment pressure. Perform ultrasound of the lower extremity. Monitor and continue giving narcotics Remove cast and check compartment pressure

A 27-year-old man is admitted to the hospital following a motor vehicle accident. He sustained lacerations to his arms bilaterally and has fractures of the right tibia and fibula. A cast is placed and the patient is scheduled for surgery the following day. A few hours after the cast is placed, he develops severe pain; the pain is unresponsive to several doses of intravenous morphine. His pain increases when he extends his right leg. Peripheral pulses are weak but present. Based on the history and physical, what should be done next?

Anteroposterior (AP) and lateral X-ray Ultrasonography Computed tomography (CT) scan Magnetic resonance imaging (MRI) Positron-emission tomography (PET) scan Magnetic resonance imaging (MRI) (meniscal tear and the mcmurray test is postive ) - see soft tissues

A 27-year-old man presents with knee pain that started while he was playing basketball the previous afternoon. The patient states that he twisted his left knee with his left foot still planted on the floor as he attempted to pass the ball to a teammate. He describes a popping sensation at the time of the injury, but he could still bear weight on his leg. He does not recall any immediate swelling. The morning of presentation, the patient noted swelling of the knee and pain that was significantly worse than it was the day before. He has taken 600 mg ibuprofen for the pain but has experienced only mild relief. Upon examination of the left knee, there is a limitation in range of motion, the presence of medial joint line tenderness, and an effusion. McMurray's test is positive. What is the optimal imaging modality or procedure for confirming the suspected diagnosis?

History of PID Diaphragm use Condom use Oral contraceptive pills Vaginal spermicides History of PID

A 27-year-old patient presents with cramping abdominal pain and vaginal bleeding. Further history reveals amenorrhea for the past 2 menstrual cycles. On examination, there is left lower abdominal tenderness and an adnexal mass. Lab values reveal an elevated beta HCG level. What associated risk factor may be revealed by further history?

Postpartum psychosis

A 27-year-old woman is brought to the ER by her husband because of bizarre behavior 4 days after the uncomplicated delivery of a full-term healthy boy. Her personal and family history of mental illness is negative. The second day after delivery, she accused her husband of poisoning her food. She has problems falling asleep, generally sleeping only 2-3 hours nightly and complaining of unpleasant smells waking her up. She started arranging toys and the newborn's things in a specific symmetrical order and became aggressive when someone moved them. She would stop the ongoing activity from time to time with a blank expression and was found several times staring at the wall and silently counting. Even simple tasks require the help of others, and she often forgets what she started doing. This makes her irritable and sometimes tearful. She feels guilty about being a terrible mother and states that she did not deserve to have a baby. The morning when her husband asked for your advice, she told him that she hears voices telling her to take her son and jump out the window. What is the most likely diagnosis?

Penicillin Acyclovir Fluconazole Metronidazole Doxycycline Acyclovir (herpes)

A 27-year-old woman presents due to labial pain and burning and reportedly noticed a cluster of blisters in the area 2 days ago. She had myalgia, malaise, and a temperature of 100.5°F a few days prior to appearance of the blisters, which kept her home from work. She is monogamous and her partner denies similar symptoms. On physical examination, inguinal lymphadenopathy is noted in addition to the presence of multiple vesicles on the labia and perineum; some are ruptured and some are crusted. There is no vaginal discharge, and the rest of the pelvic exam is unremarkable. What is the proper treatment for this condition?

Cord prolapse Placental abruption Placenta previa Uterine rupture Vasa previa Cord prolapse - the fetal hear monitoring decrease - clear fluid

A 27-year-old woman presents in active labor. She is G3P2 and at 39 weeks of gestation. She has been receiving prenatal care since 6 weeks gestation, and her pregnancy has been uncomplicated. Both of her prior births were normal spontaneous vaginal deliveries. Her cervix is 6 cm, 90% effaced, mid-position, and soft. The fetus is not engaged and is thought to be vertex. Initial fetal monitoring shows a heart rate in the 140s with good accelerations, and it is reassuring. Contractions are 4 minutes apart, and she is comfortable. 20 minutes later, she experiences a large gush of clear fluid, and severe variable decelerations appear on the fetal heart rate monitor. What is the most likely diagnosis?

Schedule frequent visits and involve her in the decision-making process

A 27-year-old woman presents with excruciating back pain. She states that the pain started earlier that morning and caused her to miss work. She has had similar pain like this before, but no doctor has ever found anything wrong. She is currently on fluoxetine for depression and periodic ibuprofen for pain. On physical exam, tissue texture changes are noted at L1 to L3, with no additional findings. The woman seems to be in an immense amount of pain, however, and she continues to insist that something is horribly wrong with her. Diagnostic results are negative, and a pain medication is prescribed. She returns later in the week with the same issue but no new findings. History shows episodes similar to this involving joint pain, abdominal pain, headaches, bloating, diarrhea, a "lump in the throat" feeling, and menstrual cramps. What is the best way to manage this patient?

Radiation therapy Chemotherapy Aspiration Surgical excision Observation Surgical excision (ganlgion cyst)

A 28-year-old female patient presents with a "lump" on the right lateral wrist, first noticed 6 months ago. It was initially 1 inch in diameter and not painful. Over 2 months, it has grown to 3.5 inches and is extremely painful when using the right wrist. She has taken ibuprofen 200 mg orally every 8 hours as needed with only mild relief. Physical examination reveals somewhat limited range of motion of the right wrist due to the pain and size of the mass. It has a translucent appearance when shining a penlight through it. It is firm and smooth to palpation. What treatment regimen is most appropriate based on the most likely diagnosis?

Right inversion ankle sprain Right eversion ankle sprain Right ankle strain Right ankle fracture Right ankle hematoma Right ankle fracture (cant bear weight)

A 28-year-old female patient presents with severe right ankle pain after a fall on ice. The patient cannot move the ankle. She states that it is exquisitely tender and the pain worsens with movement or when attempting to bear weight. She states that she was immediately unable to take any steps due to how substantially painful the right leg is. During the physical examination, the patient still cannot bear weight on the injured leg. The right ankle is painful to even light palpation throughout the malleolar zone. The skin is a healthy color, and there are no signs of lacerations or tenting. Pulses are palpable. The entire area of the right ankle is swollen and appears dislocated. The left leg, foot, and ankle are unremarkable. What is the most likely diagnosis?

Nitroglycerin ointment

A 28-year-old male patient presents with rectal bleeding and three instances of a little bright-red blood with bowel movements. He reports severe rectal pain, especially with passing hard stools. Over-the-counter hemorrhoid creams have not provided relief. He admits episodic constipation for 2 years; he has eaten a high-fiber diet and taken over-the-counter stool softeners for 3 months. He denies dark tarry stools, easy bruising, and prior episodes of rectal bleeding. He has not noticed blood when urinating or brushing teeth. He denies nausea, vomiting, diarrhea, fevers, and weight loss. No known medical conditions. Family history is negative for gastrointestinal disorders. Social history reveals he is in a heterosexual relationship, denying anal intercourse. On physical exam, the abdomen is unremarkable. The anus has no visible protrusions or rash, but there is a very small, erythematous, tender "paper cut" area. Digital rectal exam (DRE) is painful. No masses in the rectal vault. In addition to treating constipation, what is the most appropriate prescription treatment for this patient's condition?

Vitamin D deficiency

A 3-year-old boy presents with his mother; she reports that he is unable to walk without support. He is an only child who rarely leaves his tenth-floor apartment home. On examination, he has a rachitic rosary and genu valgum. What is the most likely diagnosis?

IV opioids Oral NSAIDs PCA opioids Topical lidocaine Topical NSAIDs Oral NSAIDs (after rice)

A 28-year-old man presents due to a left ankle sprain. The patient describes walking in the woods in flip-flops yesterday, inadvertently stepping into a hidden hole. The patient immediately noticed extreme pain with weight bearing on the left ankle, making it extremely difficult for him to walk out of the woods. Ambulation is extremely difficult. He describes the pain as an 8-9/10. The patient admits to previous ankle sprains, but this one has been much more severe and debilitating. He has noted significant swelling and exquisite tenderness to touch. He has treated his sprain with elevation, ice for 20 minutes at a time, and acetaminophen 1000 mg every 6 hours. Physical examination reveals a moderate degree of ecchymosis of the left ankle with substantial mechanical instability and moderate restriction of range of motion. In addition to RICE guidelines and immobilization of the ankle, what pharmaceutical intervention should be initiated to further help reduce swelling and pain?

Bipolar disorder type I Delusional disorder Schizoaffective disorder Schizophreniform disorder Schizophrenia Schizoaffective disorder - AFFECTED by schizophrenia and depression The requirement for diagnosis is that the psychotic symptoms have to persist more than 2 weeks after the mood disturbances resolve or in the presence of a mood disorder.

A 28-year-old man presents for a checkup. Medical records indicate a 3-year history of hospitalizations for mania and depression. The last admission occurred 7 months ago and was due to a suicide attempt after a major depression. The patient has been taking lithium since the last hospitalization; there have been no further mood disturbances, but his mother relates that the patient remains convinced that the "city officers" poisoned the water system over the last 2 months. He refuses to drink tap water. His mother heard him talking alone in his room, and she found several bottles of water under his bed. She reports he has not been very social with his friends over the last 6 months and is showing signs of depression. On examination, the patient appears disheveled; he experiences auditory hallucinations and persecutory delusions throughout the interview. His speech is disorganized. What is the most likely diagnosis?

Toxoplasma gondii

A 28-year-old man presents with severe headaches. He states that they began a few weeks ago and that he has been taking over-the-counter medications with some relief. He reports that he has AIDS. You ask what the patient was doing prior to experiencing these headaches. He says that he was taking care of the neighbor's cat while they were on vacation. Concerned, you order a CT scan; it demonstrates multiple ring-enhanced lesions. What organism do you suspect?

Epinephrine Normal saline Blood products Somatostatin Vasopressin Blood products (sepsis)

A 28-year-old patient with no significant past medical history is rushed to the local trauma center after a stab wound to the chest. Paramedics report significant blood loss. The patient has lost consciousness. The extremities are cool and moist to touch. Physical exam is remarkable for tachycardia, tachypnea, depressed systolic pressure, and immeasurable diastolic blood pressure. What is the preferred initial pharmacologic agent of choice for this patient?

Endometritis Septic shock Thrombophlebitis Appendicitis Drug fever Thrombophlebitis - prolonged fever swings despite broad-spectrum antibiotics, and she has a normal physical and pelvic examination.

A 28-year-old primipara woman presents with wide fever swings 6 days after the delivery of a healthy male infant. Her pregnancy was uneventful, but because of the prolonged membrane rupture and prolonged labor, an emergency cesarean section was performed. On the second postpartum day, she started having fever and reporting uterine tenderness. She was treated with IV gentamycin and clindamycin. Today, her physical examination is normal; there is no uterine tenderness, and her WBC count is 11500, but she continues to spike fevers up to 39°C. What is the most likely diagnosis at this point?

Cardiac stress test Psychiatric evaluation Thyroid function studies CBC with iron studies Urinalysis Thyroid function studies

A 28-year-old woman at 24 weeks gestation presents with insomnia and fatigue. Her symptoms have worsened as the pregnancy has progressed. She has noticed dyspnea on exertion that is associated with palpitations when grocery shopping or going to the laundromat. Vital signs are blood pressure of 128/88 mm Hg, pulse is 102/minute, respirations are 16/minute, and temperature is afebrile. Weight is 138 lb, which is stable from her last visit 4 weeks ago. On physical exam, the patient is a gravid woman in no acute distress. Clinical findings include mild periorbital edema without ankle edema and brisk ankle reflexes. Examination is otherwise unremarkable. What will confirm the most likely diagnosis?

Prolactin level

A 28-year-old woman presents because she has not gotten her period in 2 months. She is worried that she may be pregnant, but she is concerned because she and her boyfriend broke up about 2 months ago. She does not want to be a single mother. The patient mentions a discharge from both breasts. She denies any significant weight loss/gain or heat/cold intolerance. You order a urinary HCG, which is negative. What diagnostic study would you order next?

RET mutation test in the patient RET mutation test in children Plasma calcitonin levels in children Prophylactic total thyroidectomy Carcinoembryonic antigen test in children RET mutation test in the patient

A 28-year-old woman presents because she is concerned about her children, ages 2, 4, 6. She is asymptomatic and there is nothing abnormal in her medical records, but her father was recently diagnosed with medullary thyroid cancer due to the presence of MEN 2A. She is asking if the condition is hereditary; if so, wants to know what can be done to prevent her children from developing the malignancy. You explain the autosomal dominant trait of the disease. What additional advice should you provide?

Vibrio parahaemolyticus

A 28-year-old woman presents to the emergency department due to a 3-hour history of nausea, abdominal cramping, vomiting, and watery diarrhea. She recounts her lunch approximately 8 hours ago, which consisted of a shrimp salad and a diet soda. She denies any neurological or other symptoms. She also denies any similar problems prior to this episode and was feeling well just before this incident. She denies fever or recent travel. What is the most likely organism responsible for her symptoms?

Acute appendicitis Acute salpingitis Tubo-ovarian abscess Ruptured ectopic pregnancy Ruptured ovarian cyst Ruptured ectopic pregnancy

A 28-year-old woman presents with abdominal pain and vaginal bleeding. The pain began last night as a dull ache in the right lower quadrant, but this morning it became much more severe. She reports dizziness and nausea. Her last menstrual period was 6 weeks ago. On examination, the patient is afebrile; pulse 100/min, BP 86/60 mm Hg, RR 20/min. Physical exam reveals moderate to severe tenderness in the right lower quadrant with rebound tenderness and guarding. Pelvic exam reveals a small amount of blood at the cervical os with cervical motion tenderness. A variety of laboratory tests have been ordered. What is the most likely diagnosis?

Ulcerative colitis

A 28-year-old woman presents with bloody diarrhea for 1 week. After further questioning, she reveals that she had similar attacks in the past that subsided on their own. Her stool specimen is negative for ova and parasites. Stool culture did not grow any pathogens. A sigmoidoscopy is performed. There is friable erythematous mucosa extending from the rectum to the mid-descending colon, with broad-based ulcers in the descending colon. A biopsy taken from the rectum shows diffuse mononuclear inflammatory infiltrates in the lamina propria with crypt abscesses, but no granulomas are seen. What is the most likely diagnosis?

Acute tubular necrosis (muddy brown cast)

A 28-year-old woman receives a lung transplant due to cystic fibrosis. 2 months later, she is readmitted with left-sided paralysis. CT scan is consistent with basal ganglia stroke. After 2 weeks in the hospital, she develops cardiac failure and is intubated. A posterior mitral valve leaflet vegetation with severe regurgitation is identified by transesophageal echocardiography (TEE). She is referred for mechanical mitral valve replacement. Colonies of Aspergillus fumigatus are detected in the excised mitral valve leaflets. Intravenous antifungal therapy with amphotericin B is started. A week later, she presents with fever, shortness of breath, and edema. Laboratory results include elevated creatinine and increased fractional excretion of muddy brown casts on urinalysis. What is the most likely cause of her impaired kidney function?

Testicular malignancy Scrotal hydrocele Epididymitis Orchitis Varicocele Testicular malignancy

A 29-year-old Caucasian man presents for a routine physical examination. He has a history of right-sided cryptorchidism corrected by orchiopexy at age 6 months. He has no problems at present. His physical exam reveals bilateral gynecomastia and a painless, firm right testicular mass approximately 1.5 cm in diameter. The right testicular mass does not transilluminate, nor does it disappear when the patient lies supine. There is no femoral or inguinal lymphadenopathy and no palpable hernia. What is the most likely diagnosis?

Combined oral contraceptive pill Progestin-only pill Non-hormonal copper intrauterine device Levonorgestrel-releasing intrauterine system Contraceptive patch Levonorgestrel-releasing intrauterine system

A 29-year-old female patient is taking D-penicillamine for Wilson's disease. Today, her laboratory findings (including liver function tests) are within reference ranges; there are no neurological signs. Her work requires frequent travel (3-5 days per week), including international overnight travel all over the world. She wants contraception that is both effective and easy to use. What is the best form of contraception for this patient?

Embolization of dilated veins Surgical mesh repair Low-dose testosterone Repeat semen analysis Surgical excision of dilated veins Repeat semen analysis

A 29-year-old man and his wife have been trying to have a child for the past 3 years. The wife was thoroughly evaluated for infertility; the workup revealed no abnormalities. Examination of the man's inguinal region reveals an ill-defined tortuous swelling that increases with standing and coughing. Ultrasonography is suggestive of varicocele. His initial semen analysis reveals mild oligospermia. What is the next step in management?

Diagnostic laparoscopy Ultrasound Venography CT MRI Ultrasound

A 29-year-old man is seen in the clinic for groin pain. He works as a cashier at a local supermarket. The pain increases through the day. When asked to point out its location, he localizes it to his left groin fold. Examination of his inguinoscrotal region reveals an ill-defined tortuous swelling that increases with standing and a palpable thrill on coughing. What is the next step in diagnosis?

Explain risks and benefits (deeming him competent) and leave the decision to him. Contact those close to the patient to influence his consent to hospitalization. Proceed with the procedure for involuntary hospitalization. Call the police to have patient arrested. Call a responsible adult who can ensure the safety of patient and his girlfriend. Proceed with the procedure for involuntary hospitalization

A 29-year-old man presents with bouts of severe depression accompanied by suicidal thoughts. Although he is well-educated, he has been unable to get a job that fits his training because of his illegal status in this country. He has been doing odd jobs, and he has barely been making a living. His relationship with his girlfriend has become stormy over the last few months, and he confesses that he wants to kill his girlfriend. You believe his intent to kill his girlfriend to be serious and suspect that the underlying cause of his fury and homicidality, is untreated depression. For his own safety, you recommend he admit himself to a psychiatric hospital for further evaluation and treatment. He vehemently refuses. Under these circumstances, considering your professional obligations, what should you do?

Idiopathic central diabetes insipidus Familial central diabetes insipidus Acquired central diabetes insipidus Hereditary nephrogenic diabetes insipidus Acquired nephrogenic diabetes insipidus Acquired central diabetes insipidus

A 30-year-old male patient presents with excessive thirst for a few days. He consumes 3-4 liters of water daily but is still thirsty. He has been passing urine very frequently. He gives a history of a road traffic collision 1 month ago where he hit his head against the dashboard. Urine specific gravity 1.002 and osmolality 180 mOsm/kg. There is an increase in urine osmolality with exogenous antidiuretic hormone (ADH) administration. What is the most likely diagnosis?

Cervical cancer Chorioamnionitis Placenta previa Placental abruption Uterine rupture Placental abruption (Placental abruption is defined as a complete or partial separation of the placenta prior to delivery)

A 29-year-old woman, G4P2011, LMP 9 months prior, presents with sudden onset of severe lower abdominal pain. The pain is sharp and tearing and was not preceded by the contractions she recalls from previous deliveries. She also endorses vaginal spotting prior to presentation. There is no history of prior medical problems or surgery, and she is on no medications. All previous deliveries were vaginal. She has smoked 1 pack of cigarettes a day over the past 10 years. She denies alcohol use but does admit to a remote history of heroin abuse by insufflation. On physical examination: Temp 99.4°F; BP 110/70 mm Hg; Pulse 85/min; RR 20/min. Pertinent findings on the PE were relegated to the pelvic exam; fundal height measures 39 cm, and there is profuse bleeding from the vagina. Fetal monitor shows contractions every minute with elevated baseline uterine tone. Fetal tachycardia is evident at 180 beats/minute, and late decelerations are also present. What is the most likely diagnosis?

Intravenous vitamin K

A 3-day-old male newborn starts to have mild epistaxis after vaginal delivery at home. The mother is a 38-year-old G2P2 who had diet-controlled gestational diabetes. She took a prenatal vitamin and iron supplement. Active labor lasted 3 hours. The midwife who examined the newborn after the delivery declared the newborn healthy. Since the delivery, the mother has been exclusively breastfeeding. The baby is eating every 1-2 hours and has had several wet diapers and 3 stools. What would be the most definitive treatment for this newborn's epistaxis?

Staphylococcus aureus Streptococcus species Haemophilus influenzae type b Salmonella species Neisseria gonorrhoeae Salmonella species

A 3-year-old black female patient with sickle cell disease presents with a limp; her mother first noticed it yesterday morning. The girl has not had fever, nausea, or vomiting, but she is in pain. Her sickle cell disease has been relatively mild; she is fully immunized and takes prophylactic oral penicillin 125 mg 2 times a day. On exam, she is afebrile. There is pain in the left hip on internal and external rotation, but there is no definite point tenderness. Exam of the left knee and right hip are negative. The provider is concerned about septic arthritis and plans further workup. What organism is the provider most concerned about?

Multicystic dysplastic kidney disease Splenomegaly Lymphoma Wilms tumor Renal cell carcinom Wilms tumor (unilaterial child tumor)

A 3-year-old girl is brought to the clinic by her mother, who tells you that the child has not been eating well over the past month and has developed swelling in the abdomen. On exam, the child has a smooth abdominal mass that is the size of a baseball on the left side. Vital signs reveal a blood pressure of 134/82 mm Hg, temperature of 99.8°F, and respirations of 16 breaths per minute. Urinalysis shows only 1+ red blood cells, and CBC and CMP are within normal limits. What is the most likely diagnosis?

Reassure parents and discharge patient.

A 3-year-old girl presents with a 5-day history of fever, coughing, sneezing, and watery nasal discharge. According to her mother, she passed out and began shaking yesterday. 1 hour before that, her temperature peaked at 40.1°C (104.2°F). The episode lasted for 1.5 minutes, and she remained drowsy for about 10 minutes afterward. She suffered a similar shaking episode during a bout of otitis media 1 year ago. The child reached all developmental milestones at appropriate ages, received all immunizations, and has no other relevant past medical or family history. Her only medication is acetaminophen. Vital signs: BP 110 / 70 mm Hg, HR 86 bpm, RR 20 rpm, and temperature 37.3°C (99.1°F). On physical examination, there is some wheezing in both lung fields. The remainder of the examination, including a full neurologic exam, is normal. What is the most appropriate next step?

Wilms tumor Neuroblastoma Renal medullary carcinoma Renal cell carcinoma Mesoblastic nephroma Wilms tumor - Wilms tumor accounts for most renal tumors in children under 15

A 3-year-old girl presents with progressive abdominal enlargement associated with abdominal pain. The mother denies any past medical history. Physical examination shows a palpable mass over the right upper quadrant extending to the right flank. Patient looks pale and the BP is slightly elevated. Urinalysis shows microscopic hematuria. What is the most likely diagnosis?

Intravenous drug abuse

A 30-year-old Caucasian man presents with a 3-day history of fever with chills and severe weakness. There are no other complaints. The patient has had multiple sex partners in the past. He also gives a history of travel to South America and consumption of street food while working there 1 month before presentation. He admits to intravenous drug abuse and cocaine abuse (snorting) in his early 20s. He often ventures out into the woods and has been bitten by several insects in the recent past. Abdominal exam reveals mild hepatomegaly. You send for routine lab investigations, including CBC, comprehensive panel, and serology of HIV and Hepatitis B and C. Liver enzymes are elevated and anti-HCV comes back positive. Anti-HIV and HBV are negative. How did this patient most likely acquire the hepatitis C infection?

Niacin deficiency

A 30-year-old male patient presents with a 1-week history of a painful tongue. He has been having diarrhea and forgetting things more easily. He recently emigrated from India and lives alone. He eats mainly a corn-based diet. On examination, the tip and margins of the tongue are a bright scarlet color. He has a symmetrical erythematous rash on the forearms. What is the most likely diagnosis?

Typical onset occurs in the elderly with underlying vascular disease. Associated personality disorders and depression are common. The cornerstone of treatment is benzodiazepines. Highly-educated married Caucasian men are the most likely to be affected. Serological antibody testing is the most specific diagnostic test available Associated personality disorders and depression are common (Lab studies, urine testing, plain radiographs, CT scans, and MRIs are all unremarkable and show no explanation for his symptoms --> somatic sydrome: mental health condition)

A 30-year-old male presents for routine follow-up. He has a history of intermittent headaches, low back pain, knee pain, and dysuria over the last year. Additionally, he notes nausea, diarrhea, poor libido, and extremity numbness. He states he feels worthless. He admits to daily alcohol use in an attempt to feel better. Lab studies, urine testing, plain radiographs, CT scans, and MRIs are all unremarkable and show no explanation for his symptoms, but he is consumed by worry about his illness. What is correct regarding this patient's condition?

Dialyze. Discontinue ibuprofen. Prescribe antibiotics. Prescribe cyclophosphamide (Cytoxan). Prescribe steroids Discontinue ibuprofen

A 30-year-old man presents to the emergency department with generalized malaise. Temperature 98.7ºF, blood pressure 135/85 mm Hg without orthostatic change, examination otherwise unremarkable. Post-void residual and renal ultrasound are unremarkable. Urine output is estimated at 1.5 L/day. What is the initial treatment for the suspected diagnosis?

Initiate twice weekly vaginal estrogen. Prescribe antifungal (eg, fluconazole). Prescribe empiric antibiotics. Prescribe oral or intravaginal metronidazole. Recommend douching with vinegar solution Prescribe empiric antibiotics

A 30-year-old patient presents with a 2-week history of vaginal discharge. There is no vaginal burning or itching, urinary frequency, hesitancy, and dysuria. The discharge is light yellow. The patient hesitates to provide much detail, so a physical exam is performed. The patient is in no distress, is afebrile, has unremarkable vitals, and the exam is normal except for the pelvis. On speculum examination, the cervix is inflamed, bright red, and slightly friable with yellow discharge at the os. Vaginal mucosa is pink, moist, and without inflammation. Uterus and ovaries palpate to normal size and are non-tender on bimanual exam. Considering the most likely diagnosis, what is the most appropriate intervention for this patient?

Delayed menarche Diabetes and virginity Menopause Recent antibiotic use Unprotected vaginal intercourse Unprotected vaginal intercourse

A 30-year-old patient presents with a 2-week history of vaginal discharge. There is no vaginal burning or itching, urinary frequency, hesitancy, and dysuria. The discharge is light yellow. The patient hesitates to provide much detail, so a physical exam is performed. The patient is in no distress, is afebrile, has unremarkable vitals, and the exam is normal except for the pelvis. On speculum examination, the cervix is inflamed, bright red, and slightly friable with yellow discharge at the os. Vaginal mucosa is pink, moist, and without inflammation. Uterus and ovaries palpate to normal size and are non-tender on bimanual exam. If the patient provided full details of her history, what would be most consistent with this her presentation and likely condition?

Suprapubic distension Hypertension Adnexal mass Cervical motion tenderness Diminished vibratory sensation Suprapubic distension

A 30-year-old woman presents to her primary care office with increasing polyuria of pale colorless urine, urinary frequency, enuresis, and nocturia. She has a past medical history of Sheehan's syndrome and a craniopharyngioma, for which she has followed up with her neurologist. She states that as a busy office worker, she finds it difficult to drink water throughout the day and has developed an "unquenchable thirst." She also finds that her sleep is disturbed and has mild daytime fatigue and somnolence as a result. She denies a family and personal history of diabetes mellitus, thyroid dysfunction, illicit drug use, smoking, and alcohol use. She denies fever, chills, polyphagia, weight changes, vaginal discharge, hematuria, or abnormal urinary odor. What physical exam finding would be most likely in this patient?

Norepinephrine

A 30-year-old woman presents with episodic headache, palpitations, and sweating. She has had multiple episodes of approximately 20 minutes over the last few weeks. Her primary care physician noted hypertension on her last several visits, and her blood pressure today is 210/98 mm Hg. She has no past medical history. The production of what hormone is associated with her condition?

Atopic dermatitis Candidiasis Lichen planus Psoriasis Candidiasis - HIV and Diabetes

A 30-year-old woman presents with lower abdominal pain; she is thought to have pelvic inflammatory disease (PID). She reports being a sex worker and recreational drug use. Labs on admission reveal blood glucose of 260 mg/dL, a positive HIV screen, and a non-reactive RPR. Aside from the abdominal/pelvic pain, the admitting physical is notable for moderate obesity, absence of pronounced lymphadenopathy, and an erythematous macerated rash in the intertriginous distribution. The patient reports her rash is "really itchy and wet all the time," and it began within the last 3 months; it is now at its worst. She reports intense itching of the vulva over the last few weeks. The intertriginous rash is most likely a manifestation of what condition?

Fibrocystic condition (changing in size)

A 30-year-old woman presents with recurrent bilateral breast lumps and pain. She states that it seems to be worse during the last few days of her menstrual cycle; the lumps appear to get smaller after her cycle. She has felt different sized lumps in her breast that occur at the same time as the pain. On examination, several small, nodular lesions are noted in both breasts; they are freely movable. The axillary lymph nodes are unremarkable bilaterally. What is the most likely diagnosis?

Steroids Supportive care Antibiotics Transfusion Bactericidal/permeability-increasing protein Antibiotics

A 31-year-old female nurse who works at a local hospital presents with a purpuric rash covering her arms, legs, and abdomen, as well as fever, chills, nausea, abdominal tenderness, tachycardia, and generalized myalgias. Prior to the development of the rash, the patient noted that she had a headache, cough, and sore throat. Laboratory studies were positive for gram-negative diplococci in the blood, along with thrombocytopenia and an elevation in PMNs. Urinalysis showed blood, protein, and casts. Vital signs are as follows: BP 92/66, P 96, RR 14, T 39. The patient denies any foreign travel and does not have any sick contacts. She is admitted to the hospital and placed in respiratory isolation. Question What major course of therapy should this patient receive?

Glucose Lorazepam Phenytoin Phenobarbital Naloxone Lorazepam

A 31-year-old male patient with a known generalized seizure disorder is brought to the emergency department. His friends say that the patient had a seizure and did not wake up. When he did not wake up after 30 minutes, they called 911. On examination, he is breathing and his heart is beating. He is warm, dry, and appears well-oxygenated. Basic laboratory values are within reference range. Computerized axial tomography (CAT) scan of the brain is unremarkable. Emergency electroencephalogram (EEG) is not available. His only medications are phenytoin and phenobarbital. What is the most appropriate initial treatment for this patient?

Hemolysis of red blood cells in the fetus High red blood cell count Increased maternal IgG Rh factor is present in the fetal blood Swelling of the fetus Hemolysis of red blood cells in the fetus (Coombs: break down of blood) (bilirubin is the break down of heme)

A 31-year-old multigravida known to have blood group A and Rh-negative red blood cells is pregnant with her third child. Her husband is also type A, but he is Rh-positive. She has an indirect Coombs titer at 1:16 dilution of her serum at 28 weeks gestation. Her past medical history includes two pregnancies; her first child (a boy) was healthy, while the second child (a girl) was born at 36 weeks gestation after the mother was noted to have an indirect Coombs titer at 1:16. Amniotic fluid obtained at 26, 28, 30, 32, and 34 weeks of gestation was analyzed by determining the optical density (OD) for bilirubin and indicated a progressive increase in the bilirubin level. What is the most likely explanation for the increase in bilirubin?

Cervix Endometrium Ovary Vagina Vulva Vulva (lump and itching outside)

A 31-year-old patient presents for genital itching that has occurred for a while. They have self-treated with OTC medication for a yeast infection, which has not helped. Now the patient notices a lump, which appears to be raised and nodular on exam. Menstrual cycles have remained unchanged since puberty. The patient is G0P0 and has been taking oral contraceptives for 11 years. Family and social history are insignificant. This presentation is most consistent with what type of cancer diagnosis?

Addison's disease

A 31-year-old woman presents due to being "really tired." In addition to this extreme fatigue for the last several months, you note the patient has had unintentional weight loss of 20 lb, decreased appetite, salt cravings, nausea and mild vomiting, irritability, and loss of sexual interest. Physical examination reveals a hypotensive female patient who appears her stated age but fatigued. She has multiple areas of hyperpigmented skin located in her bilateral palmar creases, both axillary regions, and areas in the oral mucosa. She has noted thinning of body hair throughout. What is ranked highest as your diagnosis at this time?

Intrauterine device (few years)

A 31-year-old woman presents with her husband to discuss methods of temporary contraception. They explain that they want to have children in the future but would like to wait a few years. They are both in good health. The woman's routine pelvic and physical examinations show a healthy young woman; her past medical history is unremarkable. She is a non-smoker and there is no family history of breast, ovarian, or uterine cancer. What would be the most effective means of temporary contraception for this couple?

Periodic DEXA scans are useful in detecting steroid-induced increased bone density. Close clinical monitoring during treatment is unnecessary. A decrease in salt intake is necessary in hot weather. A reduction of steroid replacement doses is needed in stressful situations. Instructions on self-administration of IM hydrocortisone on an as-needed basis. Instructions on self-administration of IM hydrocortisone on an as-needed basis.

A 32-year-old Caucasian woman has a past medical history of Hashimoto's thyroiditis, type 1 diabetes mellitus, and pernicious anemia; she presents with a 2-year history of insidious and intermittent fatigue, anorexia, involuntary weight loss, nausea, abdominal pain, vomiting, and dizziness that is associated with position changes. Her physical exam is noteworthy for postural hypotension, with a maximum systolic blood pressure of 104 in the supine position. Additionally, she has a low-grade fever and a generalized pigment change to her skin. What health maintenance recommendation should be provided?

CT scan Laparoscopy MRI Ultrasound X-ray Ultrasound (fast and easy)

A 32-year-old G2P2 woman presents 1 month post-intrauterine contraceptive device placement for follow-up position check. The patient states that she can no longer feel the strings from the device. She was told to check occasionally to make sure it stayed positioned properly. Upon vaginal exam, you confirm that the strings are no longer visible in the cervical os. What is the most appropriate procedure to evaluate this patient?

Vitamin C deficiency

A 32-year-old female patient presents with a 1-month history of bleeding gums when brushing her teeth. She reports that her wounds are taking longer than usual to heal. She is a stay-at-home mother and is breastfeeding her 6-month-old twins. On examination, multiple splinter hemorrhages on the nails and ecchymoses over the lower limbs are noted. What is the most likely diagnosis?

Candida spores and hyphae on vaginal wet mount NAAT for detection of mRNA Vaginal pH of 5.2 Numerous white blood cells in the vaginal exudates Red blood cell casts upon urinalysis Vaginal pH of 5.2 - BV (Baterial vaginosis) = foul smell and the thin discharge

A 32-year-old female patient with no significant past medical history presents with a 4-day history of "thin, light-grayish, uniform consistency" vaginal discharge. Last sexual intercourse encounter was 1 week ago; it was unprotected. The patient is not in a monogamous relationship. There is no fever, chills, swollen glands, dysuria, hematuria, urinary frequency, dyspareunia, or back pain. There is no vulvar or vaginal pruritus. Physical exam includes a pungent ammonia-like scent with an associated thin gray-white vaginal exudate, but it is otherwise unremarkable. Microscopic evaluation of the vaginal exudates is remarkable for the presence of clue cells. What additional laboratory finding is expected in this patient?

Herpes simplex virus Adenovirus Influenza A West Nile virus Epstein-Barr virus Herpes simplex virus (encephalitis)

A 32-year-old man is admitted to the hospital following loss of consciousness. The patient had been ill with fever and headache for several days, then developed double vision, confusion, and loss of consciousness. He has been otherwise healthy with no past medical conditions. Imaging reveals edema of the frontal lobe. What virus most likely caused this patient's infection?

Musculocutaneous Median Axillary Radial Ulnar Axillary (shoulder - axillary)

A 32-year-old man is working on his farm when a wild horse suddenly jerks the bridle, forcing the man's right arm over his head and backward. There is a "clunking" sound, and the man is in immediate pain and unable to move his arm. There is a deformity of the shoulder with a depressed area (dimple) noted in the anterior shoulder. What nerve must be carefully assessed in this patient before and after it is fixed because of the possible injury to it?

Acute multiple sclerosis Central nervous system lymphoma Herpes simplex virus encephalitis Multifocal leukoencephalopathy Viral meningitis Herpes simplex virus encephalitis (gadolunium-enhacing lesions) - temporal

A 32-year-old man presents with a 2-day history of high fever and progressive severe headaches that are associated with blurred vision and increasing confusion. The patient is normally healthy, and he does not have a remarkable past medical history. He is married. He does not drink alcohol, smoke, or use recreational drugs. He has not had any blood transfusions and takes no medications. On physical examination, he appears ill and disorientated to time, place, and person. His pulse is 110 bpm, temperature 39°C (102°F), respirations 22/minute, and blood pressure 115/70 mm Hg. He is well-hydrated. He has no scleral icterus, pupils are equal and reactive, and fundi are normal. His cranial nerve examination is intact. He does not respond to pain stimuli appropriately. He can move all of his limbs. Deep tendon reflexes are normal; plantar reflexes are equivocal. His neck is supple and there is no palpable adenopathy. Skin exam is normal without rashes. His lab work includeswhite blood cell count (WBC) = 18x109/L with 70% polymorphonuclear neutrophils and 30% lymphocyteserythrocyte sedimentation rate = 90 mm/hour An MRI scan of the brain demonstrates gadolinium-enhancing lesions in the temporal region. What is the most likely diagnosis?

Antigen binds to IgE, leading to the release of histamines Growth of a tumor near the base of the skull, leading to pressure on the cerebellum Increased emotional stress, leading to contractions of head and neck Increased trigeminal nerve and parasympathetic activity, leading to vasodilation Reactivation of varicella-zoster virus, leading to inflammation in a ganglion Increased trigeminal nerve and parasympathetic activity, leading to vasodilation

A 32-year-old man presents with a severe headache; he has had 2 similar headaches within the past week. He describes a burning, "hot poker" type of pain located primarily behind his right eye. He notes that his eye waters profusely with the headache. His nose is initially congested, then it starts running. Only his right side is affected. The headache is so severe that he cannot work or sleep through it, and he is unable to concentrate on anything else. The headaches have been unresponsive to over-the-counter pain medications. The episodes seem to last about 1 hour. He denies any other symptoms. This patient has no chronic medical conditions, and he takes no regular medications. What is the most likely underlying pathophysiology of this patient's condition?

Bursitis Fibromyalgia Gout Rotator cuff tear Tendinitis Tendinitis (shoveling)

A 32-year-old man presents with pain in his right shoulder. He tells you he plays softball every weekend and does a lot of shoveling around his house. The shoulder has been sore for some time, but now it hurts to the point where he tries to avoid using it. The drop arm test is negative, and there is no redness, warmth, or obvious swelling, but the patient is unable to lift his arm up to 90° without pain. What is the most likely diagnosis?

Ventricular fibrillation QT prolongation Fusion of QRS-T Torsades de pointes Short Q-T interval Short Q-T interval - alot of calcium with the stones and constipation

A 32-year-old man reports anorexia, constipation, fatigue, thirst, weakness, drowsiness, nausea, and muscle pain that has developed over the last few days. He has been bedridden for the last 3 months after a traffic accident wherein his 3 cervical vertebrae were fractured. A few weeks ago, a diagnosis of kidney stones and chronic renal insufficiency was established. The rest of his personal and family history is non-contributory. His physical examination today demonstrates quadriplegia, and the rest of examination is within normal limits. While waiting for the laboratory results, you order an EKG. What do you expect it to reveal?

Resume oral contraceptive pills Repeat ultrasound in 2-3 months Plan an oophorectomy Schedule a cystectomy Perform a hysterosalpingogram Repeat ultrasound in 2-3 months - no family history - cystic -one sided

A 32-year-old nulliparous woman presents for a routine gynecological exam. She has been married for 5 years and plans to start a family in the near future. Menarche occurred at age 11. Menstrual cycles are regular, occurring every 28-30 days, and lasting for 4-5 days each. She denies menorrhagia and dysmenorrhea. She has used oral contraceptive pills since age 18; she recently discontinued them and began taking prenatal vitamins. On physical examination, you palpate a mass on the left side. A transvaginal ultrasound confirms a 3 cm complex cystic mass on the left ovary, without free fluid detected within the pelvis. The patient has no family history of any malignant tumors. What is the best therapeutic approach for this patient?

Uterine prolapse Placenta accreta Intra-amniotic infection Uterine atony Uterine rupture Uterine atony

A 32-year-old woman is 2 hours status post cesarean delivery of a singleton gestation at 38 weeks. The course of her pregnancy was uncomplicated. She presented in early labor, which became prolonged despite oxytocin infusion. A cesarean section was performed when she failed to progress in labor and there were signs of fetal distress. While in the recovery area, she begins to complain of nausea and lightheadedness. On exam, her heart rate is 133 beats per minute and blood pressure is 76/42 mm Hg. Significant vaginal bleeding is noted, and abdominal palpation reveals a soft "boggy" uterus. What is the most likely diagnosis?

"Take a pregnancy test before starting therapy."

A 32-year-old woman presents for a follow-up visit. She was diagnosed as having hyperthyroidism 2 weeks ago, after presenting with tremors, heat intolerance, weight loss, and diarrhea. She was prescribed propranolol, pending the results of her test during her last visit. She feels slightly better now. She denies any family history of thyroid disorder and has no drug allergies. She has a supportive husband. She does not smoke, drink, or use recreational drugs. Her physical examination is normal. She has been reading about management of hyperthyroidism; she prefers the iodine-131 treatment. What is the most appropriate response to her request?

Anger Acceptance Grieving Denial Impotence Denial (feeling denial because he does not think the nurses are fixing him)

A 40-year-old man presents with stage 4 liver cancer. The patient, despite the diagnosis, continues to make business transactions over the hospital telephone and tells everyone he is fine. He gets angry with the nurses for thinking he is sick and giving him medicine. What is this man experiencing?

Normal CSF: serum glucose ratio Increased CSF protein Extremely elevated opening pressure Increased CSF glucose Predominance of polymorphonuclear cells Normal CSF: serum glucose ratio (normal because of the seizure)

A 32-year-old woman presents several hours after a grand mal seizure. Her husband states that she experienced headache, nausea, vomiting, fever, and "was not herself" a few days before the seizure. She is on glucocorticoid therapy due to a kidney transplant 1 year ago. On examination, you find a lethargic febrile (39°C) dysphasic patient; there is right-sided hemiparesis, and meningeal signs are present, but detailed examination cannot be performed because of lack of cooperation. EEG shows focal abnormalities over temporal lobes, and her CT is normal. A lumbar puncture is performed. Based on your suspected diagnosis, you initiate supportive care, anticonvulsants, and acyclovir. The patient improves over the following days. What additional finding would support your suspected diagnosis?

Bartholin gland cyst

A 32-year-old woman presents with a 2-day history of having a vaginal "bump"; the bump is painful to sit on. She has never had this problem before, and she has been monogamous with the same sex partner for 7 years. On physical exam, you notice a solitary 2 cm smooth, slightly tender mass at the introitus. A KOH/wet mount demonstrates squamous cells with no white blood cells (WBCs), hyphae, or motile organisms. Whiff test is negative. Refer to the image. What is the most likely diagnosis?

Thenar atrophy

A 32-year-old woman presents with a 3-month history of right hand pain and paresthesia of the thumb, index, and middle fingers. She works in a retail sales office and spends much of the day typing. What associated physical finding may indicate an advanced case of this suspected diagnosis?

Serum ceruloplasmin Computed tomography (CT) of the head Acid-fast bacilli (AFB) smear Genetic testing Rapid plasma reagin (RPR) testing Genetic testing

A 32-year-old woman presents with increasing irritability. She reports involuntary movements of her arms and increasing incoordination. Her husband is worried because she is having trouble remembering things. She was adopted, and her family history is unknown. Physical examination is remarkable for rapid involuntary movements of fingers bilaterally, impaired ability to concentrate, slurred and disorganized speech, and difficulty responding with appropriate words or phrases when prompted. What test will best confirm the suspected diagnosis?

Essential tremor

A 32-year-old woman with a history of a tremor presents seeking help. She has experienced emotional stress from work over the past 6 months. Upon physical exam, a tremor is observed in her right hand when hands are outstretched. Tremor is also present in her head. Laboratory findings showed no abnormal findings, but her family history reveals that her father was also diagnosed with a tremor around the same age. What is the most likely explanation of the findings?

Right knee meniscal tear Tear of the medial collateral ligament (MCL) Anterior cruciate ligament (ACL) tear Iliotibial band syndrome Patellofemoral pain syndrome Patellofemoral pain syndrome (postive patellar grind test - patella)

A 32-year-old woman with no significant past medical history presents with a 3-month history of right anterior knee pain described as a dull and aching pain that is "right under the kneecap." Provocative activities include bending movements, descending stairs, and performing squatting maneuvers. Pain is relieved during rest. She notes that she loves the outdoors and her hobbies include running and hiking; her symptoms began following a run. She denies a history of falls, prior surgeries or instrumentation, fever, chills, malaise, myalgias, changes in weight, joint swelling, skin changes or rashes, or other joint pains. Her physical exam is normal with the exception of a tender undersurface of the patella, with crepitus upon passive range of motion of the right knee. There is abnormal patellar tracking upon right knee flexion and apprehension of the patient upon passive manipulation of the patella. Additionally, there is a positive patellar grind test. McMurray's, Lachman, the anterior and posterior drawer, Apley's compression and distraction tests, and varus/valgus tests are all negative. There is no joint line tenderness, effusion, or restriction of range of motion of the right knee. What is the most likely diagnosis?

Hypercalcemia Hyperkalemia Hypocalcemia Hypokalemia Hyponatremia Hypercalcemia

A 33-year-old African American woman with a PMH of sarcoidosis presents with loss of appetite. She reports constipation and lethargy. She thinks that her urine output has increased. On physical examination, some muscle weakness and hyporeflexia are present. Laboratory tests are ordered and are pending. An EKG shows a shortened QT interval. What electrolyte abnormality would be expected?

Osteoarthritis Gouty arthritis Osteopenia Osteomyelitis Psoriatic arthritis Osteomyelitis

A 33-year-old Caucasian man with a history of severe type 1 diabetes presents due to a red swollen pinky toe on his left foot. He was running to answer his home phone and stepped on an exposed carpet tack 2 days ago. He washed it out and covered it as soon as he could. Since then, he has noted a significant increase in redness and severe restriction of movement of this toe; these characteristics have worsened tremendously in the last 12 hours. In addition, he admits to "feeling feverish" for the past day, and he experienced significant night sweats the previous night. He denies formally measuring his temperature. He has tried to take acetaminophen 500 mg, 2 tablets every 8 hours without any noticeable relief. Pertinent physical examination findings include an oral temperature of 102.8°F, significant erythema and edema extending the entire fifth toe of the left foot accompanied by restricted range of motion due to tenderness and swelling. There is a small amount of purulent drainage at the site of the wound. What is the most likely diagnosis?

5 days 1 week 10 days 2 weeks 6 weeks 6 weeks (osteomyelitis treament is typically 4-6 weeks)

A 33-year-old Caucasian man with a history of severe type 1 diabetes presents with a red and swollen pinky toe on his left foot. Further details from the patient include that he was running to answer his phone at home 2 days ago and stepped on an exposed carpet tack. He washed it out and covered it as soon as he could, but he has noted a significant increase in redness and severe restriction of movement of this toe, worsening significantly over the last 12 hours. He admits to "feeling feverish" for the past day and experiencing significant night sweats the previous night. He denies formally measuring his temperature. He has tried to take acetaminophen 500 mg 2 tablets every 8 hours without any noticeable relief. Pertinent physical examination findings include an oral temperature of 102.8°F, significant erythema and edema extending the entire fifth toe of the left foot, and restricted range of motion due to tenderness and swelling. There is a small amount of purulent drainage at the site of the wound. Based on the most likely diagnosis, how long will this patient need to be on antibiotics?

Combination estrogen-progestin pill Ethinyl estradiol/norelgestromin patch Levonorgestrel IUD Tubal ligation Total abdominal hysterectomy Levonorgestrel IUD

A 33-year-old G5P4 woman presents to her gynecologist. She has been using condoms and would like to discuss alternative birth control options. She has regular periods and is not currently trying to get pregnant. She is sexually active with one partner. Past medical history includes asthma, deep vein thrombosis during her first pregnancy, and a C-section for her fourth pregnancy. She has no known drug allergies. What is the best birth control option for this patient?

Physostigmine Naloxone Acetylcysteine Sodium bicarbonate Phenytoin Sodium bicarbonate Amtitrpyline = TCA sodium channel blockade Treatment: sodium bicarbonates

A 33-year-old female patient presents after being found unresponsive in her bedroom. She has a past medical history of depression, and her mother found an empty bottle of amitriptyline by the bedside. On physical exam, pulse 138/minute, blood pressure 80/60 mm Hg, temperature 101.2°F (38.4°C), respirations 6/minute. She moves her limbs to painful stimuli. Her skin is flushed. The electrocardiogram demonstrates a wide complex tachycardia without ectopy. The patient is intubated and is being hyperventilated. What is the next step in this patient's management?

Referral to general surgeon

A 33-year-old man with no past medical history presents with groin mass. He denies pain and trauma to the region. When the patient stands, there is a round swelling in the inguinal area. If the patient is supine, the mass disappears. What is the appropriate intervention in this patient?

Weight gain Reduction of running exercises Prescription for narcotic analgesia Referral for knee arthroscopy Cast immobilization Reduction of running exercises

A 33-year-old man with no significant past medical history presents with a 2-month history of persistent right knee pain. The knee pain is located in the anterior part of the knee; "behind the kneecap," according to the patient. Pain is worse as he descends stairs, performs squatting maneuvers, and sits for excessive periods of time. He is an avid runner and states that running also increases pain. He denies any trauma, falls, accidents, or prior surgeries. He further denies any fever, chills, insect bites, rashes, effusions, grinding, popping, or clicking sensations in the knee. He denies any hip or ankle pain. The physical exam reveals tenderness to palpation along the medial undersurface of the right patella and a positive patellar apprehension test. The anterior and posterior draw tests, McMurray's test, and Apley's compression and distraction tests are all negative. What is the most appropriate clinical intervention for this patient at this time?

Bradycardia Hypertonic uterus Hypotonic uterus Pedal edema Suprapubic tenderness Hypertonic uterus (retroplacental hemorrhage)

A 33-year-old primigravida female patient is brought to the hospital at 34 weeks gestation due to back pain, cramping abdominal pain, and vaginal bleeding for 3 hours. She has had hypertension for 3 years. She has smoked half a pack of cigarettes daily for 10 years. Blood pressure 130/84 mm Hg, respirations 19/min. A transabdominal ultrasound demonstrates retroplacental hemorrhage. Fetal examination shows tachycardia and mild late decelerations. Question Physical examination is most likely to show what clinical feature?

Hyperkalemia Metabolic acidosis Elevated FSH and LH levels Hypoglycemia Elevated urinary free cortisol (UFC) levels Elevated urinary free cortisol (UFC) levels

A 33-year-old woman presents due to a 15-pound weight gain over 2 months; there is also muscle weakness, menstrual irregularities, amenorrhea, infertility, skin bruising, memory loss, and periods of depression. She denies any medication use or dietary changes; she has tried to lose weight unsuccessfully through increased exercise. She denies any headache, vision changes, hearing changes, chest pain, abdominal symptoms, polyuria, polydipsia, or breast discharge. Her physical exam reveals a blood pressure of 145/94 mm Hg. Her skin physical exam is remarkable for the findings in the image. Refer to the image. What is an expected diagnostic finding in this patient?

Screening colonoscopy beginning at age 50 Screening colonoscopy beginning at age 45 Screening colonoscopy beginning at age 40 Screening colonoscopy if the patient becomes hemoccult positive Immediate screening colonoscopy Immediate screening colonoscopy - (colonscopy 10 year before age of family member)

A 33-year-old woman presents for an annual physical. She has past medical history of GERD, asthma, and irritable bowel syndrome. She drinks 1-2 alcoholic beverages per week and has never smoked; she does not use illicit drugs, and she consumes a vegetarian diet. Her past surgical history includes an appendectomy at age 14. Her father passed away from a heart attack at age 63. Her mother is alive with history of colorectal cancer, which was diagnosed at age 41. What is the recommended colorectal cancer screening for this patient?

Primary lung cancer Metastatic osteogenic sarcoma Lymphoma Pneumonia Pulmonary thymoma Metastatic osteogenic sarcoma (history of osteogenic sarcoma)

A 33-year-old woman presents for routine follow-up with an abnormal computed tomography (CT) that was done due to gradual worsening exertional dyspnea. Her past medical history includes a leg amputation 4 years ago for osteogenic sarcoma treated with neoadjunctive chemotherapy. She is married and a lifelong nonsmoker. Her physical exam includes a BP 111/67 mm Hg, pulse 70/min, respirations 14/minute, temperature of 98.4°F. She is in no acute distress. Pulmonary exam reveals clear breath sounds bilaterally. There is a healed laparotomy incision for a colon resection from perforated diverticulitis several years ago. A chest CT demonstrates a new solitary irregular non-calcified 3 cm nodule. What is the most likely diagnosis of this nodule?

Consume cranberry juice. Change contraception method. Begin oral probiotics. Vaginally douche regularly. Decrease fluid intake Change contraception method

A 33-year-old woman presents with a 1-day history of urinary frequency and dysuria. She is sexually active with one partner and uses a diaphragm with spermicide for contraception. Based on positive leukocyte esterase dipstick test and abnormal urine culture, she is diagnosed with a urinary tract infection (UTI). During a follow-up visit, she tells you that this is the third episode of UTI in the past few months and asks you what she can do to reduce future risk. What step can she take to reduce the risk of infection?

Age Sex Depression BMI Pregnancy Sex (Women get Rheumatoid arthritis 3x more than men)

A 33-year-old woman presents with joint pain and morning stiffness that lasts more than an hour. Her past medical history is significant for postpartum depression 2 years ago. Her BMI is 18. Physical examination is remarkable for bilateral pain and swelling of her metacarpophalangeal (MCP) joints and ulnar deviation. Laboratory evaluation reveals positive antibodies to citrullinated peptide. Based on the most likely diagnosis, what is the primary risk factor in this patient's case?

Flucloxacillin-induced hepatitis Imipramine-induced hepatitis Oral contraceptives Chronic hepatitis from hepatitis C virus Imipramine use Oral contraceptives

A 33-year-old woman presents with spider angiomas on her trunk and face. They appeared during pregnancy and have gotten worse in the 9 months since delivery. She takes imipramine for depression and resumed oral contraceptives after delivery. She had acute infection with hepatitis C virus 3 years ago. She was also frequently treated with flucloxacillin during the past few years for recurrent respiratory tract infections. Physical examination reveals spider angiomas on the patient's face, forearms, and back. Laboratory analyses reveal AST (26 IU/L), ALT (22 IU/L), and alkaline phosphatase (43 IU/L) levels within reference ranges. HBsAg, HBeAg, antiHbc antibodies, and HCV RNA are negative, and IgG antibodies are present. What is the most likely cause of the patient's spider angiomas?

Ulnar nerve Median nerve Radial nerve Musculocutaneous nerve Posterior interosseous nerve Median nerve (carpal tunnel syndrome)

A 33-year-old woman presents with tingling and numbness in the palms, thumbs, and index fingers bilaterally. Her symptoms are worse during the night. Recently, she noticed that she has difficulty grasping small objects. She works at a poultry farm and is involved in assembly packing. On examination, Tinel's sign and Phalen's test are positive. What nerve is compressed?

Electrolyte panel Oral glucose tolerance test Random plasma glucose Urinalysis Fasting blood glucose Random plasma glucose

A 34-year-old female patient presents with frequent urination over the last few months. She has not seen a physician in 5 years (since her last pregnancy). She has been overweight for most of her adult life, but she has recently had a 15-lb unintentional weight loss. She reports a history of recurrent vaginal yeast infections. What laboratory test will be most helpful in diagnosing this patient's most likely underlying condition?

Genetic testing for possible familial cancer syndrome

A 34-year-old male patient presents for his first doctor's appointment. He had always neglected his health, but he finally sought medical attention at his wife's insistence. He explains that he was from an "unhealthy family" and that he was tired of being around doctors while growing up, so he avoided medical attention. His father died at age 30 of "very high blood pressure" and "heart failure." His older brother underwent an operation to remove a cancer from his neck. Before that, the brother had surgery to remove a mass from his adrenal gland. The patient wants to know what he can do to be healthy. Blood pressure and BMI are unremarkable. He does not use alcohol or tobacco, and he exercises regularly. QuestionHighlights What is the most appropriate next diagnostic step?

IV opioids Oral acetaminophen Oral NSAIDs PCA opioids Topical NSAIDs IV opioids (compartment sydrome)

A 34-year-old man is brought to the emergency department after a single-car motor vehicle collision. Preliminary radiologic studies show a comminuted fracture of the left tibia. The patient is describing a markedly increasing amount of pain felt in the injured extremity. He describes the pain as being a 10/10 that is becoming worse with each passing minute. With anguish on his face, he describes it as a deep achy burning pain. You quickly examine the left leg and note pallor, a tense "wood-like" feeling of the extremity, diminished sensation, and muscle weakness. Based on the most likely diagnosis, what is the best pharmaceutical intervention for this patient's pain?

Pain out of proportion to exam Calf compartment pressure 5 mm Hg Erythematous streaks Palpable calf cord Groin lymphadenopathy Pain out of proportion to exam (compartment syndrome)

A 34-year-old man presents for evaluation of right calf pain after being struck by a car. He reports burning pain in his calf and numbness and tingling in his right foot. Physical exam is remarkable for calf swelling, increased pain with passive muscle stretching, and muscle weakness and decreased sensation in his right ankle and foot. What other findings are expected in this patient?

Check serum uric acid level; if elevated, initiate therapy with indomethacin and colchicine Obtain plain radiograph of the right ankle to assess for chondrocalcinosis. Obtain plain radiograph of the right ankle to assess for structural damage. Perform arthrocentesis of right ankle with analysis of synovial fluid. Treat with ibuprofen and have patient return to office in 1 week for follow-up Perform arthrocentesis of right ankle with analysis of synovial fluid

A 34-year-old man presents with a 2-day history of right ankle pain and swelling. He reports experiencing discomfort with bearing weight, ambulation, and when driving a car. On further questioning, he denies experiencing a recent trauma, although he does recall spraining his ankle approximately 1 year ago. The patient is not married and is heterosexual; he usually uses condoms as contraception, "but not every time." On physical examination, the patient's temperature is 99.9°F. His right ankle shows swelling, is warm to palpation, and reveals an effusion. With passive range of motion of the right ankle, significant pain is elicited. What is the most appropriate next step in the management of this patient?

Fasciotomy (compartment syndrome)

A 34-year-old man was the driver in a single-car motor vehicle accident. Preliminary radiologic studies show a comminuted fracture of the right tibia. The patient is describing a substantially increasing amount of pain felt in the injured extremity. He describes the pain as being a 10/10; it seems as if it is becoming worse every minute. He describes it as a deep achy burning pain. You quickly examine the right leg; you note pallor, a tense wood-like feeling of the extremity, diminished sensation, and muscle weakness. What is the most appropriate clinical intervention based on the most likely diagnosis?

Colposcopy (she has no done a Pap Smear in a while)

A 34-year-old multiparous woman presents for a routine Pap smear after being "too busy" to have annual exams for the past 7 years. Three Pap smears in her 20s have all been normal. She has had one episode of venereal warts in her late teens; there was no recurrence. She had two vaginal deliveries. She does not smoke. Remainder of her history is negative. Her Pap smear is reported as "atypical squamous cells of undetermined significance." The HPV test is positive. What is the most appropriate next step in regards to evaluating the patient?

Bilateral ureteral stents

A 35-year-old man presents by EMS after MVA that occurred 24 hours ago. He was found lying by the road by a local citizen after the MVA. Emergency department evaluation shows a fractured pelvis, acute kidney injury, and bilateral hydronephrosis near the upper ureters. His kidneys are of normal size with normal cortical thickness. He was previously well, and his only medications were multivitamins. He has not yet voided, and he notes decreased urine output after his accident. What will most likely assist in urine flow and resolution of this acute kidney injury?

Surgical release Oral corticosteroids Splint in flexion Rest and neutral splinting Corticosteroid injection Rest and neutral splinting (help with night pain)

A 34-year-old pregnant woman presents with what she describes as "tingling in her right arm." She is at 34 weeks of gestation. This is her second pregnancy. It comes on along her palm, and it has been increasing in severity. She also notices it more at night and while attempting to carry her shopping bags. Her pain has been so bad lately that she has been losing sleep. Examination reveals a positive Phalen's test; there is no weakness or atrophy of the thenar muscles. She is not known to have diabetes or hypertension, and she is otherwise healthy. Her thyroid function is normal. Her primary care physician advised treating with NSAIDs and vitamin supplements, neither of which have worked after 3 months of regular use. What is the next step in management?

Bone cyst Patellar fracture Ligament tear Tibial fracture Femoral shaft fracture Patellar fracture (fell on his knees)

A 34-year-old woman presents Monday morning with knee pain. She states that she was playing with her two children in their backyard over the weekend and fell onto solid ground, landing directly on her knees. She noted immediate and significant right knee pain. She notes significant swelling; it accompanies the knee pain, which she rates as an 8/10 on a numerical pain scale. Physical examination reveals significant obvious joint effusion and exquisite focal tenderness to palpation over the patellar area of the right knee; the left knee has no obvious abnormalities. Results of the anterior drawer, McMurray, and varus/valgus stress testing are within normal limits. Given the history and physical examination findings, what is the most likely diagnosis?

Checking the active range of motion of the wrist Palpating the mass for size and shape Obtaining radiographs of the wrist Transilluminating the mass Palpating the mass for tenderness Transilluminating the mass

A 34-year-old woman presents to your clinic with a 3-month history of a mildly tender mass on the dorsocentral aspect of her dominant right wrist. She says the lump "comes and goes," but this time it has stayed and become tender. She denies any known trauma of the wrist or hand. What examination technique would help to show that this mass is a ganglion cyst?

Dopamine agonists

A 34-year-old woman presents with worsening headaches. She says that the headaches are present throughout the day and that she has been feeling nauseous. She has also noticed difficulty in seeing vehicles on the freeway lately. She has had several close calls while driving due to this impairment. Her previously regular periods are now irregular, with heavy bleeding every 3-4 months. She has also noticed a milky discharge from both nipples. Her pregnancy test is negative. An MRI of the brain confirms the diagnosis. What is the best initial treatment for the most likely diagnosis?

Anal fistula

A 35-year-old Hispanic man presents due to a sore that will not heal around his rectum and anal area. This lesion has been draining pus consistently for the last week. He has had intermittent pain with this lesion (4/10 on a 0-10 pain scale) that is made slightly worse when he has a bowel movement (ranking it a 6/10). More recently, especially in the last few days, he has noted pain increases with just sitting. The patient also admits to intermittent periods of itching. He denies fever or diarrhea. No past medical history of inflammatory bowel disease. Physical examination of the anorectal area reveals excoriated and inflamed perianal skin with a palpated induration. What is this patient's most likely diagnosis?

Ascorbic acid Iron Protein Retinol Vitamin B12 Retinol (Vitamin A: dry eyes and dry conjunctiva)

A 35-year-old female patient presents with amenorrhea. She missed her last period and reports fatigue, morning episodes of nausea and vomiting, dry eyes, and difficulty seeing at night. Physical examination reveals dry conjunctiva and hyperkeratotic skin lesions. Laboratory evaluation is remarkable for a significantly elevated serum hCG. Transvaginal ultrasound reveals a molar pregnancy. Question A blood test will most likely indicate low levels of what compound?

Perianal abscess Crohn's disease Anal fissure Anal cancer Internal hemorrhoid Anal fissure

A 35-year-old male patient presents for an appointment, but he is too embarrassed to describe the problem to the nurse. After encouragement from the physician, the patient describes severe, intense itching around the anus that has worsened over several weeks. He has noticed increasingly severe and tearing pain in the anal area with each bowel movement. The pain is a 10/10 on a pain scale, and it lasts hours afterward. This intense pain makes him not want to have any bowel movements. He reports only one episode of a small amount of bright red blood on the toilet paper and on the stool itself. There is no fever, diarrhea, or previous inflammatory bowel disease diagnosis. What is most likely diagnosis?

Alcohol use Chronic constipation Hyperlipidemia Hypertension Lipoma removal Chronic constipation

A 35-year-old male patient presents with a groin mass. The patient says the mass is painless, and there is no known trauma to the region. The mass is present upon standing and disappears when lying flat. Past medical history includes chronic constipation, hypertension, and hyperlipidemia. Past surgical history includes lipoma removal from the left shoulder. The patient denies tobacco use; he drinks about 6 beers per week. Ultrasound confirms the diagnosis, and surgical repair is scheduled. What is the most significant risk factor for this patient's condition?

Colonoscopy

A 35-year-old man has a routine physical examination with no abnormal findings. His family history, however, is positive for familial adenomatous polyposis. What screening test would be best for him to obtain?

Abdominal computed tomography scanning

A 35-year-old man presents after several episodes of vomiting in the last 24 hours; there is loose stool and strong pain localized in the upper middle region of the abdomen. Physical examination indicates a temperature of 101°F and a tender epigastrium. Lab tests reveal an initial WBC count of 18x109/L. C-reactive protein level is 325 mg/L, and amylase is 130 U/L. There is a lactate dehydrogenase level of 816 U/L. The patient has no history of pancreatic disease and denies alcohol use. He is overweight. He has a history of type 2 diabetes and hypertension. He takes medicine to control his high blood pressure and obesity. What is the most appropriate next step in establishing the diagnosis?

Discontinue donepezil via a slow taper

A 65-year-old female patient received a diagnosis of mild dementia due to Alzheimer's disease. She was started on donepezil 5 mg once daily, which was subsequently increased to 10 mg daily. After 12 months, there has been no improvement in symptoms. The patient's family is encouraging the patient to stop taking the medication, and the patient agrees. What is the most appropriate course of action?

Topical nitroglycerin 0.2-0.4%

A 35-year-old man presents for medical care, but he is too embarrassed to tell the nurse his chief symptom. Eventually, he admits to severe, intense itching around his anus; it has been worsening the last several weeks. He further states that he has noticed increasingly severe and tearing pain in the anal area with each bowel movement. He ranks this pain as a 10/10 on a pain scale. This intense pain makes him try to avoid having bowel movements regularly. He admits to one episode of a small amount of bright red blood on the toilet paper and on the stool itself. Reviewing documentation on this patient reveals that this is the fourth similar episode in the last 14 months. The patient denies fever and diarrhea; according to him, he has never been diagnosed with inflammatory bowel disease. Considering the most likely diagnosis, what pharmaceutical intervention should be initiated?

Complex regional pain syndrome (forearm are noted to have some localized edema, warmer temperature, and increased hair growth compared to his right hand and forearm)

A 35-year-old man presents with a 2-week history of constant burning and throbbing pain in his left hand. The pain also seems to affect his distant forearm. Contact with normal clothing and bed sheets worsen his hand pain. He also reports that his hand is swollen. He denies trauma to his hand, but he does report an uncomplicated left wrist fracture 3 months ago that resulted from a sports accident. By the time of cast removal at 7 weeks post-injury, he denied any pain or edema in the affected limb. He was instructed to return to normal activities, as tolerated. He was initially treated with ibuprofen and opioid pain pills. He discontinued all pain medicines within 2 weeks of the fracture. He now describes his hand pain as an 8/10. On physical exam, the left hand and forearm are noted to have some localized edema, warmer temperature, and increased hair growth compared to his right hand and forearm. No rash or skin lesions are noted. With even light palpation of the affected region, the patient cries out in pain. Range of motion is decreased, and reflexes are increased the left upper extremity (in comparison to the right). The rest of his exam is normal. X-ray of the left wrist and hand are normal, with good fracture resolution. What is the most likely diagnosis?

Incision and drainage Fistulotomy Sphincterotomy Moist compresses Incision and drainage

A 35-year-old man presents with a painful perirectal lump. It began 6 days ago as a small firm mass and has gradually increased in size. As the mass has grown, it has become more tender. On examination, there is a 4 cm fluctuant red perirectal mass. What is the appropriate treatment?

Cluster headaches

A 35-year-old man presents with headaches; he describes them as occurring on one side of his head with a sensation of a sharp stabbing pain just above his left eye. The episodes last for 2 hours, have occurred daily for 2-3 weeks, and have been presenting in this pattern every 3-4 months for the past 2 years. The patient describes his nose as feeling congested during these periods; his left eye also appears red. Aggravating factors appear to be alcohol and stressful situations. He denies any family history of migraines or headaches similar to his. Diagnostic and laboratory studies are pending. What is the most likely diagnosis?

Water Physical activity Spicy foods Fiber Warm baths Spicy foods

A 35-year-old man presents with intense itching around his anus that has been worsening over the last several weeks. He states that he has noticed an increasingly severe and tearing pain in the anal area with each bowel movement. He would rank this pain as a 10/10. This intense pain makes him not want to have any bowel movements. He admits to only one episode of a small amount of bright red blood on the toilet paper and on the stool itself. The patient denies fever, diarrhea, or ever being diagnosed with inflammatory bowel disease. After confirming the suspected diagnosis via physical examination, you educate the patient to avoid what in order to help initiate healing?

Cluster headache

A 35-year-old man presents with recurrent headaches. They occur on one side of his head and feel like sharp stabbing pain just above his left eye. He notes eye redness and watering with the headaches. Each headache lasts for 2 hours, and they have been occurring daily for 2-3 weeks. He reports a similar pattern of headaches last year. He denies any known family history of headaches similar to his or migraines. Physical examination is within normal limits. What is the most likely diagnosis?

Acromioclavicular sprain Adhesive capsulitis Fracture of the clavicle Glenohumeral dislocation Rotator cuff tendonitis Rotator cuff tendonitis

A 35-year-old man presents with right shoulder pain that is becoming progressively worse. Although he visits the gym 3 times a week, over the past month he has not been able to increase the amount of weight he lifts secondary to the shoulder pain. He has not tried anything to alleviate the pain. He reports that the pain is at its worst at night while he is trying to sleep. He also reports pain while in the shower washing his hair, or when using the shoulder press machine at the gym. He denies any history of recent trauma or sports-related injury. Upon questioning, however, he reports that about 1 month ago he and his wife painted their entire house in one weekend. On exam, there is no notable swelling, atrophy, redness, or bruising of the shoulders. Point tenderness is noted over the right lateral deltoid muscle. Active ROM of the right shoulder at 80° of abduction elicits pain. Patient has a negative drop arm test, negative apprehensive test, and a positive Neer impingement test of the right shoulder. What is the most likely diagnosis?

Rest with arm in sling Modification of activity, NSAIDs, and physical therapy Corticosteroid injections Surgical treatment and corticosteroid injections Opioid analgesics Modification of activity, NSAIDs, and physical therapy

A 35-year-old man presents with right shoulder pain that is becoming progressively worse. He expresses concern that, although he visits the gym 3 times a week, over the past month he has not been able to increase the amount of weight he lifts secondary to the shoulder pain. He has not tried anything to alleviate the pain. The pain is at its worst at night while he is trying to sleep. He also reports pain while in the shower washing his hair or using the shoulder press machine at the gym. He denies any history of recent trauma or sports-related injury; however, upon questioning, he reports that about 1 month ago he and his wife painted their entire house in a weekend. Upon physical exam of the shoulders, no swelling, atrophy, redness, or bruising is noted. Point tenderness is noted over the right lateral deltoid muscle. Active ROM of the right shoulder at 80 degrees of abduction elicits pain. Patient has a negative drop arm test, negative apprehensive test, and a positive Neer impingement test of the right shoulder. What is the suggested first line of treatment?

Order a serum uric acid level and prescribe indomethacin. Wrap the right knee in an elastic bandage and prescribe physical therapy. Perform a joint aspiration with microscopic cellular assessment. Obtain a blood culture and begin empiric dicloxacillin. Order an LDH, ALP, and a plain radiograph of the knee Order an LDH, ALP, and a plain radiograph of the knee (there a mass need imaging)

A 35-year-old man with no significant past medical history has been experiencing progressive, moderately severe right knee pain for the past 3 months. Pain was initially felt only at night, but it is now constant throughout the course of the day for the last several weeks. It is especially severe upon ambulation and during knee ranges of motion, causing him to limp. He denies fever, chills, weight changes, history of gout, sexually transmitted diseases, hip or back pain, recent instrumentation, trauma, or injuries. His physical exam reveals an antalgic gait with limp, limited ranges of motion of the right knee, and a 3 cm diameter firm, tender mass at the distal femur. There is no erythema, crepitus, alignment deformity, or effusion noted. What is the next appropriate step in the management of this patient?

Normalize weight Episiotomy Vaginal childbirth Prescribe duloxetine Prescribe oxybutynin Normalize weight

A 35-year-old woman just found out she is pregnant. She is experiencing polyuria, but she denies dysuria and incontinence. Her urinalysis is unremarkable. Her fetal ultrasound is normal, and her renal ultrasound shows normal physiological hydronephrosis of pregnancy. Her pre-pregnancy weight was 155 lbs, and she is 5 feet tall. Her calculated body mass index (BMI) is 30.3 kg/m2. She takes no medications. She smokes half a pack of cigarettes per day. In this patient how can you best prevent urinary stress incontinence postpartum?

Hashimoto's thyroiditis Toxic goiter Grave's disease Plummer's disease Thyroid storm Grave's disease

A 35-year-old woman presents because of weight loss and palpitations. She lost 10 kg over 5 months despite having a good appetite. Her heart pounds and her hands tremble "all the time." She feels hot, is sweating profusely, and has difficulties going to sleep and maintaining sleep; the slightest stimulus wakes her. Her job is suffering because of her nervousness, and her supervisor became concerned because she uses the bathroom 3-4 times a day in a need to move her bowels. She thinks that poor sleep quality and frequent bowel movements make her weak; she cannot climb stairs anymore and has to take a rest every 10 steps or so. Physical examination reveals a slim anxious woman with pronounced stare, fine postural hand tremor at rest, and slight proximal weakness. Her thyroid is diffusely enlarged and non-tender; her pulse is 100/min; the rest of examination is within normal limits. What is the most likely diagnosis?

Autoimmune Infectious Neoplastic Inflammatory Malingering Autoimmune

A 35-year-old woman presents with a 2-month history of palpitation and nervousness. She mentions that she always feels hot, even if the weather is cold. Her menses have been irregular lately, and she has had no fevers recently. She was also told that her eyes are "weird-looking." On examination, blood pressure is 150/70 mm Hg, and pulse is 89 beats per minute. Her eyes show exophthalmos, and she has lid lag when looking down. Refer to the image of her thyroid scan result. What is the nature of this patient's illness?

Fibroadenoma Breast carcinoma Breast cyst Fat necrosis Tuberculosis Fibroadenoma (young and bengin)

A 35-year-old woman presents with a single firm, well-delineated, round, non-tender nodule in her left upper breast. It is very mobile with respect to its surrounding tissue. What is the most likely etiology of this mass?

Determine TSH levels. Evaluate for infectious etiology. Evaluate for malignancy. Perform ECG. Refer to a psychiatrist for panic attacks Determine TSH levels

A 35-year-old woman presents with excessive sweating and fatigue for the last few months. She has lost 4 kg (8.81 lb) over that time despite having an increased appetite. Her menstrual cycle has been regular with moderate bleeding. She has no significant past medical history. She has never been pregnant and denies smoking, alcohol consumption, and recreational drug use. Temperature 37.2°C (99°F), blood pressure 126/88 mm Hg, pulse 90/min. Her neck is unremarkable on palpation. She has unremarkable bilateral breath sounds. S1 and S2 are unremarkable, and no murmurs are heard on auscultation. What is the most appropriate next step in the diagnosis of the patient?

Hemoglobin electrophoresis Glucose-6-phosphate dehydrogenase assay Echocardiography Hepatitis B panel Direct agglutination test Echocardiography

A 35-year-old woman presents with fatigue and yellowish coloration of her eyes and skin that started several weeks after non-eventful implantation of a prosthetic mechanical heart valve 6 weeks ago. She denies any history of similar episodes. She has a history of severe aortic stenosis. Other past medical history is non-contributory. Physical examination reveals the presence of regurgitant murmur and subicterus. Laboratory results: hemoglobin 7.0 g/dL, reticulocytes 21%, WBCs 11,500/µL, platelets 80,000/µL, undetected levels of haptoglobin. Lactate dehydrogenase (3100 U/L), direct bilirubin (2.1 mg/dL), and indirect bilirubin (1.2 mg/dL) levels are all elevated. Peripheral blood smear shows burr and helmet cells (schistocytes) and polychromasia. Direct and indirect Coombs tests are negative. You suspect microangiopathic hemolytic anemia. What is the next step in management?

Semen analysis

A 36-year-old G1P0010 woman and her 40-year-old husband present for a family planning visit as they have been trying to conceive for 10 months without results. She has history of one spontaneous abortion five years ago. The spouse has never fathered a child. Neither partner has any other significant past medical or past surgical history. Physical examination of both partners is unremarkable. What do you advise as the next step in the couple's evaluation?

de Quervain tenosynovitis (holding a baby, diffcult to grasp something, and ulnar deviation)

A 36-year-old G1P1001 woman presents with a 4- to 5-week history of pain along her right lateral wrist; the pain worsens when she tries to grasp something. There was no trauma. She gave birth 6 weeks ago and carrying her baby is difficult secondary to pain. She is right-handed and has never had any issues like this before. On exam, there is tenderness and edema over the radial styloid. You then have the patient fully flex her thumb, adduct, and grasp it with that hand. You then place her hand in ulnar deviation, which reproduces the pain described above. What is the most likely diagnosis?

Intravenous pyelogram Plain radiography Ultrasonography Non-contrast computed tomography CT

A 36-year-old man presents with severe left flank pain that started at 4 AM. After pacing around his house for several hours and taking ibuprofen and acetaminophen (Tylenol), he comes to the office at 7:30 AM asking to be seen. He describes the flank pain as burning and steady, with some radiation to the left lower abdomen. He has had some nausea, and he unsuccessfully tried to vomit. He has not had any diarrhea or urinary symptoms. He describes his past medical history as negative for significant illnesses or injuries. Physical exam reveals normal vital signs except for tachycardia, a negative heart and lung exam, and a negative abdominal exam. There is no tenderness noted in the CVA regions. Which test is recommended to most accurately confirm the diagnosis?

As needed anxiety medication Decrease or stop caffeine intake Oral steroid taper Oral daily levothyroxine Antidepressant medication Decrease or stop caffeine intake

A 36-year-old man with a history of diabetes and obesity presents with weakness and flu-like symptoms. His girlfriend reports that he had taken several caffeine pills the day before, but he denies a suicide attempt. During evaluation at the hospital, he experiences vomiting and seizures. A laboratory workup is within normal limits. He is admitted to the hospital and improves over the course of his 3-day admission. He is subsequently released with no lingering effects. What is the most common treatment for this patient after hospital discharge?

Subinvolution of the placental site Gestational trophoblastic disease Endometritis Disseminated intravascular coagulation Leiomyoma uteri Subinvolution of the placental site

A 36-year-old woman gravida 1 para 1 came to the emergency department with severe abdominal pain and two episodes of profuse vaginal bleeding, chills, and light-headedness 10 days after cesarean delivery. Because of her age, she was closely monitored during the pregnancy and several ultrasound examinations were performed. Today, physical examination reveals tender and firm abdomen with bloody vaginal discharge; her pulse is 100/min; blood pressure of 100/60 mm Hg; and temperature 36.0°C. Laboratory reveals a white blood cell count of 10,000/μL and hemoglobin level of 11 g/DL. Coagulation tests and urine beta hCG are within normal limits. Ultrasound reveals low-resistance vessel in the inner third of the myometrium. What is the most probable cause of her bleeding?

Bipolar II disorder Persistent depressive disorder Adjustment disorder Major depressive disorder Anxiety disorder Persistent depressive disorder (The depression has been longer than 2 year)

A 36-year-old woman presents because she has been feeling very tired and unhappy for the past 3 years; she thinks that she has no hope of better days in the future. She states that it is amazing her boss has not fired her yet because she is one of the company's worst employees. She cannot recall the last time she was excited about anything. She denies other symptoms. Her vital signs are stable. Her height and weight are within normal limits. What is the most likely diagnosis?

Defective plasma cholinesterase Immune-mediated destruction of the acetylcholine receptor Deficient quantity of plasma cholinesterase Impaired repolarization at the motor endplate Deficiency in dystrophin Immune-mediated destruction of the acetylcholine receptor (Myasthenia gravis)

A 36-year-old woman presents with fatigue and diplopia; symptoms started in the morning and worsened during her time at work. On examination, the patient has ptosis bilaterally and decreased ocular muscle power. A CT scan of the chest shows a thymoma. What is the pathophysiology of this patient's neuromuscular disorder?

Endometrial cancer

A 36-year-old woman presents with lower pelvic pain. She noticed that pain worsens during intercourse. She also reports heavy menstruation for the past 4 months. She has lost weight over the past few months. Upon physical exam, her gynecologist did a pelvic exam and felt a mass, but cervical motion tenderness and suprapubic tenderness was negative. A transvaginal ultrasound showed an increase of >4 mm of endometrial thickness. A D&C was then conducted and positive for dysplasia cells. Upon laboratory findings, the patient was negative for STDs and Escherichia coli. What is the most likely diagnosis?

Waist-to-hip ratio less than 0.8 Increased proximal muscle strength Hyperreflexia Hypertension Hypotension Hypertension

A 36-year-old woman with a past medical history of diabetes mellitus presents with weight gain and skin changes. Review of systems is positive for menstrual irregularities with extended periods of amenorrhea, infertility, depression, cognitive dysfunction, and emotional lability. Lately, fasting glucose levels have been elevated. Physical exam notes increased adipose tissue in the face, upper back, and above the clavicles. Skin reveals ecchymosis, telangiectasias, purpura along the back and lower extremities, facial acne, cutaneous atrophy, and purple striae of the abdomen. What manifestation is expected in this patient?

Cluster headache Headache secondary to brain tumor Migraine headache Subarachnoid hemorrhage Tension headache Tension headache (base of the head)

A 37-year-old Hispanic man presents with a 4-month history of mild-to-moderate headaches; on average, he gets them 3-4 days per week. He has tried over-the-counter analgesics with minimal relief. He is seeking care now because he had been promoted to store manager several months prior to presentation, and he is worried that his headaches are affecting his concentration. His headaches are generalized in location, described as starting at the base of his head and extending all over, feeling "tight" in nature. He denies memory loss, photophobia, nausea/vomiting, rhinorrhea, lacrimation, and upper respiratory symptoms associated with the headaches. He also denies seizures, syncope, incoordination, vertigo, weakness, and paresthesias. The patient mentions his concerns for his work several times. Although he enjoys his work, he admits to having some anxiety about being able to handle his new duties. His family history is negative for headache. Physical exam is performed; vitals, neurological, cardiovascular, and HEENT findings are all normal. What is the most likely diagnosis?

Epidural hematoma Subdural hematoma Subarachnoid hemorrhage Intracerebral parenchymal hemorrhage Acute meningitis Epidural hematoma

A 37-year-old man fell from a ladder as he finished working on the roof of his house. The right side of his head hit the alley cement, and he lost consciousness for about 1 minute; he woke up with a headache, but he had no other complaints. A few hours later, the patient is brought to the emergency room by his neighbor because of an intense headache, confusion, and left hand hemiparesis. On examination, the patient has a bruise located over the right temporal region, mydriasis, and right deviation of the right eye, papilledema, and left extensor plantar response. An emergency CT scan of the head without contrast reveals a lens-shaped hyper-density under the right temporal bone with mass effect and edema. What is the most likely diagnosis?

Trigeminal neuralgia Bell's palsy Multiple sclerosis Myasthenia gravis Primary lateral sclerosis Bell's palsy

A 37-year-old woman presents with a history of right-sided facial paralysis and periauricular discomfort since she awoke this morning. She is afebrile. She had a "cold sore" 1 week prior to her symptoms, but this resolved without complications. During her neurologic exam, she was discovered to have an inability to raise her right eyebrow and close her right eye completely. She also has drooping of the right corner of her mouth. The rest of her neurologic exam is normal. There are no masses or rashes evident. She denies history of prior CVA or neurologic illnesses. She does not take any medications. What is the most likely diagnosis?

Major depressive disorder

A 37-year-old woman was diagnosed as HIV-positive 3 years ago; she is unable to work and is physically debilitated, so she asks you to provide her with medications with which to take her own life. What is the most common emotional disorder associated with such a request?

Benign essential (familial) tremor Huntington's disease Multiple sclerosis Myasthenia gravis Parkinson's disease Huntington's disease (Chorea and eye movement)

A 38-year-old man with uncontrolled facial movements states that he has noticed himself over the last few months making expressions without even realizing it or being able to control it. Further questioning reveals that he also has noted an inability to intentionally move his eyes quickly without blinking. Very recently, he noted an inability to sustain physical movements, such as grasping objects with his hands. Physical examination reveals a puppet-like gait and obvious chorea. The patient admits that he does not know anything about his family history due to the fact that he was adopted when he was 4. Based on the history and physical examination findings, what is the most likely diagnosis?

Megaloblastic erythropoiesis in bone marrow

A 38-year-old woman of northern European origin is brought to the emergency room by her relatives who report abnormal behavior. The patient denies the accusation and reports numbness and a tingling sensation in both her hands and feet (gloves and stockings) and recurrent diarrhea. Physical examination shows an atrophic tongue (glossitis), and a neurologic examination reveals a loss of her sense of vibration and fine touch. Endoscopic examination shows atrophic gastric mucosa and fasting achlorhydria. Considering this patient's diagnosis, what test finding would you anticipate?

Exophthalmos

A 38-year-old woman presents with a new tremor. She also reports weight loss and heat intolerance along with increased anxiety and palpitations. She has noticed swelling of her neck. What finding on physical exam would help support your suspected diagnosis?

Propranolol Alprazolam Paroxetine Diazepam Hydroxyzine Paroxetine (anxiety med)

A 39-year-old woman presents with a history of repeated short episodes of intensely anxious and fearful moments with physiologic manifestations, such as trembling, tachycardia, dizziness, sweating, and a smothering sensation. She has these episodes almost daily and feels they greatly impact her life when they occur. She denies symptoms of agitation, insomnia, and depression, and she states she does not have a history of recreational drug use. After an extensive workup, a diagnosis is made. What is the best drug for treatment of her most likely condition?

Acetaminophen Indomethacin Prednisone Aspirin Oxycodone Indomethacin (NSAIDS and excersise for ankylosing spondylitis)

A 40-year-old man presents with ongoing back pain; there is increased stiffness in the morning, and the pain has been going on for a few months. The patient reports that it can take him up to 30 minutes after waking up for the discomfort to improve. Physical examination and diagnostic testing confirms the diagnosis of ankylosing spondylitis. In addition to regular exercise, what medication will best control the patient's symptoms?

Positional plagiocephaly Craniosynostosis Hydrocephalus Neurofibromatosis Catch-up growth Hydrocephalus

A 4-month-old female infant is presented for a well-child checkup. She was a spontaneous vaginal delivery at 39.5 weeks without complications. The mother notes she has been irritable and has not been eating well. No cough or fever. No one smokes at home. On exam, you note impaired extraocular movements, especially in the upward gaze, and a bulging anterior fontanel. There is increased tone of the legs. Skin exam is normal. Like her last visit, the length and weight are 50th percentile. Head circumference was formerly at the 75th percentile and is now above the 99th percentile. What is the most likely diagnosis?

Vitamin D

A 4-month-old girl has been gaining weight well and achieving normal milestones. She was a full-term infant born via vaginal delivery without complication. Her past medical history is significant only for a mild URI the previous month, and she is current with her vaccines. She has been exclusively breastfed since birth. Her mother eats a well-rounded diet that includes meat and dairy. What supplementation does the infant require?

Neisseria meningitidis Staphylococcus epidermidis Staphylococcus aureus Escherichia coli Listeria monocytogenes Neisseria meningitidis (common)

A 4-year-old boy currently hospitalized with pneumonia develops vomiting and stiff neck. The new symptoms are followed by a seizure. On physical examination, he is febrile and has signs of meningeal irritation. A lumbar puncture is done to determine if he has meningitis. What organism is most likely causing his meningitis?

Analgesia and local ice application Circumferential incision of the constricting band Subcutaneous epinephrine Subcutaneous terbutaline Vertical incision of the constricting band Vertical incision of the constricting band

A 4-year-old uncircumcised boy presents with a 2-day history of penile pain. The patient is afebrile and vital signs are stable. On genital examination, his foreskin is retracted proximally and the glans is edematous and cold. You are unable to reduce the proximal foreskin distally over the glans penis; it is strongly suspected that arterial flow is compromised. The only urologist available will arrive in 1.5 hours. What is the most appropriate treatment for this patient?

Synovial fluid analysis Urine protein/creatinine ratio Antibodies to the Sm antigen Cerebrospinal fluid analysis Complement system test Antibodies to the Sm antigen (Her fingers are swollen and red, and she has a malar rash and oral ulcers- Lupus)

A 40-year-old African American woman presents with diffuse headache and joint pain. The headache started few days ago; it is dull and becoming progressively worse. Joint pain is localized in fingers, starts in the morning, and improves during the day; it returns when she gets tired. 2 months ago, she was treated in the ED because of several weeks of lasting fatigue, low-grade fevers, joint pain, hair loss, and oral ulcers. Her laboratory tests were normal, except for positive VDRL and antinuclear antibody tests results. Urine and blood cultures were negative for evidence of infection, and her chest X-ray was normal. Because she felt better after a 10-day tapering course of prednisone, she did not follow up with a healthcare provider at that time. She did not appear to the scheduled control. Today, your examination reveals an ill-appearing woman in distress. Her temperature is 39°C. Her fingers are swollen and red, and she has a malar rash and oral ulcers. What additional test should you order as the most specific for her condition?

Endometrial cavity and shape Spermatogenesis and motility Endocrine and liver dysfunction Mucus and sperm interaction Size of genitalia Spermatogenesis and motility

A 40-year-old Caucasian man presents to your office to ask for advice regarding the inability of his wife to conceive. She is younger than him by 8 years and he believes she is in good health. Additionally, she has two children from a previous marriage, both pregnancies without complication. He was also married once before, but his first wife did not conceive children. He has not yet undergone any fertility tests. What is the most common cause for infertility in this case?

Autoimmune destruction of arginine vasopressin-secreting cells Deficient antidiuretic action of arginine vasopressin Inactivation of arginine vasopressin by increased production of N-terminal aminopeptidase Inappropriate thirst due to reduction in the set of the osmoregulatory mechanism Lithium-induced destruction of magnocellular neurons in the hypothalamus Deficient antidiuretic action of arginine vasopressin

A 40-year-old female patient presents with excessive thirst and urination for 3 months. She has had bipolar disorder for 15 years and has been taking lithium regularly since the diagnosis. Vitals and physical exam are unremarkable. Plasma glucose 98 mg/dL, urinary specific gravity 1.002, urinary osmolality 180 mOsm/kg. Under conditions of unrestricted fluid intake, plasma arginine vasopressin level is 1.2 pg/mL. What mechanism best explains this patient's presenting symptoms?

Start strengthening exercises now to get ready for next tournament. Prescribe him muscle relaxants since spasms cause the pain. Tell him he should switch arms to give his normal playing arm a rest. Inject him today with a long-acting anesthetic agent to relieve his pain. Prescribe an NSAID of choice and recommend skipping the next tournament Prescribe an NSAID of choice and recommend skipping the next tournament

A 40-year-old male recreational tennis player presents with pain in his playing-side elbow since a tournament last weekend. He played five matches of singles and two matches of doubles games. He cannot even hold a pen without pain. He says he needs to get better fast because he has another tournament coming in a week. On exam, he has pain with extension against resistance of his affected side wrist. What treatment option should be recommended on this first visit?

Gallstones Intestinal ulcers Macrocytic anemia Kidney stones Weight gain Intestinal ulcers

A 40-year-old man presents with a 2-year history of severe, burning epigastric pain. A detailed history reveals that the pain is greatest in the early hours of the morning and wakes him up from sleep. The pain is also felt 2-3 hours after meals. He reports diarrhea for the past 2 years. On examination, his pulse is 74/min and blood pressure 136/84 mm Hg. There is slight epigastric discomfort on palpation. Lab examination shows hyperchlorhydria. What is a potential complication of this patient's diagnosis?

Pelvic floor exercise

A 40-year-old otherwise healthy nulligravida patient presents with involuntary loss of urine that occurs after drinking a small amount of water, when washing the dishes, when hearing water running, and sometimes for no reason discernable to the patient. It is preceded by suddenly feeling the need to urinate, and it happens during the day and at night. Urinalysis and culture, pelvic, gynecological, and neurological examinations are unremarkable. Cystometric studies show residual volume of 45 mL (normal) with involuntary detrusor contractions, starting already with 200 mL. What is the most appropriate treatment for this patient?

Cholelithiasis Cholangitis Pancreatitis Hepatocellular carcinoma Gilbert's syndrome Cholangitis (fever, jaudice and abdominal pain)

A 40-year-old woman is seen in the emergency department because of right upper quadrant pain and fever. She has been experiencing episodic epigastric pain over the past few months, but this is the worst her symptoms have been. She also reports anorexia and vomiting. She is in obvious distress. On physical examination, her doctor notes the presence of jaundice. Her lab results are as follows: TestResultNormal Valuealkaline phosphatase89 U/L20-70 U/Lbilirubin (total)3.8 mg/dL0.2-1.5 mg/dLAST50 U/L10-40 U/LALT70 U/L10-60 U/Lamylase35 U/L25-85 U/Llipase80 U/L0-160 U/L What diagnosis is most likely to be the cause of her symptoms?

NSAIDs (conducted of the extremity reveals a slightly cyanotic, mottled right arm with generalized pain of the entire extremity) -complex regional pain syndrome

A 40-year-old woman presents because of a 7-day history of pain in her right arm. The patient denies any trauma or injury to this extremity just prior to the pain starting but does admit to having a Colles' fracture in this arm around 2 months ago. She denies any injury to her back, neck, or other musculoskeletal system prior to the event of pain. She describes the pain as burning and throbbing with an extremely diffuse, uncomfortable aching accompanying it. She further states that this limb has become extremely sensitive to touch and to cold, and it does appear somewhat more swollen than her left arm. The patient is very upset because she does not know why her arm is so painful when she has not done anything to it. She is a non-smoker, does not drink, and exercises 3 times a week. Physical examination conducted of the extremity reveals a slightly cyanotic, mottled right arm with generalized pain of the entire extremity. Pulses are faint (1+) and ROM is limited. A multidisciplinary approach is recommended in the treatment of this patient's most likely diagnosis. What pharmaceutical agent is recommended as part of the initial treatment plan?

Bone densitometry after menopause Hip and spinal X-ray after menopause Bone densitometry at age 65 Hip and spinal X-ray at age 65 Bone densitometry at age 55 Bone densitometry at age 65

A 40-year-old woman presents occasional constipation that she relates to her diet. She is married with two sons. Aside from mild anxiety, she is dealing appropriately with her duties as a librarian. She does not smoke, she consumes alcohol only recreationally, and she weighs 187 lb. She is well-nourished and communicates well. Lung, cardiac, abdominal, and neurological examinations are unremarkable. Thyroid is normal in size and consistency. Menses are normal, and drug history is negative except for "hormone pills" for contraception. What imaging modality would you recommend for osteoporosis screening and when should it be done?

Nephrolithiasis

A 40-year-old woman presents to the emergency department with unremitting left flank pain. She denies dysuria or fever. She notes that her urine output has decreased over the last few days. As you observe the patient, she is writhing on the gurney and unable to find a comfortable position. On further inquiry, the patient states that she has been trying to lose weight by increasing protein in her diet, exercising, and decreasing her normal fluid intake. On urinalysis, red blood cells are noted. All other labs are within normal range. Question What is the most likely diagnosis?

Acetaminophen 500 mg 2 tablets daily Acyclovir 800 mg once a day Naprosyn 500 mg twice daily Cephalexin 500 mg daily Colchicine 0.6 mg daily Naprosyn 500 mg twice daily (NSAIDs)

A 40-year-old woman presents with a 7-day history of pain in her right arm. The patient denies any trauma or injury to this extremity just prior to the pain starting, but she admits to having a Colles fracture in this arm about 2 months ago. She denies any injury to her back, neck, or other components of the musculoskeletal system prior to the event of pain. She describes the pain as burning and throbbing with an extremely diffuse and uncomfortable ache accompanying it. She further states that this limb has become extremely sensitive to touch and cold; it appears somewhat more swollen than her left arm. The patient is very upset because she does not know why her arm is so painful when she has not done anything to it. She is a non-smoker. She does not drink alcohol, and she exercises 3 times a week. Physical examination of the extremity reveals a slightly cyanotic hue and generalized pain of the entire right upper extremity. Pulses are strong (2+), but due to the pain, the patient states ROM is limited. Considering the most likely diagnosis, what pharmaceutical regimen should be initiated?

Complex regional pain syndrome

A 40-year-old woman presents with a 7-day history of pain in her right arm. The patient denies any trauma or injury to this extremity just prior to the pain starting, but she does admit to having a Colles fracture in this arm around 2 months ago. She denies any injury to her back, neck, or other musculoskeletal system prior to the event of pain. She describes the pain as burning and throbbing with an extremely diffuse, uncomfortable aching accompanying it. She further states that this limb has become extremely sensitive to touch and cold; it does appear somewhat more swollen than her left arm. The patient is very upset; she does not know why her arm is so painful when she has not done anything to it. She is a non-smoker. She does not drink, and she exercises 3 times a week. Physical examination conducted of the extremity reveals a slightly cyanotic, mottled right arm with generalized pain of the entire extremity. Pulses are faint (1+) and ROM is limited. Radiograph studies are obtained and reveal spotty areas of apparent osteopenia of the right upper extremity. What is the most likely diagnosis?

Gastroesophageal reflux disease Squamous cell esophagus neoplasm Peptic ulcer disease Esophageal varices Biliary colic Gastroesophageal reflux disease

A 44-year-old man comes in with reports of heartburn, substernal pain, regurgitation, and difficulty swallowing. He likes to eat foods that have "substance," like hamburgers, steaks, fries, rich desserts, etc. He says his wife is a great cook and prepares all his favorite dishes with extra butter. He has had heartburn for years. To relieve the heartburn, he has taken antacids. This time, the pain is worse. He has eaten a large fatty meal within the last hour. He denies other medical problems. He does not smoke and only occasionally uses alcohol. On physical exam, he weighs 280 lb and is 5'10". What is the most likely diagnosis?

TNF inhibitors NSAIDs Sulfasalazine Etanercept Calcitonin NSAIDs (Complex regional pain syndrome - NSIADs)

A 40-year-old woman presents with a 7-day history of pain in her right arm. The patient denies any trauma or injury to this extremity just prior to the pain starting, but she does admit to having a Colles' fracture in this arm around 2 months ago. She denies any injury to her back, neck, or any other musculoskeletal system prior to the event of pain. She describes the pain as burning and throbbing; there is an extremely diffuse and uncomfortable aching accompanying it. She further states that this limb has become extremely sensitive to touch and cold, and it appears somewhat more swollen than her left arm. The patient is very upset because she does not know why her arm is so painful when she has not done anything to it. She is a non-smoker, does not drink, and exercises 3 times a week. Physical examination of the extremity reveals a slightly cyanotic, mottled right arm with generalized pain of the entire extremity. Pulses are faint (1+), and ROM is limited. Keeping in mind the most likely diagnosis, what is the most appropriate first-line therapy for the patient's signs and symptoms?

Addition of ascorbic acid Increased dose of ferrous sulfate Iron dextran IM or IV in the ER Iron sucrose 200 mg IV in the ER Red blood cell transfusion Red blood cell transfusion

A 40-year-old woman undergoing a gynecologic workup for metromenorrhagia presents with a several-week history of fatigue and lightheadedness. Laboratory evaluation reveals a hemoglobin of 11 g/dL (12-16 g/dL), hematocrit of 34%, MCV of 70 fL (80-100 fL), and MCH of 24 pg (27-33 pg). Further studies reveal a ferritin level of 25 ng/mL (12-300 ng/mL), TIBC of 500 mcg/dL (250-450 mcg/dL), and an iron level of 45 mcg/dL (60-170 mcg/dL). The patient is diagnosed with iron deficiency anemia. She is prescribed ferrous sulfate 325 mg po tid. Several days later, she presents to the ER. She reports resting dyspnea and chest discomfort. Oxygen and nitroglycerin are given. Cardiac enzymes are pending. The patient's hematocrit is 23% with hemoglobin of 7.5. What treatment is most appropriate for this patient?

Abdominal x-ray Abdominal MRI Abdominal ultrasound HIDA scan Liver biopsy Abdominal ultrasound

A 41-year-old man presents with right upper quadrant pain. The pain began gradually after a meal, but it is now constant. He mentions previous episodes of similar pain, but it has never been quite this severe. The pain radiates to the right shoulder and worsens with inspiration. He has experienced nausea and vomiting, and he notes feeling chills. Examination reveals an overweight male patient in moderate distress. He develops rigors during the physical exam. There is scleral icterus, and the skin has a yellow hue. Heart and lungs are clear. Abdomen is soft and non-distended with positive Murphy sign. BP 109/62 mm Hg, pulse 112, respirations 18, and temperature 102.3°F. What is the most appropriate initial diagnostic study for this patient?

Hyperkalemia

A 41-year-old woman has been hospitalized for over a week. Her laboratory results reflect an electrolyte abnormality and her EKG demonstrates peaked T waves and a widening of the QRS complex. These EKG abnormalities are characteristic of what condition?

IgM Anti-HBc HBsAg IgG Anti-HBe Anti-HBs IgG Anti-HBc IgM Anti-HBc

A 42-year-old IV drug user is experiencing vague symptoms consisting of fatigue, aches and pains, and nausea. They have developed a distaste for their cigarettes. They appeared jaundiced for a few days, but the condition spontaneously resolved. The patient goes to the free clinic, and preliminary blood tests are run. If this patient is in the window period of hepatitis B infection, what marker will most likely be positive?

Projection Reaction formation Denial Dissociation Sublimation Sublimation

A 42-year-old man is angry, frustrated, disillusioned, and disappointed with his wife's involvement with his best friend. He enrolls himself in a men's tennis club to relieve his negative emotions. What defense mechanism is he using?

Amitriptyline Fluoxetine Propranolol Sumatriptan Verapamil Verapamil (oxgyen or CCB)

A 42-year-old man presents to the emergency department with a severe headache. He has had several of these headaches over the last few days and has tried over-the-counter pain relievers with no relief. His current headache started 15 minutes ago. The pain is located next to and behind his left eye and is "stabbing/excruciating." His left eye tears up profusely with these headaches. He reports he has been healthy otherwise, with no other significant medical history and no drug allergies. On physical exam, the patient appears slightly agitated and uncomfortable. Left eye conjunctiva is mildly injected, and lacrimation is noted. Right eye is normal. Cranial nerves II-VII are intact, but the patient expresses discomfort when the light is shown in his left eye. Speech, gait, coordination, and reflexes are all normal. The remainder of his exam is normal. Head MRI is performed and reported as normal. What medication is the most appropriate prevention for this patient's type of headache?

Obtain a urine culture and start ciprofloxacin empirically. Order an intravenous pyelogram to rule out a kidney stone. Repeat the urinalysis and observe patient. Order a 24-hour urine to quantitate urine protein. Order a renal biopsy Order a 24-hour urine to quantitate urine protein

A 42-year-old man presents with lower extremity swelling. His past medical history and review of symptoms are otherwise negative. The patient looks comfortable, with vitals showing the following: BP 142/91 mm Hg, HR 90 beats/min, RR 16 breaths/min, T 98°F, height 5'9"', and weight 158 lb. His examination is only remarkable for 2+ pitting edema in the lower extremities. The patient is counseled on a low-salt diet. The abnormal laboratory values are as follows: 3+ protein, coarse granular casts, 2-5 WBCs, 0-2 RBCs What should be the next step in the management of this patient?

Administer halopedirol. Discontinue risperidone and begin quetiapine. Increase the risperidone to 4 mg daily. Prescribe both diazepam and paroxetine. Refer to the hospital for inpatient evaluation Refer to the hospital for inpatient evaluation

A 42-year-old patient with a history of schizophrenia has begun demonstrating new and unusual behavior for 2 weeks. They have been compliant with taking risperidone 2 mg for 1 year. The patient's son says the patient has been acting "silly" and exhibiting inappropriate behavior, like removing clothes, repeating odd noises and gestures, and speaking incoherently with random loud and violent outbursts. This morning, the patient was observed to have used a knife to cut themself; the son stopped him. What is the most appropriate next step in managing this patient?

Abnormal autonomic nervous system activity Altered immune system response Demyelination of neurons Metastatic proliferation of muscle and soft tissue Vascular occlusion prohibiting oxygenation of distal tissue Abnormal autonomic nervous system activity (complex regional pain syndrome (CRPS))

A 42-year-old woman presents with a 1-month history of severe worsening pain in her right foot, ankle, and lower leg. The pain is constant and burning. She reports some initial swelling and warmth in the leg, which has lessened. Now, her right lower extremity is always cool to the touch. Hair growth has dramatically decreased on her right leg. She has tried multiple over-the-counter pain medications and topical analgesics with no relief. She denies trauma to the affected limb and reports her symptoms just "came out of the blue." Her sleep is poor, and she reports fatigue secondary to unrelenting pain, but she denies any other symptoms. The patient is thin, with normal vital signs. On neurologic exam, her right lower calf, ankle, and foot appear to be hypersensitive to light touch stimulus (cotton swab), compared to the left. The same region, which spans several dermatomes, exhibits decreased sensation to pinprick and decreased range of motion. The skin of the right lower extremity is cool; there are no lesions, rash, edema, or erythema. Ridges are noted on the right toenails, but not the left. The rest of her physical exam is normal. Blood count, glycohemoglobin, metabolic panel, D-dimer, and the X-ray and ultrasound of the entire right lower extremity are normal. What best describes the underlying pathophysiologic process in this patient's case?

Fibromyalgia Osteoporosis Polymyalgia rheumatica Rheumatoid arthritis Systemic lupus erythematous Fibromyalgia

A 42-year-old woman presents with a history of chronic fatigue and pain around her neck, shoulders, and lower back. She is also experiencing chronic headaches and irritable bowel symptoms. Upon physical exam, no abnormal findings were found except for trigger points that produced pain around the trapezius, lateral epicondyle of her elbow, and the medial fat pad of her knee. Laboratory findings showed a normal ESR, negative RF factor, and a negative ANA. What is the most likely diagnosis?

Adrenal CT Aldosterone-renin ratio Aldosterone suppression test MRI of the brain 24-hour urine Aldosterone-renin ratio

A 43-year-old Caucasian woman, previously in good health, presents to the emergency department with headache, blurred vision, and dizziness. Symptoms started 3 days ago and progressively worsened. Past medical history: hypertension, hypothyroidism, prior cholecystectomy. No known drug allergies. Medications: HCTZ 25 mg daily, Diltiazem CD 120 mg daily, and Levothyroxine 88 mcg daily. She ran out of all medications 2 weeks ago. Vital signs were normal, except for blood pressure 210/114 in the right arm, 215/115 left arm, 220/100 right leg, and 215/112 left leg. Physical exam: Heart - no visible or palpable PMI; normal S1 and S2 without murmur, rub, or gallop. Pulmonary - few faint RLL crackles, which cleared upon coughing. Remainder of the physical exam, including neurologic exam, was unremarkable. CBC and BMP were unremarkable except for K+ 2.3 mEq/L. EKG - NSR with one PVC. Chest X-ray - clear lung fields; normal pulmonary vasculature. CT head - no evidence of intracranial pathology. Renal artery sonogram - Unremarkable. Patient was treated with IV Nitroprusside and IV KCl 40 mEq x 2 doses and was admitted for further treatment. Over the next 2 days, patient's blood pressures gradually normalized with medical therapy; potassium levels remained low despite treatment. What is the best screening test for the suspected diagnosis?

Family members with the same condition Male predominance More common in children than adults More common in older adults than young/middle age Presence of a psychiatric comorbidity (PTSD, depression) Presence of a psychiatric comorbidity (PTSD, depression)

A 43-year-old female patient presents for possible seizures. She was arguing with their supervisor at work when the patient slumped back in their chair, became pale, and had a 5- to 10-second spell of generalized tonic-clonic jerking. Electroencephalogram reveals no evidence of seizure activity, and the physician suspects psychogenic non-epileptic seizures. From an epidemiological perspective, what factor is most often associated with this condition?

Papillary

A 43-year-old woman is found to have a palpable thyroid nodule that is 1.5 cm in size and located in the right lobe without regional lymphadenopathy. Upon questioning, the patient denies noticing this or any increase in the size of her thyroid. She denies hoarseness, a personal or family history of thyroid disease, and thyroid cancer. Which form of thyroid cancer is the most likely risk to this patient?

Request anti-Ro/SSA and anti-La/SSB serological tests. Prescribe a topical antibiotic cream. Request her immunization records. Determine the medication history of the patient. Instruct patients about sun-avoidance techniques Determine the medication history of the patient

A 43-year-old woman presents with reddish-pink eruptions on her neck, upper back, shoulders, and arms. The skin lesions appear annular and do not hurt or itch. The patient is concerned about having a contagious disease, but she does not feel sick. Her body temperature is 38.4°C, her pulse is 65, and her blood pressure is 150/70. She has a history of hypertension and gastritis, which she has been treating for several years. She exercises regularly, mostly outdoors, and uses sunscreen whenever she feels it is needed. What is the most appropriate next step in the management of this patient?

Electromyogram Computerized tomogram (CT) of the wrist Magnetic resonance image (MRI) of the wrist Plain radiographs of the wrist Erythrocyte sedimentation rate (ESR) Electromyogram (electricity- carpul tunnel)

A 43-year-old woman who works as a secretary and spends most of her day typing on the computer has symptoms of carpal tunnel syndrome. What test is most useful in confirming the most likely diagnosis?

Mixed anion gap metabolic acidosis and metabolic alkalosis

A 44-year-old female patient with insulin-dependent diabetes presents with a 5-day history of weakness, polyuria, polydipsia, and frequent vomiting. A physical exam reveals she is pale, ill-appearing, and in moderate distress. BP 85/55 mm Hg, P 130/min, RR 18/min, T 37.3°C. She has dry mucous membranes and diffuse abdominal tenderness. Room air blood gas shows: pH 7.01, PCO2 19 mm Hg, PO2 85 mm Hg. Glucose 590 mg/dL, Na 131 mEq/L, Cl 80 meq/L, HCO3 10 mEq/L, BUN 29 mg/dL, serum creatinine 1.7 mg/dL. What is this patient's acid-base status?

Perform primary HPV testing every 5 years

A 44-year-old female presents for an annual well-woman examination. She is not sexually active, and the last cervical smear done 4 years ago was unremarkable. History is notable for a supracervical hysterectomy 3 years ago for endometriosis. She has been vaccinated against the human papillomavirus (HPV). What is the preferred strategy for screening for cervical cancer in this patient?

Inhibition of acetylcholine release Blockage of the sodium channels Demyelination Subacute combined degeneration of the spinal cord Antibodies to the acetylcholine receptor Antibodies to the acetylcholine receptor (myasthenia gravis) Tensilion test = myasthenia gravis

A 44-year-old man starts to notice that his eyelids are drooping. Some time afterward, his jaw becomes weak. He has difficulty swallowing and experiences weakness in his limbs. He is quite embarrassed when he eats because he must use his hand to help support his jaw. His weakness gets progressively worse. Finally, he seeks medical attention. His physical examination demonstrates the weakness in his limbs, but no sensory defects are present. A Tensilon test is done and is positive. What is the most likely underlying pathology of these symptoms?

Hyperthyroidism Hypothyroidism Type 1 diabetes Type 2 diabetes Cushing syndrome Type 2 diabetes

A 44-year-old obese woman presents with increased nighttime urination. She has never had issues with having an increased urge to urinate nocturnally before, and it is extremely bothersome to her. She states that she has been waking up at least 3 times a night despite lifestyle modifications designed to help reduce this number. The patient admits to increased fatigue, worsening blurry vision, and two vaginal yeast infections in a span of 3 months. What is the most likely diagnosis?

Acute cholecystitis Chronic cholecystitis Appendicitis Acute cholecystitis

A 44-year-old premenopausal Caucasian woman with a BMI of 36 presents with persistent upper right quadrant abdominal pain that radiates to the back. It has gotten so bad that she has difficulty eating any food and needs to force herself to eat. She has nausea with some episodes of vomiting. She denies bulimia but admits to anorexia. On exam, the patient has a positive Murphy's sign and tenderness to palpation in the epigastric and upper right quadrant area. Patient has a slightly elevated temperature. The physician assistant is awaiting labs and imaging. What is the most likely diagnosis?

Bladder infection Menopause Multiple sclerosis Pelvic inflammatory disease Stress urinary incontinence Stress urinary incontinence

A 44-year-old woman G5P5 presents for her annual pelvic examination. Her menses are regular, and she is currently mid-cycle. She notes "leaking urine" when she coughs, sneezes, or strains. There is a bulge into the anterior vaginal wall; it is exacerbated when the patient is asked to "bear down." A urinalysis is unremarkable. There is no cervical motion tenderness or discharge noted on pelvic examination. The patient is afebrile and in no distress. No lesions are noted on the external genitalia and the pelvic examination is unremarkable except for the noted bulge. Vaginal cultures for gonococcus (GC) and chlamydia are pending. What is the most likely diagnosis?

Aspirin intake Chronic alcoholism Insulin-producing tumor Deliberate insulin intake Haloperidol therapy Insulin-producing tumor - Think about the headache and the levels

A 44-year-old woman is referred to a hospital because of occasional headache accompanied by irritability, confusion, sweating, and hunger. On one occasion, during the crisis period, a decreased blood glucose level (55 mg/dL) was detected despite the fact that she consumes a diet rich in glucose. During echosonographic examination of abdomen, no abnormalities were noted. Fasting test was attempted, but it had to be discontinued due to a fall in blood glucose level from baseline value of 75-33 mg/dL 8 hours later. It required intravenous glucose administration and termination of the test. Laboratory analyses taken at the time of test termination also revealed elevated serum insulin (9 uU/mL), elevated serum proinsulin (6.3 pmol/L), and elevated C peptide (0.3 mmol/mL) levels. Insulin receptor antibodies are not present in the patient's serum, and sulfonylurea is absent in the patient's sera and urine. What is the most likely cause of this patient's symptoms?

Nephrotic syndrome

A 44-year-old woman with diabetes presents with a 2-week history of lower extremity edema. She has no other symptoms. Physical examination of her lower extremities reveals bilateral 2+ pitting edema. What is the diagnosis? there is a major chart with quesiton Protien 4+ oval fat bodies pitting edema

Cervical muscle tenderness Temporal artery tenderness Visual scintillations and scotomas Lacrimation and conjunctival injection Papilledema Lacrimation and conjunctival injection

A 45-year-old African American man with no significant past medical history presents with a 1-hour history of left retro-orbital headache. The headache was of a sudden onset, and it began upon waking this morning. It is described as excruciating, stabbing, sharp, and lancinating; it is rated as severe in intensity. He denies any preceding infections, nausea, vomiting, fever, chills, focal weakness, numbness, tingling, hearing, gait, or speech changes. He recalls a similar episode several months ago; it lasted about 3 hours and dissipated without complications. His physical exam is remarkable for painful distress, nasal congestion with rhinorrhea, left ocular miosis, and left forehead flushing diaphoresis. What is an additional expected manifestation in this patient?

Bone

A 45-year-old Caucasian man was employed to demolish several painted structural steel railroad trestles. The project was scheduled to take 2 years. He was not given protective masks or clothing. Towards the end of the project, he started to develop abdominal pain, so he sees his family doctor. On questioning during the medical history, he comments that he frequently drops things with his right hand. On physical examination, his lungs are clear to auscultation and no abnormal breath sounds are heard. His cardiac exam is unremarkable. His skin exam is normal other than a pigmentation at the gingivodental margin. His extensor muscles of the wrist and finger are weak. A CBC is done. ***Large Chart*** Where is the toxin stored?

Nulliparity

A 45-year-old Caucasian woman presents with a 2-month history of a lump in her right breast. She has had no children. Her menstrual periods began at age 15 and are still regular. She is a successful businessperson who does not smoke cigarettes and drinks 6 cups of coffee per day. She gets 30 minutes of physical activity at least 5 times weekly. On examination, she has a nontender lump in the upper outer quadrant of her right breast. Several matted right axillary lymph nodes are also palpable. Given her history, what is her greatest risk factor for developing breast cancer?

Hiatal hernia Esophageal pulsion diverticulum Barrett's esophagus Esophageal squamous cell carcinoma Esophageal laceration Esophageal laceration

A 45-year-old chronic alcoholic man presents with history of massive hematemesis. This hematemesis followed a bout of prolonged vomiting. Patient has been a known alcoholic for 20 years. On examination, he has a pulse rate of 100/min and a BP of 90/70 mm Hg with cold extremities. These findings are typical for what condition?

Cluster headache Migraine headache Post-traumatic headache Tension headache Giant cell arteritis headache Tension headache

A 45-year-old female patient presents with a 2-week history of almost daily headaches. She describes feeling as if a rubber band is around her temples, making it extremely hard to concentrate on completing tasks. She denies auras, nausea, vomiting, or specific neurological issues, but she does describe some sensitivity to excessive noise and glaring lights. She has not been able to identify any obvious triggers. What is the most likely diagnosis?

Check FSH. Check prolactin level. Check TSH. Order mammogram. Perform microscopy of fluid Order mammogram

A 45-year-old female patient presents with a 3-day history of persistent nipple discharge from the left breast. Menstrual history reveals that periods have spaced out over the last year; they occur 28-53 days apart, and they last 2-3 days. She reports weight gain of 15 lb over the last few months. She denies hot flashes and the possibility of pregnancy; her spouse had a vasectomy. She is otherwise healthy with an insignificant past medical history; there is no history of recreational drugs, medications, or herbs except for vitamin E that she takes every day. Breast exam is negative for lumps, dimpling, and nipple retraction. Clear, serous discharge is seen coming from the left nipple. Axillary exam is negative for any enlarged lymph nodes; the rest of the physical exam is unremarkable. What is the most appropriate next step in the management of this patient?

Cluster headache

A 45-year-old man goes to a party and enjoys several glasses of an alcoholic cocktail. His past medical history is significant for headaches. The drinks trigger a unilateral right-sided headache. The headache is behind his right eye and spreads to his forehead. He also notices that his right nostril has a watery discharge and his right eye is tearing. He describes the pain as if he were "being stabbed in my eye." What is the most likely diagnosis for this patient?

Oral ferrous sulfate

A 45-year-old man is evaluated for a 6-month history of palpitations, easy fatigability, and chest pain on exertion. He does not smoke or consume alcohol; he has no significant past medical history. His body weight has remained stable. He has spent the last 2 years traveling the world. His wife says that he eats "unhealthily." He is a strict vegan, consumes a lot of snacks, and has never taken any vitamin supplements. Examination shows a 5'7" male with a BMI of 19. His BP is 130/70 and pulse 90/min; his temperature is 98.4°F. Conjunctival pallor is present. Auscultation shows a grade 2/6 murmur ejection systolic murmur heard all over the precordium. Lab investigations are sent and are given below: Hb: 8.8 g/dLWBC: 6000/mm3MCV: 72Ferritin: lowTIBC: elevatedStool occult blood: negativeESR: 8 mm/hourTSH: normal What is the most appropriate medical therapy?

Gastroesophageal reflux Pneumonia Aortic dissection Pulmonary embolism Unstable angina Gastroesophageal reflux

A 45-year-old man presents with a 30-minute history of substernal chest pain. He describes the pain as burning. He denies any trauma to the chest. He has had similar episodes like this many times. He denies any additional symptoms such as shortness of breath or diaphoresis, but he states that his voice is often hoarse. His medical issues include diabetes mellitus and heavy alcohol use. What is the most likely cause of his chest pain?

Huntington's disease

A 45-year-old man presents with concerns of uncontrollable movements that he has noticed for the past 2 months. He feels he cannot control these involuntary movements of his upper body. His wife reports that he appears irritable and impulsive. She feels that his personality has changed, but she is more concerned about the sudden jerking in his body. She shared that the patient's father passed away in his 50s with similar symptoms. Upon physical exam, the patient appears to have tics that are sudden and appear depressed. Additionally, Hoffmann's sign and Babinski's sign are positive. A CT scan shows cerebral atrophy and genetic testing from the lab is pending. What is the most likely diagnosis?

Calcium

A 45-year-old woman presents as a new patient. She was recently seen in the emergency department for right flank pain, and a CT scan revealed a right-sided ureteral stone. The stone was 4 mm, a passable size, and she was sent home with analgesics and advised to hydrate well and strain her urine. She was straining her urine and noticed a small dark fleck. She brought the sediment to the urologist's office to undergo a stone analysis, as this is her first episode of a renal or ureteral stone. What is the most likely composition of this stone?

Reassure and send home with oral rehydration

A 45-year-old woman presents with diarrhea and vomiting that started last evening. She says she warmed up leftover rice for supper last night and symptoms began shortly thereafter. She has no fever, and her blood pressure and pulse are within normal limits. What would be the best next step?

FSH level is consistent with menopause

A 46-year-old female patient has not had her period for almost 1 year. She has had hot flashes and feels fatigued most of the time. She reports insomnia and states that she smokes despite trying to quit several times. Libido is low, and she does not use any hormone-based birth control. TSH 3.1 mcU/mL, FSH 55.3 mIU/mL. What is the most accurate description of this patient's menstrual status?

Small bowel obstruction

A 46-year-old woman presents with nausea, vomiting, crampy abdominal pain, and loud bowel sounds for the past several hours. She denies weight loss. She has had one normal bowel movement since the symptoms began, but it did not help her symptoms. She has a past surgical history of an abdominal hysterectomy 7 years prior. On physical exam, she is afebrile, with hyperactive and high-pitched bowel sounds localized to the left upper quadrant. She also has mild diffuse abdominal tenderness. What is the most likely diagnosis?

Antibiotic Immobilization Oxycodone Ambulation Operative treatment Immobilization (medial malleolar fracture)

A 47-year-old female patient with a history of obesity presents to the emergency department due to pain in the right ankle after a fall 2 days ago. On physical exam, there is pain and tenderness at the tip of the medial malleolus and an inability to bear weight for at least 4 steps. The ankle is swollen but not erythematous. The foot appears to be neurologically intact. X-ray reveals an isolated medial malleolar fracture. What is the most appropriate next step in management?

Amylase 310 U/L and lipase 760 U/L Amylase 50 U/L and lipase 10 U/L Aspartate aminotransferase 32 U/L and alanine aminotransferase 29 U/L Aspartate aminotransferase 120 U/L and alanine aminotransferase 40 U/L Amylase 310 U/L and lipase 760 U/L

A 47-year-old man presents to an urgent care center with 18 hours of abdominal pain, nausea, vomiting, and chills. He is a single construction worker, denies smoking, and has at least a 10-year history of drinking 2-4 alcoholic beverages daily. A series of lab work is performed on the patient to evaluate his abdominal pain prior to abdominal imaging. What laboratory results would be most indicative of the patient suffering from acute pancreatitis?

Portal hypertension

A 47-year-old man presents with abdominal pain and difficulties breathing. He has a history of alcohol abuse and confirmed cirrhosis of the liver. On examination, you see a malnourished and jaundiced patient with a distended belly. Percussion of the abdomen reveals a huge amount of fluid and wave sign. What is the primary cause of the ascites?

Sensory loss in anterolateral thigh down to left upper knee Sensory loss on the dorsum of the foot Foot drop Loss of foot inversion in plantar flexion Loss of left quadriceps reflex Sensory loss in anterolateral thigh down to left upper knee (compartment syndrome spread up)

A 47-year-old man presents with numbness, tingling, and pain anterior and to the left side of his left thigh. Pain is provoked even with light touch, and tingling continues for several minutes after the touch. Symptoms started a couple of months ago and worsen when he wears a belt and walks down slopes and stairs, as well as after prolonged standing. Symptoms are relieved when he puts a pillow between his thighs. Patient also assumes hunched posture while standing to avoid unpleasant sensory symptoms. His BMI is 40, and he has a protruding pendulous abdomen. What else do you expect to find on examination?

Plain radiographs CT EMG Ultrasound MRI MRI

A 47-year-old obese woman presents with left non-dominant shoulder pain and limited motion. The pain began about a month ago but has reduced over time. Her shoulder has progressively lost motion during that time; she cannot reach overhead with that arm. There is no history of trauma, it does not wake her at night, and she can sleep on the left side. Her past medical history is significant for type 1 diabetes and hypothyroidism. She is on regular insulin, NPH insulin, and levothyroxine. What diagnostic study would best help evaluate the suspected diagnosis?

Decreased active range of motion, full passive range of motion Decreased active and passive range of motion ≥50% Minimal pain with motion, especially at extremes Weakness with isometric rotator cuff muscle testing Tenderness of the sternoclavicular join Decreased active and passive range of motion ≥50%

A 47-year-old obese woman presents with left non-dominant shoulder pain and limited motion. The pain began about a month ago but has reduced over time. Her shoulder has progressively lost motion during that time; she cannot reach overhead with that arm. There is no history of trauma, it does not wake her at night, and she can sleep on the left side. Her past medical history is significant for type 1 diabetes and hypothyroidism. She is on regular insulin, NPH insulin, and levothyroxine. What finding do you expect on physical exam?

Nasogastric feeding

A 48-year-old Caucasian man presents with severe epigastric pain radiating to the back after a bout of drinking. He seeks medical attention and receives treatment. Symptoms improve within 5 days. 4 weeks later, his symptoms return with epigastric pain, weight loss, and decreased appetite. A repeat ultrasound shows a round thin-walled hypoechoic lesion near the pancreas tail measuring 4 cm in its largest diameter and with some calcifications in its walls. MRCP visualized a communication between this cavity and the pancreatic duct, which confirms the suspected diagnosis. What is the most appropriate next step in management?

Offer blood tests and provide education/prescription for hormone therapy. Offer blood tests and provide education/prescription for raloxifene (Evista). Order cancer antigen 125 (CA-125). Order pelvic ultrasound. Order thyroid ultrasound Offer blood tests and provide education/prescription for hormone therapy

A 48-year-old Caucasian woman presents due to feeling like she is losing her mind. She wants some tests done. Upon further questioning, she reports she is having multiple episodes daily in which she suddenly becomes very hot, flushed, and diaphoretic. These episodes last about 1 minute before resolving. She has not measured a fever. The patient reports that these episodes occur during the day and at night, during which she awakes drenched with sweat. As a result, her sleep has been poor, and she feels fatigued and irritable at both work and home. She has noticed these symptoms for about the last 2 months, and they seem to be increasing in severity. The patient has not had a period for 3 months; she recently did a home pregnancy test, which was negative. Prior to that, she had regular menses. This patient denies weight changes, palpitations, cold intolerance, bowel changes, as well as changes in her nails, skin, and hair. Although she admits irritability, she denies anxiety, depressed mood, and suicidal ideation. Her family history is remarkable for diabetes in her maternal grandfather and hypertension in her father. She is a G4P3Ab1. She denies any major psychosocial stressors recently. She drinks alcohol rarely, and she denies use of other drugs. Vitals and a urine specimen for hCG were obtained prior to the physical exam. Complete screening physical exam is normal, with normal sexual development and absence of hirsutism and acne. Based on this patient's history and physical, what is the most reasonable management for this patient?

Increase dietary sodium. Initiate an antidepressant with anti-anxiety properties. Initiate steroids. Refer to oncologist. Restrict dietary potassium intake Initiate steroids

A 48-year-old man presents with fatigue, weakness, and nausea—symptoms progressively worsening over 6-8 months. He reports fatigue despite adequate sleep, overall feeling of muscle weakness, nausea with occasional vomiting, 12 lb weight loss, headaches, and muscle aches. His wife thinks he appears tan year-round, despite lack of sun exposure. He admits feeling anxious and irritable, but he denies any major psychosocial or traumatic events. Before symptom onset, he was healthy and active. Past medical history reveals no chronic medical conditions, medications, surgeries, or allergies. Family history is significant for thyroid disease in sister and mother, type 1 diabetes mellitus brother. He denies tobacco, alcohol, and recreational drugs. On physical exam, he is hypotensive and hyperpigmented. Remainder of his physical exam is normal. UrinalysisNormalComplete blood countMildly decreased hemoglobin and hematocritComprehensive metabolic panelMildly decreased sodium and elevated potassium, rest normalAdrenocorticotropic hormone (ACTH)ElevatedCortisol (morning level)DecreasedACTH stimulation testDecreased cortisol What is the most important intervention for this patient's most likely diagnosis?

Decreased free thyroxine (free T4) Decreased luteinizing hormone (LH) Increased estradiol Increased follicle stimulating hormone (FSH) Increased testosterone Suspected Cause Increased follicle stimulating hormone (FSH) -menopause

A 48-year-old Caucasian woman presents due to feeling like she is losing her mind. She wants some tests done. Upon further questioning, she reports she is having multiple episodes daily in which she suddenly becomes very hot, flushed, and diaphoretic. These episodes last about 1 minute, then resolve. She has not measured a fever. The patient reports that the episodes occur during the day and at night, causing her to wake up drenched in sweat. As a result, her sleep has been poor, and she feels fatigued and irritable at both work and home. She has noticed these symptoms for about 2 months, and they seem to be increasing in severity. The patient has not had a period for 3 months; she recently did a home pregnancy test, which was negative. Prior to that, she had regular menses. This patient denies weight changes, palpitations, cold intolerance, bowel changes, as well as changes in her nails, skin, and hair. Although she admits irritability, she denies anxiety, depressed mood, and suicidal ideation. Her family history is remarkable for diabetes in her maternal grandfather and hypertension in her father. She is a G4P3Ab1. She denies any major psychosocial stressors recently. She drinks alcohol rarely, and she denies use of other drugs. Vitals and a urine specimen for hCG were obtained prior to the physical exam. Complete screening physical exam is normal, with normal sexual development and absence of hirsutism and acne. What diagnostic study result is most consistent with the suspected diagnosis?

Estradiol/norethindrone acetate (Activella) 1 mg/0.5 mg by mouth daily

A 48-year-old Caucasian woman presents due to feeling like she is losing her mind. She wants some tests done. Upon further questioning, she reports she is having multiple episodes daily in which she suddenly becomes very hot, flushed, and diaphoretic. These episodes last about 1 minute, then resolve. She has not measured a fever. The patient reports that these episodes occur during the day and at night, the latter causing her to awaken drenched in sweat. As a result, her sleep has been poor, and she feels fatigued and irritable both at work and at home. She has noticed these symptoms for about the last 2 months, and they seem to be increasing in severity. The patient has not had a period for 3 months; she recently did a home pregnancy test, which was negative. Prior to that, she had regular menses. This patient denies weight changes, palpitations, cold intolerance, bowel changes, as well as changes in her nails, skin, and hair. Although she admits irritability, she denies anxiety, depressed mood, and suicidal ideation. Her past medical history is significant for seasonal allergies, which are relieved with over-the-counter antihistamines and taken as needed. Her surgical history includes a tonsillectomy and bilateral tubal ligation. Her family history is remarkable for diabetes in her maternal grandfather and hypertension in her father. She is a G4P3Ab1. Social history reveals the patient is an office manager for a dental clinic; she is a married nonsmoker with 3 children living at home. She denies any major psychosocial stressors recently. She drinks alcohol rarely, and she denies use of other drugs. After discussing the likely diagnosis with you, the patient decides she does not want any tests now and wants to start treatment immediately. What medicine should be initiated?

Abciximab Heparin Coumadin Eptifibatide Tirofiban Heparin

A 48-year-old Caucasian woman with a past medical history of hypertension and hypercholesterolemia was diagnosed recently with a cerebral aneurysm. The treatment plan for the aneurysm is endovascular coiling. Among other complications, this patient has an increased risk of thromboembolism postoperatively. What is the initial preventive medication that will be used to help minimize risk in this patient?

Avoidance of vitamin C Low-fat, low-cholesterol diet Moderate fatty or greasy meals Prolonged fasting Rapid weight loss Low-fat, low-cholesterol diet

A 48-year-old female patient with a past medical history of obesity presents with 2 months intermittent mild epigastric and right upper quadrant pain. The pain is intermittent and occurs in "waves." She notes nausea, some vomiting, and radiation of pain to the right shoulder. Physical exam reveals unremarkable vital signs, but tenderness is noted in the right upper quadrant. There is no guarding or rebound. Bedside ultrasonography is obtained. Refer to the image. What health maintenance advice is recommended for this patient?

Kernig's sign Babinski sign Kussmaul's sign Quincke's sign Chvostek sign Kernigs Sign To elicit Kernig's sign, the patient is placed in the supine position with knees bent Cryptococcus neoformans is an opportunistic yeast infection found in immunocompromised patients, which includes HIV-positive patients

A 48-year-old male patient positive for HIV starts to develop headaches. At first, he attributes the headaches to stress, but they persist and become worse over the next few weeks. He develops nausea and vomiting, and he thinks he has a fever. He starts to become confused, so he seeks medical attention. On physical examination, temperature is 100°F. What special maneuver of the neurological exam should be performed to support your diagnosis?

Ultrasound - (sounds like a hernia)

A 48-year-old man presents with a 2-day history of left-sided groin and scrotal pain. He has had similar pain episodically for several months, but it has recently become much worse after a weekend helping his brother move furniture. He admits that he is not in good physical shape, and he thinks he may have pulled a groin muscle. He is in a monogamous relationship with his wife of 17 years. He has never had any testicular or scrotal conditions, and he has a negative surgical history. He denies fever and urinary symptoms. He has no allergies and takes no other medications. On physical exam, the patient has normal sexual development with no edema, warmth, or erythema present in the scrotum. No skin lesions are present. On palpation, there is mild tenderness on the left scrotum. With the Valsalva maneuver, however, a small bulge is palpable in the left scrotum and the patient's reported pain level increases. When he lies supine, the bulge is no longer palpable. Based on this patient's history and physical what would be the most helpful test in establishing the suspected diagnosis?

Topiramate Gabapentin Alprazolam Propranolol Phenobarbital Propranolol (beta blocker for essential tremors)

A 48-year-old man presents with a 5-year history of hand tremor. The tremor was initially mild and has progressed over the last year. It diminishes at rest, but it intensifies with intentional movements and upon emotional stress and fatigue. He is unable to write or drink from a cup during stressful situations. He never drinks alcohol. Family history is negative. Physical exam is unremarkable. His gait, speech, and posture are normal. What is the most appropriate first-line agent for treatment of this condition? Answer Choices

Exercise-associated muscle cramps Neuroleptic malignant syndrome Non-exertional hyperthermia Malignant hyperthermia Acute heat exhaustion Non-exertional hyperthermia (heat stroke)

A 48-year-old woman presents after a seizure. Before the seizure, she experienced confusion, disorientation, and poor coordination preceded by nausea, dizziness, flushing, and weakness. Symptoms appeared after bird watching several hours in her garden under the sun. Her medical history is significant for the presence of schizophrenia, for which she takes chlorpromazine at bedtime. Her temperature is 41°C, blood pressure 90/59, heart rate 110, respiratory rate 25; BUN, creatinine, and transaminases are elevated, and there is leukocytosis and lactic acidosis. There is normal urine myoglobin, PT, and PTT. What is the most likely diagnosis?

Total hip replacement Vascularized bone grafting Daily oral bisphosphonates Daily oral corticosteroids Non-vascularized bone grafting Total hip replacement

A 49-year-old Caucasian man presents with pain in his left lower extremity. During questioning, the patient states that while doing construction work 3 months ago, he jumped from an elevated height of 6 feet. Ever since this episode, he has noted increased issues with left-sided hip and knee pain. He describes the pain as radiating into the left groin and front middle thigh area. The pain is relieved with sitting and aggravated by walking and climbing up stairs. The patient denies any paresthesias, numbness, bowel or bladder dysfunction, fever, night sweats, or chills. Pertinent medical history includes a 20-year extensive history of alcohol. A radiograph interpretation shows the presence of a crescent sign and marked irregularity of the left femoral head with sclerosis. Considering the most likely diagnosis, what would be the ultimate clinical intervention necessary for this patient?

Wrist arthritis Scaphoid fracture Dorsal wrist ganglion Thumb arthritis de Quervain's tenosynovitis de Quervain's tenosynovitis (markedly exacerbated when she places her thumb into the palm of her hand, and when you passively move her hand in an ulnar direction)

A 49-year-old right-hand dominant woman presents with a 2-week history of progressive pain in her right thumb and wrist area. She says that her thumb seems to "stick" in place upon movement. She states that she has never experienced this before. You ask her if she has been using her hands more often than normal, and she tells you she recently began to crochet a sweater for a family member. On physical examination, her pain is markedly exacerbated when she places her thumb into the palm of her hand, and when you passively move her hand in an ulnar direction. Question What is the most likely diagnosis?

Dysthymic disorder Major depressive disorder Normal grief reaction Obsessive-compulsive disorder Schizophrenic disorder Normal grief reaction (no harm to self, there is only 5 symptoms and there is a lose to a love one)

A 49-year-old woman presents for a consultation 1 month after her 22-year-old son was killed in a fall at a construction site near her home. The patient is upset, restless, and reports feeling lonely. She lies awake at night. She does not feel like eating. She cries easily when she looks at their family pictures around the house. She wishes she could talk to him again, and she reports she sometimes thinks she sees him walking just outside their home. She says she wishes she would have died instead of him, but she denies any thoughts or plan of harming herself. What is the most likely diagnosis?

Basal cell carcinoma Squamous cell carcinoma Malignant melanoma Kaposi sarcoma Paget's disease Paget's disease

A 49-year-old woman presents with a 1-month history of a rash on her right breast. She reports that it has not been responding to the creams that she has been applying and she is having some burning and itching at the rash. On examination, she has an erythematous area that is sharply demarcated with scaling and vesicles over her right areola and nipple. There is a nontender lump in the upper outer quadrant. Her left breast is normal. What is the most likely diagnosis?

Achalasia

A 49-year-old woman presents with dysphagia. She is having difficulty swallowing liquids and solids, and she notes regurgitation of undigested food. X-ray reveals a bird's beak appearance of the esophagus. What is the most likely diagnosis?

Hyperkalemia Hyperglycemia Hypokalemia Low urine specific gravity Hypernatremia Hypokalemia (votming)

A 5-month-old girl presents with a 3-day history of vomiting. She is exclusively breastfed, and her mother states that today she has vomited within 15 minutes of each feeding. Her last wet diaper was 10 hours ago. On physical examination, she is afebrile, tachycardic, and irritable, and she does not express tears when crying. She was a full-term vaginal delivery. She has no significant past medical history. Her 3-year-old sister has had gastroenteritis for the past few days. Based on the most likely diagnosis, what is the most likely laboratory finding?

Ultrasound of the head Lumbar puncture Initiation of empiric antibiotics MRI of the head CT of the spine Lumbar puncture

A 5-month-old male infant is brought to the emergency department after a possible seizure episode. His mother states he was born full-term without any complications, and he was well until 2 days ago when he developed a fever. He vomited multiple times yesterday and was irritable. He has not had diarrhea or a cough. He was given antipyretic medication for his fever. He has no known allergies. His immunizations are current. His developmental milestones have been in accordance with his age. On physical exam, temperature is 102.7°F, pulse is 154/min, BP is 90/50 mm Hg, RR is 20/min. He is lethargic and pale; there are no focal neurological deficits or active seizure activity. Non-contrast CT of the head is unremarkable. After drawing blood samples for investigations, what is the most appropriate next step?

Ceftriaxone and vancomycin Gentamicin and vancomycin Nafcillin and a fluoroquinolone Nafcillin and vancomycin Vancomycin and a fluoroquinolone Ceftriaxone and vancomycin (osteomyelitis)

A 5-year-old African American girl with sickle cell disease presents due to right leg pain. She began to experience right thigh pain and a slight limp following a playground injury 2 weeks ago. Upon sustaining the injury, she was taken to a local ER; X-rays were negative for evidence fracture. She was diagnosed with and treated for a leg contusion. Over the past 2 weeks, the pain has become more severe and she has experienced fever; temperatures have been as high as 102°F, and she has experienced episodic chills. On physical examination, her temperature is 101°F; her right leg is swollen, tender, and erythematous over the anterior aspect of the thigh. The remainder of the PE is unremarkable. CBC with differential reveals a leukocytosis; there is a predominance of neutrophils and a bandemia. What is the most appropriate empiric treatment?

Attention deficit hyperactivity disorder

A 5-year-old boy presents because of concerns about his behavior. His mother notes he gets extremely distracted and can only focus for 1-2 minutes at a time. He cannot seem to sit still, and he displays extremely impulsive behavior at inappropriate times. She describes impulsive behavior recently at a funeral and at an older sibling's music recital. Because the mother works full time, the patient attends daycare after kindergarten. His kindergarten teacher and the daycare staff have expressed similar concerns. What is the most likely diagnosis?

Shigella sonnei

A 5-year-old boy presents with a 4-day history of bloody diarrhea. He has had fever up to 104°F, abdominal pain, and painful defecation. His past medical history is unremarkable, and he has had no surgeries. He is on no medications and has no drug allergies. He attends a local daycare with nine other children. On physical examination, his abdomen is tender with hyperactive bowel sounds. While in the emergency department, he has a 5-minute generalized seizure. What pathogen is the most likely cause of the patient's diarrhea and seizure?

Acute lymphocytic leukemia

A 5-year-old boy presents with his mother with a history of fever, hemorrhages, and repeated bacterial infections. On exam, lymphadenopathy and hepatosplenomegaly are present. Blood work shows a white blood cell count of 50,000/µL. What is the most likely diagnosis?

Kawasaki disease Systemic onset juvenile rheumatoid arthritis Systemic lupus erythematosus Henoch-Schönlein purpura Polyarteritis nodosa Henoch-Schönlein purpura

A 5-year-old boy presents with history of low-grade fever, headache, and intermittent colicky abdominal pain localized mainly around the umbilicus since yesterday. He has vomited once. His symptoms are also accompanied by a purplish-red maculopapular rash more confluent over the lower extremities and the buttocks. There is no itching. Both knees and ankles are swollen and tender, and there is edema of the hands and feet mainly in the dependent areas. Examination of cardiovascular, respiratory, and abdominal are essentially normal. Laboratory investigations: IgA high 500 Urine Blood and protien Stool: blood What is the most likely diagnosis?

Acute lymphocytic leukemia

A 5-year-old girl is brought to her pediatrician by her mother. Her mother notes that the patient has been bruising easily for the last few weeks, but she does not recall any major injury. The patient began feeling tired around this time as well. The mother initially thought she had the flu, but she brought the patient in when it became apparent she was not improving. On examination, the patient is pale and appears fatigued. Her skin has multiple areas of bruising and petechiae. She is febrile. A CBC/Diff is significant for the following: TestResultWBC72.2 x 103 mm3Hgb8.4 gm/dLHct22.30%Platelets30 x 103/µLNeutrophils5%Lymphocytes0%Monocytes0%Basophils0%Eosinophils0%Blasts95% Bone marrow biopsy is obtained and confirms presence of blasts. Stains for myeloperoxidase and Sudan Black are negative. What is the most likely diagnosis?

Cubitus varus Myositis ossificans Median nerve injury Ulnar nerve injury Volkmann contracture Cubitus varus

A 5-year-old girl presents after falling off a shopping cart, tripping, and then falling onto her right arm. On examination, temp is 98.7, pulse 97, respirations 18, blood pressure 127/80 mm Hg. She is alert, oriented, and in no acute distress. Significant findings related to the right arm, which was mildly swollen, deformed, and diffusely tender. There was decreased range of motion of the right elbow due to pain. Sensation was intact. Pulses are within normal limits bilaterally. A radiographic examination was performed. Refer to the image. What is the most common long-term complication associated with the radiology finding?

Hepatitis antibodies Renal angiogram Post-void bladder scan/ultrasound Non-contrast CT abdomen or renal ultrasound Non-contrast CT abdomen or renal ultrasound

A 50-year-old Caucasian man presents to establish care. He has no history of diabetes, hypertension, cardiovascular disease, cerebrovascular disease, peripheral vascular disease, nephrolithiasis, or obstructive uropathy. His only problem on review of systems is musculoskeletal aches. His parents died in their 80s of unknown causes. He has never smoked, taken recreational drugs, had a transfusion, or had unprotected sex. He worked for years as a handyman. His only medication is ibuprofen, which he has taken at several times per week for the past 10 years. His exam is normal. You order some basic diagnostic tests, including a basic serum chemistry panel and a urinalysis. The urinalysis shows microscopic hematuria and a few white cells but no leukocyte esterase or nitrites. His serum creatinine is 1.5 mg/dL. His other tests are normal. What additional test should you order and why?

5% D/W 25% dextrose 0.45% NaCl Normal saline Blood transfusion Normal saline

A 50-year-old Caucasian man with diagnosed peptic ulcer disease (PUD) has nausea and severe vomiting for 2 days. Physical examination shows a patient with confusion, pale skin, tachycardia of 140 bpm, and blood arterial pressure of 90/50 mm Hg. Further evaluation reveals hypochloremic hypokalemic metabolic alkalosis. Urgent management should initially include what IV fluids?

Menopause

A 50-year-old female patient with no significant past medical history presents for an annual pelvic exam. Last menstrual period was over 14 months ago, and the last few menses were extremely irregular. The patient describes a sensation of intense heat in the face and trunk accompanied by sweating. She states that her "heat episodes" have been occurring 1-2 times weekly for several months. She denies any other symptoms. She has received her annual pap and pelvic examination yearly and a clinical breast exam without issue. On pelvic examination, there is obvious vaginal thinning, excessive dryness, and apparent vaginal wall atrophy. What is the most likely diagnosis?

Bronchogenic carcinoma Hepatocellular carcinoma Leukemia Lymphoma Pancreatic carcinoma Hepatocellular carcinoma

A 50-year-old male patient presents with a 3-month history of weakness, fatigue, and abdominal discomfort. He acknowledges a lack of sexual desire. He denies any photosensitivity. On physical examination, the liver is enlarged, and the spleen is palpable. He has abnormal skin pigmentation on the face, neck, and elbows that gives his skin a metallic gray hue. TestResultsReference RangeTIBC275 mcg/dL250-350 mcg/dLPlasma iron220 mcg/dL80-160 mc/dLTransferring saturation90%16-57% What serious complication is associated with this patient's condition?

Lifestyle modifications

A 50-year-old man presents for a follow-up exam. He has a history of hypertension and morbid obesity. His routine labs reveal a hemoglobin A1c of 6.8%. At this point, what is the most appropriate intervention?

Oral creon Oral omeprazole Intravenous normal saline Intravenous meropenem Surgical debridement Intravenous normal saline -acute pancreatitis

A 50-year-old man presents to the emergency department with epigastric pain. Pain is sharp with radiation into the back and accompanied by nausea and vomiting. On exam, the patient exhibits tenderness to palpation of the upper right quadrant and upper left quadrant without rigidity or guarding. Laboratory findings reveal the following: What is the most important component of this patient's treatment?

Neer test Tinel sign Phalen test Bragard stretch test Apley grind test Neer test (shoulder- pitcher)

A 50-year-old man was playing baseball with his company team last weekend and is now experiencing severe pain in his left shoulder. He states that he has been the team pitcher for several years now. He has developed pain progressively in his left shoulder over the last few months. He denies any numbness or tingling in his arm, hand, or fingers of his left upper extremity. Which exam would you expect to yield pain?

Meloxicam Allopurinol Oxycodone Alendronate Risedronate Meloxicam (OA)

A 50-year-old obese woman presents with severe left knee pain. She states the pain began about 8 months ago but has gotten significantly worse in the last 3 months. The patient denies any trauma or event that initiated the pain. She notes stiffness in the knee first thing in the morning; it only lasts around 5-10 minutes. The knee pain worsens with activity and is relieved with rest. The patient's medication list includes lisinopril 10 mg once daily for high blood pressure. She has a documented medication allergy to acetaminophen, which gives her hives. Physical examination reveals a female with a BMI of 40, limited range of motion of the left knee, and crepitus. Considering the most likely diagnosis for this patient, what pharmaceutical regimen would be recommended?

Bed rest Blood pressure control High impact activity Screening labs Weight loss Weight loss

A 50-year-old obese woman presents with severe left knee pain. She states the pain began about 8 months ago but has grown significantly worse in the last 3 months. The patient denies any trauma or event that initiated the pain. She notes stiffness in the knee first thing in the morning that lasts around 5-10 minutes. The knee pain is worsened with activity and is relieved with rest. The patient's medication list includes lisinopril 10 mg once daily for high blood pressure. She has a documented medication allergy to acetaminophen; she states this makes her break out in hives. Physical examination findings reveal a Caucasian female with a BMI of 40. There is limited range of motion of the left knee and severe crepitus. Considering the most likely diagnosis for this patient, what lifestyle modification would be most beneficial in relieving her symptoms?

Metformin

A 50-year-old woman comes in for follow-up of newly diagnosed type 2 diabetes mellitus. She has no other contributory past medical history. She drinks alcohol rarely. She has been working on dietary changes over the last 6 months. Most recent A1c is 7.6%. What medication would be considered first-line in this patient?

Increased serum follicle-stimulating hormone (FSH)

A 50-year-old woman presents for her annual pelvic examination. She states her last menstrual period was over 12 months ago; the last few occurrences of menses were extremely irregular. The patient also describes having the sensation of intense heat in her face and trunk; the sensation is accompanied by sweating. She further states that these "heat episodes" have been occurring 1 or 2 times a week for the last several months. She reports no other issues at this time. She has received her annual pap and pelvic examination yearly, as well as a clinical breast exam, without any issues. During the pelvic examination, you note obvious vaginal thinning and excessive dryness; there is also apparent vaginal wall atrophy. Laboratory findings in this patient would include a decreased serum estradiol and what else?

Direct inguinal hernia Epigastric hernia Femoral hernia Indirect inguinal hernia Spigelian hernia Femoral hernia

A 50-year-old woman presents with "swelling in my right groin" when she stands. On physical examination, you note a reducible bulge that is 3 cm below her right groin crease and is lateral to her pubis. The bulge is on the ventromedial surface of the anterior thigh. What is the most likely diagnosis?

Alzheimer's disease Endometrial hyperplasia Endometriosis Metastasis Osteoporosis Osteoporosis

A 51-year-old Caucasian woman presents to the clinic to follow up on multiple tests she had requested from another practitioner. She has had what she describes as "episodes," in which she feels overheated and diaphoretic. Her coworkers note that her face turns red. These episodes seem to occur multiple times during the day; they have been occurring for the last 2-3 months, and they last about 1-2 minutes before they resolve. She is also very warm when sleeping at night. She denies weight changes, palpitations, headaches, galactorrhea, acne, bowel changes, hair loss, and any changes to her skin or nails. Overall, she reports some mild malaise and irritability, but she denies depression and fatigue. Her past medical history is unremarkable, with no known medical conditions, no allergies, no medications, and no prior surgeries. She denies the use of alcohol, tobacco, and recreational drugs. Her method of contraception is vasectomy in her husband. Her last menstrual period was about 4 months ago, but periods were regular and monthly prior to that. Physical exam and vitals are normal. You plan to educate the patient about her condition and offer treatment for her current symptoms. The patient should be counseled that her current diagnosis puts her at increased for what condition?

Calcium Cholecalciferol Parathyroid hormone Estrogen Progesterone Estrogen

A 51-year-old woman presents due to menstrual irregularity, hot flashes, and mood changes. Physical examination reveals an atrophic vagina and breasts that have decreased in size. She is diagnosed with menopause, and she decides not to start hormone replacement therapy (HRT). 8 years later, a dual-energy X-ray absorptiometry (DEXA) reveals a T score of -3. Change in the serum level of what substance is most likely responsible for her changes in bone mineral density?

Nifedipine Pantoprazole Famotidine Ciprofloxacin Nystatin Nifedipine

A 51-year-old woman presents with difficulty swallowing. She reports a 2-month history of problems swallowing liquids and solids and bringing up undigested food. X-ray reveals a bird's beak appearance of the esophagus. What medication would be most appropriate in this patient?

Vancomycin

A 52-year-old man presents with a 3-day history of persistent diarrhea. He reports seven watery, non-bloody bowel movements daily. He has associated lower-abdominal cramping and mild nausea. He denies recent travel out of the country. He does not recall eating anything unusual, and none of his family members are sick. Past medical history is significant for GERD, for which he takes pantoprazole daily. He recently completed a course of oral levofloxacin for pneumonia. A stool sample is negative for ova and parasites, but PCR testing is positive for Clostridioides difficile. What is the appropriate first-line treatment?

Psyllium Topical nitroglycerin Oral ciprofloxacin Botox injection Surgery Psyllium

A 52-year-old man presents with rectal pain during bowel movements. He describes the pain as tearing. Physical examination reveals a tear in the anal mucosa. What is considered first-line treatment for the underlying cause of this patient's condition?

Hydrochlorothiazide Indomethacin Amiloride Desmopressin acetate Glucophage Desmopressin acetate

A 53-year-old man was recently diagnosed with a pituitary adenoma. He has been experiencing excessive thirst and large volume polyuria, as well as headaches and an increase in nocturia from once nightly to 3-4 times nightly. What medication should be the first-line treatment option given to him?

Chronic pancreatitis

A 52-year-old man presents with vomiting and epigastric distress for the past few hours. He has been drinking alcohol for over 20 years, and he has been a moderate-to-heavy drinker. 5 years ago, he was diagnosed with a "gastric/duodenal ulcer," for which he has been taking cimetidine and antacids. The pain now radiates towards the left along the costal margin. He has noticed his appetite has been reduced lately, and his stools are bulky and foul smelling. His friends have commented on his sickly look and weight loss. What is the most likely diagnosis?

NSAIDs and thumb spica splint Corticosteroid injection into the tendon sheath Surgical release of the tendon Short arm casting for 6 weeks Joint replacement surgery NSAIDs and thumb spica splint

A 52-year-old overweight woman has had pain in her right hand for the past month. She is employed as a pastry chef and has trouble making a fist. On exam, she is tender over the radial styloid. You have her flex her thumb into her palm and move the wrist into ulnar deviation. This movement recreates her pain. What is the initial treatment for this condition?

Menopause

A 52-year-old woman has a 12-month history of amenorrhea. She noticed hot flashes and night sweats, loss of libido, and vaginal dryness. Upon physical exam, no remarkable findings except some dry skin. Upon lab results, elevated serum FSH level over 30 mIU/mL. 24-hour urinary free cortisol is 11 mcg/24 hr, within reference range of 10-100 mcg/24 hr. Serum IGF-1 level is within reference range for a 52-year-old (90-360 ng/mL). ACTH are unremarkable. What is the most likely diagnosis?

Vaginal dryness and itching Worsening mood swings Lowering of serum HDL level Higher risk of vertebral fractures Higher risk of endometrial cancer Higher risk of endometrial cancer

A 52-year-old woman presents because her menopausal symptoms have been extremely distressing. Over the past 4 months, she has experienced severe mood swings, hot flashes, night sweats, breast tenderness, and changes in her appetite. She has never smoked; she has an occasional drink. She had an IUD that was removed at age 35. She had irregular periods in her 40s and menses stopped at 50. There is no family history of cancer. After a prolonged discussion, a decision to start hormone replacement therapy is made. What is a concern with the use of an estrogen-only supplement for this patient?

HRT is indicated for this patient's symptoms. Estrogen replacement alone is indicated. HRT will control hot flashes, but not vaginal dryness. HRT is contraindicated due to regular alcohol use. Antidepressant therapy is indicated for menopause-related mood swings. Antidepressant therapy is indicated for menopause-related mood swing

A 52-year-old woman presents to her gynecologist's office with a 6-month history of hot flashes, night sweats, mood swings, and vaginal dryness that interferes with intercourse. The symptoms seem to be worsening and are now interfering with her productivity at work and with her relationships with family and friends. Her last normal menstrual period (LNMP) was 8 months ago. She denies tobacco use, and she drinks one glass of red wine daily. She has a history of coronary heart disease (CHD) with stent placement 2 years ago. She recently read an article about hormone replacement therapy (HRT). How should this patient be advised?

Metoprolol Metformin Mesalamine Prednisone Ciprofloxacin Prednisone

A 52-year-old woman presents with left hip pain. There is no known history of trauma to the area. Past medical history includes Crohn's disease, type 2 diabetes, and hypertension. She is currently taking metformin, metoprolol, and mesalamine. She recently completed a prednisone taper for her Crohn's disease. She also finished a course of ciprofloxacin, which she took for a urinary tract infection. An X-ray is obtained, revealing a collapsed left femoral head. What medication most likely contributed to her current condition?

PET scan Colonoscopy Chest, abdomen, and pelvis CT Pelvic MRI CEA level Colonoscopy

A 52-year-old woman was noted on yearly examination to have a microcytic anemia. She has recently noted a change in bowel habits and rectal bleeding with bowel movement. She reports abdominal pain. She has no prior surgical history. Her only medical issue is an elevated cholesterol level that is controlled by diet. Her pulse is 92 BPM, blood pressure is 140/78 mm Hg, respiration rate is 14/min, and temperature is 98.7°F. Rectal exam is notable for guaiac positive stool without any masses. Neurological examination is normal. What test should be ordered to confirm the suspected diagnoses?

Cluster headache

A 53-year-old man presents with a 2-week history of severe headaches that occur primarily at night. The patient is pacing while he is talking. The pain surrounds one eye and lasts for 30-90 minutes. He also states that there is ipsilateral lacrimation, conjunctival injection, and nasal congestion during the attacks. The patient states that he has had these headaches once a day over the last week. He cannot point to any one thing that causes them. On examination, the patient has features of partial Horner's syndrome. His vital signs are temperature: 97.0°F, heart rate: 80/min, respiration 16/min, and blood pressure: 126/80 mm Hg. What is the most likely diagnosis?

Use of proton pump inhibitors

A 53-year-old man presents with increased difficulty swallowing and occasional regurgitation of his meals. His symptoms have been occurring with greater frequency and severity over the last 4 months. He also has some shortness of breath but attributes that to his weight and lack of physical activity. His past medical history is remarkable for chronic heartburn, which he treats intermittently with over the counter antacids. He takes no regular medications has no allergies. He has not had any surgeries. He is not a smoker and he denies use of alcohol and drugs. He works as a building inspector, and he lives with his wife and children. The patient is obese, but the rest of his physical exam is normal. Blood tests, electrocardiogram, and chest X-ray are done in the clinic; they are normal. He is referred for endoscopy, and esophageal biopsy shows specialized intestinal metaplastic cells (of columnar epithelium). What recommendation would best prevent complications from this patient's current condition?

Alendronate Dicyclomine Esomeprazole Famotidine Simethicone Esomeprazole

A 53-year-old man presents with increased difficulty swallowing and occasional regurgitation of meals; symptoms have been more frequent and severe over 4 months. Past medical history is remarkable for chronic heartburn, which he treats intermittently with over-the-counter antacids. He takes no regular medications, has no allergies, and has not had any surgeries. He is a smoker, but he denies use of alcohol and recreational drugs. He lives with his wife and children. The patient is obese, but the rest of his physical exam is normal. Laboratory evaluation, electrocardiogram, and chest X-ray are normal. Esophageal biopsy shows specialized intestinal metaplastic cells (of columnar epithelium). What is the medication of choice for this patient's condition?

Repeat pap smear Chlamydia serology Laparoscopy Endometrial biopsy Hormonal profile Endometrial biopsy - vaginal bleeding is cancer until proven

A 53-year-old woman is seen by her gynecologist. She has had three children and had her tubes tied through tubal ligation. She reached menopause at 48. Over the last few weeks, she has noted some vaginal bleeding that occurs unpredictably. She has had regular pap smears that have always been normal. The last one was 11 months ago. Speculum examination and bimanual palpation of the genitalia reveal no abnormalities. What is the next step in the management of this patient?

Hormone replacement therapy 1200 mg of calcium and 800 IU vitamin D daily 800 mg of calcium daily A 75 mg tablet of risedronate weekly 200 units of nasal salmon calcitonin daily 1200 mg of calcium and 800 IU vitamin D daily

A 53-year-old woman presents for an annual examination. She has a history of asthma for which she takes inhaled steroids and ß-agonists. She has no history of bone fractures and no family history of osteoporosis. She exercises regularly. Her menses used to be regular, but have just started to show some irregularity. She believes she might be entering menopause and asks for advice to prevent osteoporosis. What treatment will be recommended?

Progestin during amenorrhea if not already on contraceptives

A 54-year-old female patient with diabetes mellitus on metformin presents with a 3-week history of vaginal spotting. Endometrial biopsy pathology indicates endometrial cancer. Menarche was at age 15; menses were generally regular in the teens and 20s. The patient was amenorrheic while using depot medroxyprogesterone acetate, then menses returned but were irregular in frequency in the 30s. The patient believes last menstrual period to have been around age 42. The patient is G8P6Ab2, first child delivered at age 19; there were 6 spontaneous vaginal births with 2 first-trimester spontaneous abortions. Previous methods of contraception included combined estrogen-progestin oral contraceptives in the 20s and depot medroxyprogesterone acetate (DMPA) for 5 years in the early 30s. The patient discontinued contraceptive use in the mid-30s and has rarely been sexually active since then. What intervention would have had the best effect in preventing this patient's development of endometrial cancer?

Tenderness at the acromioclavicular joint Restricted active and passive motion of the shoulder Positive Neer and Hawkins signs Positive sulcus sign Edema of the shoulder muscles Positive Neer and Hawkins signs

A 54-year-old male patient presents with a 2-month history of left non-dominant shoulder pain. There is no history of trauma, but the pain began about a week after shoveling wet heavy snow from a 100-foot driveway. At first, the pain seemed to come and go from day to day, but it has progressively worsened and has become more constant. Pain is worse with overhead use. He cannot sleep on the left side and will wake up if he rolls over onto the left shoulder. Drop arm test is negative. Considering the most likely diagnosis, what other exam finding is expected?

Protein deficiency

A 54-year-old man is admitted to the intensive care unit and intubated on a ventilator after a serious motorcycle accident where he sustained multiple fractures, spinal cord trauma, and a liver laceration. On his fifth week of hospital admission, blood cultures from a central line reveal staphylococcal septicemia. After another month in the intensive care unit, the patient is found to have ascites and pitting edema. Vitals are within normal limits. Ultrasound demonstrates a fatty liver and unremarkable gallbladder. Echocardiogram shows no structural defects and a normal ejection fraction. Question What is the most likely diagnosis?

Nephrogenic diabetes insipidus

A 54-year-old man notices that he has very large urine output and he is constantly thirsty. In addition to urinating large volumes during the day, he awakens at night to urinate. He has a 25-year history of a bipolar disorder, which is treated effectively with lithium. His lab results are as follows: A water deprivation test with exogenous vasopressin administration is done under meticulous supervision. His urine osmolality does not increase. What is the most likely diagnosis?

Nontender fluid-filled lesion that transilluminate

A 54-year-old man presents with a recent lump in his scrotum. After answering questions about possible symptoms and undergoing a thorough genitourinary examination, it is determined that the lump is actually a collection of fluid in the patient's tunica vaginalis. What finding most closely supports the most likely diagnosis?

Negatively birefringent needle-shaped crystals Positively birefringent needle-shaped crystals Brownish clumps with elevated WBCs and negative bacteria Increased polymorphonuclear leukocytes, WBCs, and positive bacteria Decreased polymorphonuclear leukocytes, WBCs, and negative bacteria Negatively birefringent needle-shaped crystals (Gout)

A 54-year-old man presents with acute onset of excruciating pain in his right toe. The patient states the pain began shortly after dinner, and it has progressively worsened since then to the point where he is now unable to bear weight on the affected side. In addition, he explains that aside from occasional backaches, he has never experienced pain like this before. He reports overall good health and aside from a multivitamin, he uses no medications or supplements. On exam, the patient's right foot is swollen, and the joint of the great toe is tense and inflamed. His temperature is 37°C, blood pressure is 155/85 mm Hg, and pulse is 103 beats per minute. Labs reveal an elevated serum uric acid level. What is examination of synovial fluid from the affected joint space most likely to reveal?

Menopausal hormone therapy

A 54-year-old woman presents for her annual pelvic examination. Her last menstrual period was 1 year ago, and her last few cycles were extremely irregular. She describes multiple daily episodes of severe, intense heat in the face and trunk accompanied by sweating. She states that these "heat episodes" have been occurring 4-6 times daily for 4 months, and they interfere with her everyday activities and sleep. She reports no other symptoms. She has received yearly annual pelvic examinations and clinical breast exams and mammograms without any significant findings. Her past medical history is negative for cardiovascular disease, blood clots, and breast cancer. Her pelvic examination has findings of excessive dryness and apparent vaginal wall atrophy. What pharmacologic intervention can assist in minimizing these symptoms?

Stimulant medications are considered first-line pharmacologic therapy

A 7-year-old boy presents because he is fidgety, impulsive, and unable to sit still. The patient is observed running around. There is no evidence of any hallucinations or delusions. The mother notes that the child speaks excessively and loudly, makes simple arithmetic errors, and has short-term memory deficiencies. He finds it difficult to wait in lines or wait his turn in games or group situations. What is correct regarding this patient's condition?

Prescription of very hot foot soaks twice daily Wearing adhesive tapes on the feet to avoid friction Wide amputation of her forefoot as this ulcer likely will not heal Prophylaxis with daily low-dose antibiotic to cover skin flora Patient education and risk factor prevention Patient education and risk factor prevention

A 54-year-old woman with history of type 2 diabetes mellitus presents with a new foot ulcer. Her foot has also become swollen and red. She has been on hypoglycemic medication for the last 5 years; recently, her metformin dose was increased to 850 mg twice daily; it is combined with glipizide 10 mg twice daily. She is also on benazepril 10 mg daily for mild hypertension and microscopic proteinuria discovered at her annual physical earlier this year. ; blood pressure is 123/84 mm Hg. The plantar surface of the right foot has a 2 cm shallow ulcer with a film of purulent, serous drainage. She has greatly decreased pinprick sensation in both feet, and she cannot feel vibration. Dorsalis pedis pulses are 4/5 bilaterally. Laboratory evaluation reveals a WBC count of 17.7 X 103/ìL, with 93% neutrophils. Her HbA1c is 8.8%, and glucose is 241 mg/dL. What could have been done to prevent this wound (and future wounds) from forming in this patient?

Active alcohol use disorder Acute heart failure Elevation of islet-specific autoantibodies Estimated GFR 45% Ketones found in urinalysis Estimated GFR 45%

A 55-year-old male patient presents for an annual physical. On review of systems, he admits to excessive thirst and new frequent urination. Blood pressure 138/88 mm Hg, respirations 16/min, pulse 70 bpm, weight 258 lb, height 5'9", SpO2 96%, temperature 97.8°F. Labs reveal normal electrolytes, liver function tests, random glucose 247 mg/dL. The provider decides to initiate drug therapy. When is it still acceptable to use the first-line therapy drug in this patient?

Calcium restriction Caloric restriction Cholesterol restriction Phosphorous restriction Protein restriction Protein restriction (chronic kideny disease)

A 55-year-old male patient presents with a pale appearance and bilateral ankle edema that started 1 week ago and has progressively worsened. He has a 5-year history of type 2 diabetes, but the last physical exam was 2 years ago. He does not take any medications because he prefers to "control the health with a healthy lifestyle." Temperature 37°C, blood pressure 155/100 mm Hg, heart rate 80, respirations 15/min. He is otherwise well-developed and well-nourished; BMI 25 kg/m2. In addition to starting glycemic and blood pressure control, what dietary advice can help slow the progression of the new diagnosis?

Chronic lymphocytic leukemia Acute lymphoblastic leukemia Chronic myelogenous leukemia Acute myelogenous leukemia Polycythemia vera Chronic myelogenous leukemia

A 55-year-old man presents with a 1-week history of fatigue, night sweats, and abdominal fullness. On physical examination, you note a palpable spleen. You order a CBC; the results indicate a white blood count of 105,000 cells/mcL with a left shift of the myeloid series. The red blood cell count and morphology show anemia, and he has an elevated platelet count. To help confirm your suspicions, you order genetic studies, and the results come back with the BCR/ABL gene detected. What is the most likely diagnosis?

Amebiasis Occupation Alcohol consumption Metformin Hypertension Occupation

A 55-year-old man presents with painless gross hematuria since last evening. Urine cytology and cystoscopy reveal transitional cell carcinoma of the bladder. He has had diabetes and hypertension treated with metformin and atenolol, respectively, for the past 10 years. He has never smoked but consumes alcohol: about 2 pints of beer every day. He worked as a car mechanic at a garage for 30 years. He recently visited several African countries; upon returning, he had diarrhea and was diagnosed and treated for amebiasis. What in his history predisposed him to bladder cancer?

Complete blood count Fasting blood glucose Kidney biopsy Renal ultrasound Urine culture Fasting blood glucose (Diabetes rule out)

A 55-year-old patient presents to establish care. They recently went to a health fair, where they had some basic serum chemistries drawn. Serum creatinine was 1.5 mg/dL. There is no known past medical history except for occasional muscular aches, for which they take indomethacin, as needed (2-3 times in the past 8 months). There is no family history of renal disease. BP is 142/82 mm Hg, body mass index is 31 kg/m2. What additional study is most helpful in identifying the cause of this patient's renal insufficiency?

AST>ALT by a factor of 2

A 55-year-old woman presents with poor appetite and nausea. She has vomited 2 times over the past week and lost 4 pounds in the past month. Past medical history is significant for 20 years of alcoholism, 5 years with diabetes, and hypertension. She takes no medications; she is not involved in any therapy for her alcoholism. She has been drinking 4 12oz beers almost every day for the past 20 years, consuming greater quantities on weekends. Her vitals include a heart rate of 102 BPM, blood pressure of 140/100 mm Hg, respiratory rate of 20/min, and a temperature of 99.8°F. Physical exam reveals hepatomegaly. A liver biopsy reveals macrovesicular fat, spotty necrosis, and polymorphonuclear infiltration. What lab finding would be characteristic of this patient's condition?

Russell bodies (Multple Myleoma)

A 55-year-old woman presents with weakness, bone pain, and lethargy. A 24-hour urine sample reveals a spike of M protein. Based on the most likely diagnosis, what additional finding on bone marrow aspirate would support the diagnosis?

Potassium chloride Calcium carbonate Lisinopril Sodium bicarbonate Calcium citrate Lisinopril

A 55-year-old woman with a 15-year history of type 2 diabetes presents for follow-up. Her spot albumin/creatinine ratio was 100 mg/g 4 months ago and was confirmed at 100 mg/g yesterday. Her urinary analysis shows no cells, casts, or blood. Her creatinine is 0.7 mg/dL, and her estimated glomerular filtration rate is 95 mL/min/1.73 m2. What medication should you prescribe to help prevent her progression from micro- to macroalbuminuria and to help prevent progressive decline in glomerular filtration rate?

Antibiotics for 10 days Chemotherapy Left nephrectomy Hormonal therapy Observation Left nephrectomy (remove the kidney)

A 55-year-old woman with no significant medical problems presents with a CC of pink urine, stating, "I think I have blood in my urine." She states she has no pain with urination, but the hematuria is persistent. On questioning, she states that she has had a 1-month history of some progressively worsening left flank pain. The pains are not debilitating, but they are nagging. She admits to a 50 pack-year smoking history, and she states she is currently retired from her job as a teacher. Vital signs: T 98.6°F, BP 118/76 mm Hg, P 78/min, R 14/min. Abdominal exam reveals a left side abdominal mass. Urine dipstick only shows too numerous to count RBCs, and urine cultures are negative. CT scan of the abdomen and pelvis with and without contrast reveals a 4.2 cm solid enhancing lesion in the left renal parenchyma. What is the most effective treatment for this patient?

Finasteride Surgical therapy Terazosin Transrectal ultrasound with prostate biopsy Urine culture and sensitivity Terazosin

A 56-year-old African American man presents with urinary hesitancy, frequency, and nocturia. He gets up to urinate 3-4 times per night, unsure if he empties his bladder completely. This has been worsening for 2 years. His urinary stream is weaker than it was 1 year ago. He denies hematuria, dysuria, or history of UTIs. He has no significant past medical or surgical history. The remainder of the history and ROS is non-contributory. Vital signs are stable, and the patient is afebrile. General physical exam is unremarkable. Genital exam reveals a circumcised penis with no lesions or discharge. There is no inguinal adenopathy. Testicles are descended bilaterally with no lesions, masses, or hernias. Rectal exam reveals a smooth prostate with no nodules or tenderness. Urinalysis is normal, and prostate-specific antigen (PSA) test is within normal range for age. After emptying 250 mL of urine, the post-void residual urine volume is 50 mL. What is the most appropriate intervention?

Vulvar and vaginal atrophy (menopause)

A 56-year-old Asian woman presents to establish care for a wellness exam. She recently relocated from out of state; she brings a copy of her medical notes and labs from her previous medical provider. The patient tells you that sexual intercourse is very uncomfortable for her. She admits insertional dyspareunia and poor lubrication, but she denies deep dyspareunia. Outside of intercourse, she denies pelvic issues. She denies vaginal bleeding, pruritus, or discharge. She also denies urinary issues, such as dysuria, frequency, and incontinence. She has occasional hot flashes, but she feels they are tolerable. Her relationship is otherwise healthy, and she does not feel pressured into intercourse against her will. She is a G2P1. Surgical history includes open fracture reduction of the left ankle at age 22 and a dilatation and curettage (D&C) at age 24 for a spontaneous abortion (SAB). She is divorced, and she has been in her current monogamous relationship for 1.5 years. She is a non-drinker and non-smoker; she denies recreational drug use. Family history is unremarkable. The table indicates laboratory tests that were done approximately 1 year ago. Based on this patient's history and lab results, what physical exam finding do you expect?

Size of 1 cm Soft consistency Immobility Tenderness Suboccipital location Immobility - the most concerning ascept

A 56-year-old man presents with a painful lump on his neck that has been bothering him for the past 2 weeks. He denies other symptoms or recent illness. He has a past medical history of hypertension and kidney stones. Nursing staff reports that his temperature is 97.9°F, heart rate is 65, respirations 12, and blood pressure is 140/88. During your examination, you palpate an enlarged lymph node on his left suboccipital region that is soft and about 1 cm in size. The lymph node is non-movable, non-erythematous, and without warmth or fluctuance. What physical exam finding is most concerning?

Polyarticular arthritis Small lower extremity joints Caucasians more likely than African Americans Women more likely than men Under age 25 Small lower extremity joints

A 56-year-old man wakes up in the morning to find that he has a swollen, red, and painful big toe on his left foot. He had been on a cruise to the Bahamas 2 days earlier, and he spent much of the time eating and drinking. He normally has 1 glass of wine with dinner on the weekends, but his alcohol consumption increased substantially while on the cruise. He also did a great deal of walking in an attempt to make up for his excesses. He goes into his physician's office, and tests are run. An X-ray shows no acute fracture, and his vital signs are within normal limits. Blood work shows an increase in uric acid, but it is otherwise normal. He begins treatment and feels better within 24 hours. What most accurately describes the typical presentation of this disease?

Low C-peptide levels

A 56-year-old woman presents with sudden onset of palpitations, trembling, sweating, anxiety, headache, and confusion that started 1 hour ago after a 5-mile early morning run. She has had similar episodes in the past, but never any symptoms this severe. Sometimes she wakes up in the morning with headaches and trembling, but they usually go away after she has gotten ready and has breakfast. On physical exam, she is found to have heart rate 114, blood pressure 125/86, respiration rate 18, weight 160 lb, and temperature 98.7°F. Patient is alert and appears somewhat anxious and diaphoretic but otherwise well. HEENT exam is unremarkable. Cranial nerves, cerebellar function, strength, sensation, deep tendon reflexes, and balance testing/Romberg are all normal. Patient is tachycardic, but S1 and S2 are normal with no murmurs, rubs, or gallops. Lungs are clear to auscultation bilaterally. EKG shows sinus tachycardia. Lab work is significant for a glucose level of 36. What would be most likely if the patient's low blood glucose was due to accidental, surreptitious, or malicious administration of exogenous insulin such as glargine (Lantus)?

No treatment Prophylactic elective cholecystectomy Oral analgesics Urgent cholecystectomy Oral dissolution therapy No treatment

A 57-year-old woman is undergoing a workup by her primary care provider for abdominal pain. The pain is in her left lower quadrant, intermittent, "crampy," and has been present for about 2 months. As an initial imaging study, she underwent an abdominal ultrasound. The ultrasound was unremarkable except for the presence of a few small (<1 cm) stones in her gallbladder. She is currently following up to discuss the resilts of ultrasound . What is likely to be suggested for the gallstones?

Dexamethasone suppression test

A 60-year-old man presents with difficulty climbing stairs, dyspnea, and fatigue. He has gained 30 lb over the past year. On examination, he is found to have edema, pigmentation of the skin, palmar creases, and proximal muscle weakness. Chest X-ray shows an irregular mass in the right upper lobe. Lab values show an increase in evening cortisol levels and an increase in ACTH. What is the next step in diagnosis?

Levothyroxine

A 58-year-old Caucasian man presents to his primary care physician's office reporting lethargy. His heart rate is 44 beats per minute.The patient is sent to the emergency department where he is treated with atropine 0.5 mg x 1 dose. Heart rate then increases to 57 beats per minute, and the patient is admitted to the hospital for further evaluation. Cardiology consultation is requested. The patient reports no previous history of cardiac disease. He denies symptoms of chest pain or pressure but admits to intermittent lightheadedness and mild dyspnea on exertion with moderate activity in the last 2-3 months. He denies paroxysmal nocturnal dyspnea, orthopnea, lower extremity edema, and palpitations. He denies frank syncope. He admits to fatigue and lethargy for approximately the past 3 months. He had an exercise tolerance test 10 years ago that was reportedly normal. His last physician's office visit was 2 years ago. His past medical history is notable only for high cholesterol and history of tonsillectomy. He has no known drug allergies. His only daily medication is Lipitor 20 mg once daily. He does not smoke, drink alcohol, or use recreational drugs. Review of systems reveals cold intolerance and episodes of constipation. Physical examination reveals an alert Caucasian man who is somewhat slow to speak and respond. His voice is moderately hoarse. He has coarse facial features and dry skin. Mild periorbital puffiness is noted. Deep tendon reflexes are delayed. 12-lead EKG reveals sinus bradycardia, rate 52, without evidence of prior myocardial infarction, ischemia, left ventricular hypertrophy, or bundle branch block. The EKG is otherwise within normal limits. Lab work confirms the suspected diagnosis. What is the best initial treatment for this patient?

Thyroid stimulating hormone and free T4 level

A 58-year-old Caucasian man presents to his primary care physician's office reporting lethargy. His heart rate is 44 beats per minute.The patient is sent to the emergency department where he is treated with atropine 0.5 mg x 1 dose. Heart rate then increases to 57 beats per minute, and the patient is admitted to the hospital for further evaluation. Cardiology consultation is requested. The patient reports no previous history of cardiac disease. He denies symptoms of chest pain or pressure but admits to intermittent lightheadedness and mild dyspnea on exertion with moderate activity in the last 2-3 months. He denies paroxysmal nocturnal dyspnea, orthopnea, lower extremity edema, and palpitations. He denies frank syncope. He admits to fatigue and lethargy for approximately the past 3 months. He had an exercise tolerance test 10 years ago that was reportedly normal. His last physician's office visit was 2 years ago. His past medical history is notable only for high cholesterol and history of tonsillectomy. He has no known drug allergies. His only daily medication is Lipitor 20 mg once daily. He does not smoke, drink alcohol, or use recreational drugs. Review of systems reveals cold intolerance and episodes of constipation. Physical examination reveals an alert Caucasian man who is somewhat slow to speak and respond. His voice is moderately hoarse. He has coarse facial features and dry skin. Mild periorbital puffiness is noted. Deep tendon reflexes are delayed. 12-lead EKG reveals sinus bradycardia, rate 52, without evidence of prior myocardial infarction, ischemia, left ventricular hypertrophy, or bundle branch block. The EKG is otherwise within normal limits. What test is most likely to confirm your suspected diagnosis?

Anemia Chronic renal failure Depression Hypothyroidism Sick sinus syndrome Hypothyroidism

A 58-year-old Caucasian man presents to his primary care physician's office reporting lethargy. His heart rate is 44 bpm. The patient is sent to the emergency department where he is treated with atropine 0.5 mg x 1 dose. Heart rate then increases to 57 bpm, and the patient is admitted to the hospital for further evaluation. Cardiology consultation is requested. The patient reports no history of cardiac disease. He denies symptoms of chest pain or pressure but admits to intermittent lightheadedness and mild dyspnea on exertion with moderate activity in the last 2-3 months. He denies paroxysmal nocturnal dyspnea, orthopnea, lower extremity edema, palpitations, and frank syncope. He admits fatigue and lethargy for 3 months. He had an exercise tolerance test 10 years ago that was reportedly normal. His last physician's office visit was 2 years ago. His past medical history is notable only for high cholesterol and history of tonsillectomy. He has no known drug allergies. His only daily medication is Lipitor 20 mg once daily. He does not smoke and does not use alcoholic beverages or recreational drugs. Review of systems reveals cold intolerance and episodes of constipation. Physical examination reveals an alert white man who is somewhat slow to speak and respond. His voice is moderately hoarse. He has coarse facial features and dry skin. Mild periorbital puffiness is noted. Deep tendon reflexes are delayed. 12-lead EKG reveals sinus bradycardia, rate 52, without evidence of prior myocardial infarction, ischemia, left ventricular hypertrophy, or bundle branch block. The EKG is otherwise within normal limits. What is the most likely diagnosis?

Ertapenem Hyoscyamine Lactated Ringer's Pancrelipase Promethazine Lactated Ringer's

A 58-year-old man presents with a 1-day history of severe abdominal pain, nausea, and vomiting. He initially thought he had some indigestion with pain located in the epigastric region and tried some calcium carbonate (Tums) with no relief. The pain and vomiting progressed through the night and kept him from sleeping and going to work. He feels the pain boring through to his back. He denies hematemesis, fever, diarrhea, out-of-the-country travel, and contact with sick people. Prior to onset of pain, he reports good health. He has no known medical conditions and takes no medications. He has had no surgeries. He smokes cigarettes (40 pack-years), admits "moderate" alcohol use, and denies drug use. He is married and works as a welder. Vitals are: BP: 102/56 mm Hg; HR: 116 bpm; RR: 15; Temp: 98.9°F; O2 Sat: 95% on room air. On physical exam, the patient appears uncomfortable on the exam table and grimaces when changing position for exam. He is cooperative, alert, and oriented. Abnormal physical exam findings include distended abdomen, decreased bowel sounds, and epigastric region tenderness with guarding. He is tachycardic. No jaundice is noted. The remainder of the exam is normal. This patient's test results are shown in the table. What pharmacologic treatment is the most important intervention for this patient's most likely condition?

Chronic lymphocytic leukemia (CLL) - lymphadenopathy ** key

A 58-year-old man presents with a 4-month history of worsening fatigue. Physical examination is remarkable for right-sided posterior cervical and anterior clavicular lymphadenopathy. He has splenomegaly. Lab results reveal a WBC count of 250,000/mm3 (normal: 5000-10,000 mm3) with 77% lymphocytes (normal: 20-40%). Peripheral smear demonstrates small-but-mature-appearing lymphocytes. A subsequent bone marrow biopsy reveals variably infiltrated small mature lymphocytes that stain for CD5 and CD19. Philadelphia chromosome is negative. What is the most likely diagnosis?

Esomeprazole (Nexium) Cimetidine (Tagamet) Metoclopramide Nitroglycerin Esomeprazole (Nexium)

A 58-year-old man with a history of COPD, hyperlipidemia, a 40 pack-year smoking history, and obesity is being evaluated at his primary care office for complaints of post-prandial regurgitation that is associated with an acidic sensation in his mouth and a chronic, nonproductive cough, all of which have been occurring over the past year. He has taken over the counter famotidine (Pepcid) without any relief. He denies any fever, chills, changes in weight, diaphoresis, chest pain, shortness of breath, sputum, palpitations, abdominal pain, or changes in bowel habits. His physical exam reveals an obese BMI, but it is otherwise unremarkable. His stool hemoccult is negative. Bloodwork reveals an iron-deficiency anemia; a chest radiogram was without pulmonary disease. An upper barium esophagram noted an outpouching of barium at the lower end of the esophagus and a wide hiatus through which gastric folds are visible above the diaphragm. H. pylori antibody testing and urea breath tests were negative. What is the most appropriate therapy for this patient at this time?

Increase use of prednisone to daily and stop etanercept. Initiate systemic combination hormone therapy. Repeat DEXA in 6 months. Send for radionuclide bone scan. Start bisphosphonate therapy Start bisphosphonate therapy

A 58-year-old menopausal woman presents for a wellness exam with no current problems or vasomotor symptoms. She has recently been caring for her mother after a hip fracture. Past medical history is remarkable for stable rheumatoid arthritis since age 28 with weekly etanercept. Her rheumatologist has periodically prescribed a few weeks of prednisone for flares. No other medications, allergies, or surgeries. Her older sister has osteoporosis. She has four grown children, lives with her husband, and is a homemaker. She denies alcohol or drug use but admits to smoking 1 pack of cigarettes per day. Vitals and exam are normal. Some preventive screenings are done. Labs, mammography, and Pap smear return unremarkable. DEXA scan shows a T-score of -3.22. What intervention is most appropriate?

Acute alcohol intoxication Delirium tremens Wernicke encephalopathy Korsakoff syndrome Beriberi Wernicke encephalopathy - Wine = wenicke - Wacky walking (disorientation, confusion, indifference, inattentiveness, and incoordination/gait ataxia)

A 58-year-old patient presents for further evaluation after being brought in by the local police due to an unstable gait and disheveled appearance; breathalyzer test is 0.08 g/dL. On exam, the patient is gaunt and malnourished; they are disoriented and confused and unable to respond to questioning. The right elbow and knee are bruised, but the patient appears to have no other significant injuries; eye movements are uncoordinated and bounce from side to side. They cannot fixate their gaze. Blood pressure89/58 mm HgPulse109Temperature35.7°C What diagnosis is most consistent with the patient's signs and symptoms?

Diazepam (Valium) Methadone (Methadose/Dolophine) Naloxone (Narcan) Naltrexone (ReVia/Vivitrol) Varenicline (Chantix) Naltrexone (ReVia/Vivitrol) (Halt that alcholol)

A 58-year-old woman presents for a follow-up accompanied by her adult daughter. A few months ago, the patient had gone to the emergency department via ambulance after the daughter discovered her mother confused and shaking at home. The patient underwent several days of inpatient treatment for alcohol withdrawal. Initial symptoms included hallucinations, tremor, nausea, anxiety, insomnia, and a seizure. The patient denies current symptoms and admits increased tolerance to greater amounts of alcohol and loss of control with a frequent need for the substance. Her work and home relationships suffer due to her condition. The patient has no other medical conditions, is menopausal, has had no surgeries, takes no medications, and has no allergies. The patient readily admits alcohol abuse and dependence with a desire to prevent relapse. She has not used any other substances, and has been abstinent of all substance use since hospital discharge. She is currently in individual and group programs to assist her in relapse prevention. She would like pharmacological help to maintain her sobriety. What medication is most appropriate in relapse prevention for this patient?

Perform a transvaginal ultrasound. Prescribe a proton pump inhibitor. Refer the patient for an upper endoscopy. Prescribe medication for irritable bowel syndrome. Observation and reassessment in 1-3 months. Perform a transvaginal ultrasound - there is a mass

A 58-year-old woman with no significant past medical history presents with a 6-month history of "heartburn," sometimes occurring after meals. There is associated fatigue, bloated abdominal sensation, early satiety, and alternating constipation and diarrhea. She denies fever, chills, changes in weight, chest pain, shortness of breath, abdominal pain, nausea, vomiting, melena, hematochezia, and vaginal discharge. Her last menstrual period was 4 years ago. She is unmarried, and she does not have any children. Her physical exam reveals normal vital signs and a normal cardiopulmonary exam. Her abdomen is protuberant; there is a shifting dullness and a fluid wave noted. The pelvic exam reveals a solid irregular fixed lesion in the left lower abdomen. What is the most appropriate initial intervention for this patient at this time?

IgA IgM IgE IgD IgG IgG - Multiple Myeloma

A 59-year-old man starts to notice that he no longer has the energy for his morning jog. He starts to develop back pain, so he completely stops exercising, but this does not seem to help. He is always tired and seems to have a lot of aches and pains. One day, his back pain is particularly bad, so he sees his primary care physician. He also acknowledges constipation and polyuria. On physical exam, his doctor notes that he is pale. His laboratory tests are as follows: An X-ray of the lumbar spine reveals "punched out" lesions. What immunoglobulin is most frequently elevated in this condition?

Approximately 25-50% of 6- to 12-month-olds have nighttime awakening

A 6-month-old healthy male infant presents with an inability to sleep through the night. He has 3-4 nighttime awakenings, and his parents spend a long time getting him back to sleep. Both parents work, and their sleep remains disrupted, which affects their work performance the next day. What best describes the physiology of sleep in this patient?

Exploratory laparotomy Gastric aspirate for Helicobacter pylori High dose intravenous methylprednisolone Intravenous fluids and abdominal ultrasound Trial of oral rehydration Intravenous fluids and abdominal ultrasound

A 6-week-old male infant presents with a 2-day history of vomiting after every feeding of cow's milk-based formula with iron, 4 ounces per feeding. There has been no fever, diarrhea, or other symptoms except increased crying. The child appears alert and hungry. The mother describes the vomiting as forceful, traveling about 2 feet. Physical evaluation reveals minimal tear production with mild skin tenting. Bowel sounds are decreased. BUN 29 mg/dL; serum sodium 129 mg/dL; serum potassium 3.4 mg/dL; serum chloride 89 mg/dL; serum bicarbonate 34 mg/dL. What is the next step in this patient's care?

Legg-Calve-Perthes disease Osteomyelitis Primary lymphoma of bone Osteoid osteoma Ewing sarcoma Osteoid osteoma (small round lytic lesion surrounded by mild reactive bone formation.) (Pain at night)

A 6-year-old boy has gradually increasing sharp pain in his left anterior hip that seems worse at night. He is moderately overweight, but he remains active and plays baseball. There is no history of trauma. There is no redness, swelling, or fever; there is no involvement of the other hip. Ibuprofen seems to help. There is no family history of any bone or joint disease. His weight is at the 95th percentile and height is at the 50th percentile. On exam, there is mild tenderness to palpation over his left anterior hip, but the rest of the exam is normal. Laboratory studies show a normal complete blood count, hemoglobin, sedimentation rate, platelets, and C reactive protein. Plain X-rays show a small round lytic lesion surrounded by mild reactive bone formation. What is the most likely diagnosis?

Splenic sequestration Myocardial infarction Mycoplasma pneumonia Sickle cell disease Pneumocystis carinii infection Sickle cell disease

A 6-year-old boy is brought to the emergency room with chest pain and signs of respiratory distress. His mother states he has been ill with an upper respiratory infection that suddenly worsened with temperature elevation. Physical examination reveals an increased P2 and systolic ejection murmur. Chest X-ray shows lobar infiltrates. Labs show a leukocyte count of 18,000/mm3, hemoglobin 7 g/dL, and reticulocytes 12%. What is the most likely cause of chest pain and respiratory distress?

Problems with phonology Semantic deficit Weak metalinguistics Expressive language dysfunction Receptive language dysfunction Receptive language dysfunction

A 6-year-old boy is performing poorly in kindergarten. According to his teacher, he can perform some schoolwork and answer test questions, but he has difficulty understanding the instructions. He has to ask for detailed help regarding instructions, and his teacher sometimes has to demonstrate it to him with action so that he can proceed with his lessons. He is the second of three children. Except for language skills, developmental milestones are appropriate for his age. At 15 months, he could not respond to simple instructions like "no" and "give me." At the time, his mother thought, "He's just a little different from the other siblings." He has always had difficulty with school. What is the most likely diagnosis?

Type 1 diabetes mellitus

A 6-year-old boy presents due to lethargy, polyuria, nocturnal enuresis, and polydipsia. His mother tells you that he reports being tired and thirsty all the time. You note that he has lost 5 lb since his last visit 6 months ago. What is the most likely diagnosis?

GU referral Intravenous pyelography (IVP) Renal ultrasound Urine culture Urinalysis Urinalysis

A 6-year-old boy presents with his mother, who is concerned because he wets his bed 2-3 times a week at night. He has no urinary symptoms during the day. He is the second child of three. He is in the first grade and is struggling with performance. He has had no medical problems; there is no history of developmental delay, and he was the product of a normal uncomplicated pregnancy and delivery. Since the birth of the third child, his behavior has been poor. The vital signs are normal, and examination of other body systems is unremarkable. What investigation is most important in this child's workup?

Dextroamphetamin

A 6-year-old boy's parents are concerned about their son's behavior. There have been complaints from his teachers that he is frequently fidgeting and disruptive in class. His parents state that he is extremely active at home and requires frequent discipline. After a complete history and physical exam, your suspected diagnosis is attention-deficit/hyperactivity disorder (ADHD). In addition to behavioral therapy, what drug should be considered as first-line therapy for the management of this disorder?

Aversion therapy Biofeedback Flooding Systematic desensitization Token economy Biofeedback

A 60-year old woman has a history of stress-related migraine headaches. Many conventional treatments have failed. What treatment would you now try in this patient?

Glycerin suppository Oral bisacodyl Oral lactulose Oral psyllium Oral senna Oral psyllium

A 60-year-old male patient with hypertension presents with constipation. He states that he often only has two bowel movements weekly. This has been an ongoing problem for months, but the situation has worsened since being released from the hospital after treatment for a myocardial infarction a few weeks ago. What is the most appropriate treatment?

Increased fluid intake Doxazosin Sodium bicarbonate Ofloxacin Potassium Doxazosin --> help with enlarge prostate

A 60-year-old man presents with difficulty initiating voiding, incomplete emptying, and increasing urinary frequency over the past few months. He has no history of stones, cancer, surgery, diabetes, or AIDS; he takes no medications. His physical exam shows a temperature of 98.6°F, a blood pressure of 128/78 mm Hg, suprapubic fullness, an enlarged prostate, and no peripheral edema. The remainder of his exam is normal. He has been referred to urology. What medical intervention will help prevent the loss of renal function that may accompany his primary disorder?

Systemic lupus erythematosus Granulomatosis with polyangiitis Rheumatoid arthritis Anti-GBM antibody disease Granulomatosis with polyangiitis (palpable purpura, hemoptysis, hematuria, and signs of glomerular disease) - bloodo in urine - no swollen joints

A 60-year-old man with a history of recurrent sinus infections presents with hemoptysis and hematuria. Physical examination shows a temperature of 101°F, a blood pressure of 145/85 mm Hg, decreased breath sounds on his right lower lobe, and palpable purpura on his bilateral lower legs. No warm or swollen joints were noted. What is the most likely diagnosis?

Pseudocyesis Obsessive-compulsive disorder Chronic fatigue syndrome Body dysmorphic disorder Factitious disorder Body dysmorphic disorder - plastic surgery

A 60-year-old woman is referred to you for a bizarre fixation. She has become preoccupied with the issue of having breasts that are too small, and she has been shopping for plastic surgeons to fix the problem. She has become so fixated on her problem that she stays in the house, does not socialize with others, and tries to constantly hide the perceived problem by wearing clothes that enhance her breasts; sometimes, she even wears prosthetic breasts. She has become paranoid about what people are saying about her appearance. What is the most likely diagnosis?

Avoid travel. Increase dietary protein Limit frequent or prolonged walking. Restrict dietary sodium. Restrict dietary sodium

A 61-year-old man with known cirrhosis presents with a 1-week history of "puffy" ankles and increased shortness of breath. A week before symptom onset, he was on vacation, where he engaged in walking, sightseeing, and eating out. He admits more dyspnea with lying down and with increased exertion. His shoes feel snug, and he notes a definite line from wearing socks. The patient denies chest pain, leg pain, fevers, claudication, nausea, headache, lethargy, and hemoptysis. His past medical history is remarkable for cirrhosis and a history of alcoholism. He is awaiting a liver transplant. He had a liver biopsy, but no other surgeries. He takes no medications, has no allergies, and has abstained from alcohol for 9 months. He lives with his wife, works as an electrician, and smokes a pack of cigarettes per day. Vitals are normal, including oxygen saturation. On physical exam, the patient appears in no acute distress and with normal mental status. His physical exam is remarkable for mild jugular venous distention, 2-3+ edema in lower extremities, and mild dullness to lung percussion. No hepatomegaly or ascites are noted. Laboratory results are shown. ***large chart*** Once this patient is acutely stabilized, what is the best health maintenance for his condition?

Decreased CSF glucose content Decreased CSF protein content Elevated CSF polymorphonuclear cell count Elevated CSF protein content Elevated CSF lymphocyte count Elevated CSF protein content

A 62-year-old man presents with vision problems and difficulty swallowing. Over the last week, he has had a constellation of symptoms; they began with numbness and tingling in his feet and progressed to weakness that now affects both lower and upper extremities. He has started to notice difficulty swallowing and double vision since yesterday. It feels difficult for him to take a big breath. His past medical history is non-contributory, and he takes no medications. Exam reveals bilateral absence of patellar and ulnar reflexes. A lumbar puncture is performed to confirm the diagnosis. What cerebrospinal fluid (CSF) finding is most likely?

Normal saline Calcitonin Pamidronate Gallium nitrate Dexamethasone Normal saline

A 62-year-old man was diagnosed with multiple myeloma 1 year ago; he is currently hospitalized with intractable nausea and vomiting. Since he was asymptomatic at the time of diagnosis, he has not been treated for his cancer. His wife reports he has not eaten in several days, but he is constantly drinking water. He appears confused and lethargic. When adjusted for the albumin level, serum chemistries reveal an elevated level of calcium (12.0 mg/dL). What should the initial treatment of his hypercalcemia include?

Decreased perfusion on Doppler ultrasound Elevated hemoglobin A1C Elevated serum uric acid Radiograph showing patchy osteopenia Positive blood culture Radiograph showing patchy osteopenia (compartment regional pain syndrome)

A 62-year-old patient presents with excruciating pain, burning, and swelling in the left forearm and wrist. Symptoms began with a fracture 6 months ago, worsening despite strict adherence to instructions. They have continued using a sling and limiting left arm use to prevent exacerbation. Fabric touching the skin when wearing a jacket or long sleeves causes pain. The patient denies fevers, pain in other areas, new trauma, and urinary symptoms. Past medical history is unremarkable. No tobacco, alcohol, or recreational drug use. Physical exam is remarkable for mild left forearm edema, erythema, and tenderness to light touch. Distribution of findings is elbow to wrist, anterior and posterior surfaces. Left wrist strength and range of motion are decreased compared to right. Based on the most likely diagnosis, what is an expected diagnostic test finding?

CT of the spine Electromyogram Erythrocyte sedimentation rate MRI of the spine Plain radiographs of the spine MRI of the spine

A 62-year-old patient presents with pain in the back, buttocks, and posterior thighs that has persisted for several months. There are no radicular symptoms or bladder or bowel dysfunction. 2 years ago, the patient underwent a prostatectomy due to prostate cancer. What diagnostic test will provide the most information?

Fever, fatigue, weight loss Heberden's nodes Inflammatory changes in the joint fluid aspirate Redness and heat over the affected joints Swan neck deformity Heberden's nodes

A 65-year-old female patient presents with pain and morning stiffness in the knees and right toes. She was an athlete in school and college days. She has not been able to go on her daily morning walks for 3 months. Pain is worse in the evenings and improves with rest. There is no history of trauma or fractures. What additional changes are associated with this condition?

Avoidance of prolonged sitting Daily cranberry supplements Daily prophylactic antibiotics Oral or transdermal oxybutynin Weight loss Weight loss

A 62-year-old woman presents due to urine leakage for 2 years that has worsened. She leaks urine when she coughs or sneezes; she wears a pad daily as a result. The amount of leakage varies. The patient denies hematuria, dysuria, and pelvic pain. She sometimes feels vaginal pressure and fullness. She is considering quitting her job from embarrassment. Past medical history includes 4 vaginal deliveries. No other known medical conditions, medications, or allergies. She is married and works part-time at a call center; she denies tobacco, alcohol, and recreational drugs. On physical exam, she is obese, with an atrophic vulva/vagina. Pelvic examination reveals downward and forward rotation of the vaginal wall, with an anterior bulging when the patient is asked to strain. A dipstick urinalysis is normal. After completing treatment of her condition, what is the most appropriate way of preventing recurrence?

Esophageal adenocarcinoma

A 62-year-old woman presents to her physician with persistent reflux symptoms despite medical management. An upper endoscopy is performed, revealing the attached image. Biopsy findings reveal replacement of the esophageal squamous epithelium with columnar epithelium. Refer to the image. What is the most serious complication of her disorder?

Anticholinergic agents to reduce bladder reactivity Vaginal pessary placement Laparoscopic ventral suspension of the uterus Topical estrogen creams Daily manual replacement of uterus Vaginal pessary placement

A 62-year-old woman with diabetes and hypertension is evaluated for incontinence. She is found to have a cystocele and grade 3 uterine prolapse. Her diabetes and hypertension are both poorly controlled. On examination, the uterus can be repositioned with firm digital pressure. In addition to being declared unfit for surgery, she is not keen on undergoing any procedure. She has no history of any urinary infections or pelvic inflammatory disease. What is the next step in management?

Sex Anemia Diabetes Insulin Obesity Obesity

A 63-year-old man presents with pain in both knee joints that began almost 1 year ago. It was mild and present only during activity, but it has worsened and is present during rest. There is no joint stiffness. He has diabetes and has been on insulin treatment for 10 years. BMI is 30. Vitals are within reference ranges. Examination reveals pale conjunctivae. On examination of the knee joints, crepitus is present and range of motion is reduced, but there is no tenderness, warmth, or erythema. Osteoarthritis is suspected and lab investigations and x-ray are ordered. Meanwhile, exercises and ibuprofen as a pain reliever are prescribed. What is a risk factor for osteoarthritis in this patient?

A >2 cm widening of the acromioclavicular joint Cystic changes in the head of the humerus Narrowed glenohumeral joint space High-riding humeral head A Bankart lesion of the glenoid High-riding humeral head

A 63-year-old man with long-standing right shoulder pain from impingement syndrome presents because he cannot lift his arm after pulling the starter cord on his lawn mower 1 month ago. An AP view of his right shoulder is ordered. What finding on the plain films may indicate rotator cuff arthropathy?

Injected botulinum toxin Intravenous nitroglycerin Oral diltiazem Oral prednisone Oral metoclopramide Oral diltiazem (CCB) - esophagus spasms

A 63-year-old woman presents with a 6-month history of difficulty swallowing. Shortly after swallowing, she feels like something is getting stuck in her upper chest. The difficulty swallowing seems to be worsened when the patient is stressed and when she eats hot or cold food. She denies weight loss and night sweats and reports no other problems. Physical examination of the patient is otherwise non-contributory. An extensive gastrointestinal evaluation, including a comprehensive endoscopic evaluation, is negative. What is the first-line treatment for this patient's symptoms?

Abdominal examination, including liver percussion Percussion for costovertebral angle tenderness Scrotal examination with and without Valsalva Abdominal examination, including digital rectal exam Scrotal examination and inspection of urethral meatus Abdominal examination, including digital rectal exam - check the prostate

A 64-year-old man presents with progressive weakening of his urinary stream, nocturia, post void dribbling, and the sensation that he is not emptying his bladder. Annual screening blood work recently revealed a serum prostate-specific antigen of 4.3 ng/mL. What is the most important component to your physical examination of this patient?

Vaginal pessary Oral nitrofurantoin (Macrobid) Oral pentosan polysulfate sodium (Elmiron) Oral phenazopyridine (Pyridium or Azo) Oral tolterodine (Detrol) Vaginal pessary

A 64-year-old woman presents with urine leakage that has been progressively worsening for the past 6-12 months. She also admits to having vaginal dryness, poor lubrication with intercourse, and subsequent dyspareunia. Urinary leakage primarily seems to occur with coughing, sneezing, or high-impact exercise. She denies dysuria, hematuria, urgency, and vasomotor symptoms. She tries to empty her bladder every 2-3 hours and has cut out all alcoholic and caffeinated beverages, but symptoms persist. The patient is a G6P6. She has no chronic medical conditions and no past surgeries. She wants to do everything she can to avoid surgery. She is in a monogamous relationship. She was last seen 1 year ago with a normal well-woman exam and normal Pap smear. On physical exam, vitals are normal. The pelvic exam reveals dry pale vaginal mucosa with a bulge on the anterior vaginal wall that increases in size with a Valsalva maneuver. Urinalysis is normal. Given this patient's strong preference to avoid surgery, what else could be done to improve her urinary symptoms?

Sodium Potassium Vitamin C Thiamin Riboflavin Potassium - Furosemide is a diuretic and will kick out the potassium

A 65-year-old African American man presents in your office for a follow-up for hypertension. 3 months earlier, you prescribed furosemide. He checks his blood pressure daily and states that it is markedly lower since he has been on the medication, but it feels like his heart is skipping a beat once in a while. What deficiency is most likely?

Ethnicity History of BPH Smoking history Family history Occupational history Family History

A 65-year-old Caucasian man presents for a routine physical. He states that he is concerned about the development of prostate cancer. His history is significant for benign prostatic hyperplasia (BPH), for which he underwent a transurethral resection of the prostate (TURP) 3 years ago. His social history is significant for a 50 pack-year smoking history, and he worked for 40 years as a coal miner. His father died of prostate cancer at age 76. What factor would most likely contribute to his risk of developing prostate cancer?

Barium study Upper endoscopy and tissue biopsy Endoscopic ultrasound 24-hour pH monitor Manometry Barium study

A 65-year-old man presents because a morsel of meat he had eaten 3 days ago reappeared on his pillow this morning. About a year ago, he noticed difficulty swallowing—particularly solid foods—which seems to be worsening. His wife complains about his bad breath, and he notes that people avoid being close to him. He does not drink, does not smoke, and was in a good health before. His physical examination is within normal range for his age, except that you notice that he is repeatedly clearing his throat as if he is embarrassed. Question What is the best next step in the management of this patient?

Escherichia coli infection Urolithiasis Smoking Interstitial nephritis Schistosoma haematobium infection Smoking

A 65-year-old man presents with flank pain, blood in urine, and unexplained weight loss for 2 months. Past medical history is significant for numerous infections, kidney stones, cigarette use, and alcohol use. On physical exam, there is a palpable abdominal mass and a low-grade fever. What puts this patient at the greatest risk for the development of the most likely diagnosis?

Benign prostatic hyperplasia Overactive bladder Prostate cancer Prostatitis Urinary tract infection Benign prostatic hyperplasia

A 65-year-old man presents with gradual increase of urinary frequency over the past few months. Most recently, he has difficulties starting to urinate and the stream seems slow. Despite waking up several times a night to release his bladder, he continues to have a sensation of incomplete emptying. He has no history of sexually transmitted diseases, surgeries, or urinary tract infections. Upon examination, he is slightly overweight and his vitals are normal. His abdomen feels soft, the genitals are benign, and his prostate is non-tender and moderately enlarged (35-40 g). Lab tests are negative for blood presence in urine and indicate a protein-specific antigen (PSA) of 1.3 ng/mL. What is the most likely diagnosis?

Boil water before using

A 65-year-old man presents with watery diarrhea and mild abdominal pain. He has had 8-9 non-bloody bowel movements per day for the last 10 days; he currently feels a general abdominal discomfort. On examination, his pulse rate, blood pressure, and oral temperature are normal. There is no abdominal tenderness. The stool guaiac test is negative. He does not travel. He eats home-cooked meals most of the time, and he reports using water from his private well for all of his needs. In addition to consulting the EPA for decontamination advice, what recommendation would you provide this patient?

Leave the catheter to measure urine output. Assess for catheter removal with the surgeon on post-op day 3. Change the catheter tubing every 3 days. Give prophylactic antibiotics to prevent infection. Remove the catheter and monitor urine output by other means Remove the catheter and monitor urine output by other means. (removal of catheter to prevent UTI)

A 65-year-old man underwent an emergency laparoscopic cholecystectomy and arrives at the medical unit postoperatively with a Foley catheter. He is in stable condition, and his vital signs are stable. He is tolerating a regular diet, but his urine output is low, and he is admitted for monitoring. He has a foley catheter at this time to monitor urine output. What procedure is appropriate in the prevention of catheter-associated urinary tract infections in this situation?

Anal fissure

A 65-year-old man with a past medical history of hypertension, chronic arthritis, and herniated lumbar discs presents with a 1-week history of severe, tearing pain to his rectal area that occurs while he defecates. Following his bowel movement, he notes relief of the tearing pain, but he feels a throbbing sensation in the area. He has noted small drops of bright red blood on the toilet paper. He presently takes amlodipine and hydrochlorothiazide for hypertension as well as hydrocodone-acetaminophen for pain. He has noticed a reduced frequency of bowel movements of late due to pain. The physical examination is notable for a 5-mm midline crack-like lesion at the anus in the 6 o'clock position. The remainder of the physical exam is unremarkable. What is the most likely diagnosis?

CA125 Colonoscopy Genetic testing Mammogram Transvaginal ultrasound Transvaginal ultrasound

A 65-year-old woman presents with bloating, early satiety, changes in her bowel habits, and an unintentional 10 lb weight gain over 3 months. Upon further questioning, you discover that she never had children and her sister passed away 20 years ago from some sort of cancer in her belly. On her physical exam, an adnexal mass is felt. What is the most appropriate next step to further evaluate this patient?

NSAIDs Methotrexate Sulfasalazine Corticosteroids Selective COX-2 inhibitors Methotrexate

A 65-year-old woman presents with fatigue, loss of energy, decreased appetite, low-grade fever, muscle and joint aches, and stiffness. She has glucose-6-phosphate dehydrogenase (G6PD) deficiency and a history of coronary artery disease. On examination, you find swollen, painful, tender small joints in a symmetrical pattern. Laboratory evaluation reveals positive rheumatoid factor and antibodies to cyclic citrullinated peptides. What long-term therapy should be recommended to modify the course of this disease?

Myasthenia gravis (presence of antibodies to acetylcholine receptor)

A 65-year-old woman presents with general weakness for the past few days; it gets worse as the day progresses. She says she has been having double vision, and you notice she has poor posture. A blood test reveals the presence of antibodies to acetylcholine receptor. A repetitive nerve stimulation test showed a decremental response. What is the most likely diagnosis?

Multiple myeloma

A 65-year-old woman presents with pain in the back, chest, and at the right seventh rib. She appears to be quite pale, and she admits to fatigue. These symptoms have come on gradually over several weeks, with the back pain becoming the reason for consulting the physician. Physical examination reveals localized tenderness at the spine of T8 and ribcage with tenderness at the right seventh rib. CBC shows a normochromic normocytic anemia with hemoglobin of 8 g/dL. Peripheral smear shows marked rouleaux formation with normal platelet and white cell counts and morphology. Serum chemistry results include calcium elevation to 12.2 mg/dL with normal alkaline phosphatase. A dipstick urinalysis shows proteinuria. A bone marrow aspirate was dry. The bone marrow biopsy is pending. Plain film X-rays of the chest show a fracture of the right seventh rib and compression of the eighth vertebra. What is the most likely diagnosis?

2+ reflexes in upper and lower extremities Scattered cherry angiomas Respiratory rate of 18 Grade II systolic murmur Blood pressure of 138/102 Blood pressure of 138/102

A 65-year-old woman presents with weight gain, fatigue, and aching limbs that have been bothering her for 2 months. She has also been having intermittent runny noses, constipation, and dry skin. Physical exam reveals heart rate 56, respiratory rate 18, blood pressure 138/102, temperature 97.9°F, and weight 210 lb. On heart exam, there is a grade II systolic murmur heard best at the right sternal border, second intercostal space. Skin exam shows cool, dry, somewhat pale skin with scattered cherry angiomas. What physical exam finding is caused by this patient's most likely diagnosis?

Folic acid deficiency

A 66-year-old male patient presents with a 2-month history of fatigue. He reports that he has recently joined a support group to help with alcohol use disorder. On examination, he is malnourished and pale. His physical exam, including neurological evaluation, is unremarkable. Peripheral blood smear reveals macrocytic red cells. What is the most likely diagnosis?

Omeprazole - GERD is the diagnoisis

A 66-year-old male patient with a history of obesity and hyperlipidemia presents with an 8-month history of progressing "burning in his chest." This sensation is noted in the midline of the chest, and it is provoked when bending over, when wearing tight clothing, after eating a large meal, and when lying supine. He denies chest pressure, cough, shortness of breath, palpitations, dizziness, lightheadedness, and diaphoresis. Physical examination is unremarkable. An upper endoscopy is performed and inflammation in the esophagus is noted. Question What is the most appropriate pharmacotherapeutic intervention for this patient?

Acetaminophen Aspirin Ibuprofen Metformin Prednisone Prednisone

A 66-year-old man presents with monoarticular arthritis. He has stage 3 chronic kidney disease. His affected joint is warm to the touch. You suspect gout. He is allergic to aspirin. What anti-inflammatory should you prescribe? -no aspirin -no ibuprofen

Work toward resuming former exercise program

A 66-year-old woman presents for a health maintenance visit. She reports no issues and has no history of chronic illness except postmenopausal osteoporosis, with a bone mineral density >3 standard deviations below the mean. She takes supplemental calcium and is being treated with denosumab IM every 6 months. Her only recent fracture was a compression fracture at T-6, diagnosed 5 months ago. Although she had previously enjoyed tennis, dancing, and gardening, she has drastically reduced her activity for fear of suffering further fractures. She asks if she should resume any sort of regular physical activity. Past medical history is otherwise unremarkable except for anemia, which is now resolved. Vital signs are normal, as is the remainder of the examination. What advice would you give?

Osteoporosis

A 66-year-old woman presents with back pain. She has a 5-month history of taking prednisone and has been a smoker for the last 15 years, which she has difficulty quitting. Upon physical exam, her back is very tender upon palpation. When the patient walks, she appears very bent over and unable to stand upright. A laboratory finding on her DXA was a T score of -3.1. X-ray was conducted as well, and demonstrates a small fracture in her back. What is the most likely diagnosis?

Chronic renal disease

A 67-year-old man presents with a subacute onset of lower urinary tract symptoms. He cannot discuss his past medical history or current medications. An initial genitourinary workup is started, and microscopic urinalysis reveals granular and waxy casts. Question What condition are these results most closely associated with?

Quantitative cultures of urine Renal ultrasound Blood culture x 3 Plain abdominal radiograph Ultrasound scan of the abdomen Quantitative cultures of urine

A 67-year-old man presents with severe flank pain, fever, postural dizziness, dysuria, inability to pass urine for 1 day, and mild confusion. He has experienced two episodes of urine retention and incontinence over the previous 2 months, the most recent of which required urethral catheterization and outpatient antibiotic treatment for E. coli. His past medical history is significant for hypertension, prostatic hypertrophy, non-insulin dependent diabetes mellitus, and cognitive impairment that requires minimal assistance in activities of daily living. His temperature is 102.02°F (38.9°C); BP 80/50 mm Hg; pulse rate 114/minute; respiratory rate 43 breaths/minute. Physical examination reveals a tender distended bladder that drained 2700 mL of turbid urine. What will be the next diagnostic step in this condition?

Long head of the biceps brachii

A 67-year-old man presents with the complaint of a "tender bulge in his right dominant arm" following an attempt at lifting his couch 2 days ago. He admits to having mild chronic shoulder pain and taking ibuprofen (Motrin) for relief. While attempting to lift the couch, he felt a snap in his shoulder and noticed a bulging in his anterior arm. What tendon rupture has most likely occurred?

Syndrome of inappropriate ADH secretion Central diabetes insipidus Diabetes mellitus Nephrogenic diabetes insipidus Psychogenic polydipsia Syndrome of inappropriate ADH secretion

A 67-year-old woman is hospitalized because of lung cancer. During her stay in the hospital, she becomes obtunded. During a physical exam, she has a seizure. Soon thereafter, she goes into a coma. Lab results show that her urine is hypertonic. Additional laboratory results are as follows: What is the most likely diagnosis?

Spinal stenosis Osteoarthritis Polymyalgia rheumatica Scoliosis Septic arthritis Spinal stenosis (leaning forward helps)

A 68-year-old man presents due to lower back pain. The patient reports the pain has been present for the past 3 months and seems to be getting worse. Upon physical exam, the patient appears to get relief of pain when bending forward. Upon standing and extension of the lumbar spine, the patient reports pain. He denies decreased range of motion in the shoulders, neck, and hips. The spine is not in an S or C shape. T2 weighted imaging shows disc degeneration. X-rays show symmetrical joint spaces. C-reactive protein (<1.0 mg/L) and ESR (<40 mm/hr) are in normal range. Gram stain is negative for Staphylococcus aureus. What is the most likely diagnosis?

Limited range of joint motion Numbness and tingling Positive McMurray's sign Polyarticular joint pain Erythema migrans rash Limited range of joint motion (septic joint)

An 82-year-old man with rheumatoid arthritis, HIV, and diabetes mellitus presents with fever, severe pain and swelling to his right knee. He denies any history of trauma, injuries or accidents, sexual activity, or prior episodes. On exam, right knee is erythematous with increased warmth. A joint aspiration was done which reveals opaque yellow synovial fluid with 55,000 WBCs/microL. What is the most likely additional manifestation in this patient?

Non-ketotic hyperglycemic hyperosmolar coma Diabetic ketoacidosis Uremic acidosis Metformin-induced lactic acidosis Metformin-induced lactic acidosis

A 68-year-old man presents with acute chest pain. He has a 20-year history of diabetes mellitus and 10 years of hypertension. He has no history of coronary artery disease, but he has hyperlipidemia. His medications include atenolol 50 mg daily, metformin 1000 mg twice daily, aspirin 80 mg daily, and simvastatin 40 mg daily. On arrival, he has a temperature of 99°F, a pulse of 106 bpm, BP 100/60 mm Hg, and a respiratory rate of 26/min. His lungs have fine crackles bilaterally, heart sounds are heard with tachycardia, and abdomen is essentially normal. The patient is diagnosed with acute myocardial infarction. What is the metabolic abnormality in this patient?

Infuse dopamine in small doses of 1-5 mcg/kg/min. Begin high dose intravenous furosemide. Increase dietary intake of salt, water, and potassium. Perform a red blood cell transfusion. Increase the strength of enalapril and begin mannitol Perform a red blood cell transfusion

A 68-year-old man with a past medical history of congestive heart failure, hypertension, and hyperlipidemia has been admitted to the hospital for the evaluation of anemia due to a chronic gastrointestinal bleed. He takes oral enalapril and furosemide for CHF. While hospitalized, he developed polydipsia, dizziness, and decreased urine output; he notes that his urine is concentrated. His physical exam reveals orthostatic hypotension, poor skin turgor, dry mucous membranes, tachycardia, and peripheral edema. His bloodwork was remarkable for a hemoglobin of 7.0, hematocrit of 30, and serum sodium of 149. His BUN to creatinine ratio was 42 to 1, while the fractional excretion of sodium (FENa) was <1% and fractional excretion of urea (FEUrea) <35%. What is the next step in the management of this patient?

Stress Urge Overflow Functional Urge --> she feel the need to void

A 69-year-old woman presents with a 3-month history of intermittent urinary incontinence. After further questioning, she reveals that she experiences leakage after having an intense need to void. This is an example of what type of incontinence?

Right anterior cerebral artery Left anterior cerebral artery Right middle cerebral artery Left middle cerebral artery Right posterior cerebral artery Left middle cerebral artery (right eyes issue paralysis) - middle

A 69-year-old woman suffered a massive stroke 6 weeks ago. She is now recovering from the stroke, but she has residual paralysis and sensory impairment of her right arm. She is also unable to speak and unable to turn her eyes to the right. The most likely site of her lesion is in the area that is supplied by what artery?

Discharge after 4-hour observation if asymptomatic. Discharge with instructions for post-head injury precautions. Give 80 cc of 0.9% normal saline (20 mL/kg). Give intravenous mannitol to lower intracranial pressure. Obtain skeletal survey to rule out abuse Obtain skeletal survey to rule out abuse.

A 7-week-old patient presents with a 12-hour history of lethargy. Physical exam reveals an infant who cries and becomes irritable when examined. There is edema noted on the left side of the head. There is no papilledema, and the mother denies a history of the patient vomiting. There is a single faint bruise on the upper lip. A CT of the head reveals a linear skull fracture of the left parietal bone; there is no evidence of intracranial injury. On further questioning, the mother states, "My baby rolled over from the sofa onto a carpeted floor 2 days ago." What is the most appropriate management?

Splenectomy Ferrous sulfate Antibiotics Bone marrow transplant Intravenous hydration Intravenous hydration

A 7-year-old African American girl presents with a 6-hour history of severe pain in her hands and feet that started spontaneously. She has been having bouts of excruciating pain since she was 6 months old. She is an only child, and her parents have recently discovered that they are both carriers of her condition. On examination, her HR is 115 bpm and blood pressure is 90/50 mm Hg; she is also jaundiced. She has a mildly enlarged spleen but no source of infection. Laboratory investigations reveal hemoglobin of 8 g/dL. What treatment would be most appropriate?

Advise the parents to hold the child back one year in school. Contact child protective services for suspected abuse. Initiate medication. Order head magnetic resonance imaging (MRI). Refer for counseling Initiate medication (ADHD)

A 7-year-old boy presents for evaluation of behavior problems in school that have been occurring for over 6 months. The boy's teacher recommended he be evaluated. The teacher reports that he tests at grade level, but he seems to make careless mistakes on schoolwork and has trouble maintaining attention to instruction. He does not finish his homework, and he often loses his homework, pencils, and books. The boy is often seen fidgeting at his desk. He blurts out answers and has difficulty waiting his turn. The parents tell you that they have seen similar traits at home, such as forgetting to do daily activities. For the past several years, he has been easily distracted. They view him as a happy, bright boy, and they report that he is very active. He has had normal vision and hearing screenings. The father self-reports that he was very similar as a child and still struggles with focus and concentration as an adult, but he has never received any help. The parents deny any major changes in the family situation. They would like medical help to improve their son's performance in school. The boy has been seen regularly for his well-child exams, has always met milestones, and has had normal exams. Today, he is quickly moving about the exam room. He looks at a book for a few moments, and then he looks out the window for a short time before interrupting his parents. What is the most appropriate intervention in this case?

Amitriptyline Bupropion Clonazepam Methylphenidate Sertraline Methylphenidate

A 7-year-old boy presents with his parents for evaluation of behavior problems in school; the problems have been ongoing for over 6 months. The boy's teacher recommended he be evaluated. The teacher reports that he tests at grade level, but he seems to make careless mistakes on schoolwork and has trouble maintaining attention to instruction; he does not finish his homework, and he often loses his homework, pencils, and books. The boy is seen often fidgeting at his desk; he blurts out answers and has difficulty waiting his turn. The parents agree that they have seen similar traits at home for several years, such as forgetting to do daily activities and being easily distracted. They view him as a bright happy boy who is very active. He has had normal vision and hearing screenings. The father reports that he was very similar as a child and still struggles with focus and concentration as an adult, but he never received any help. The parents deny any major changes in the family situation. They would like medical help to improve their son's performance in school. The boy has been seen regularly for his well-child exams and has always met milestones and had normal exams. Today, he is quickly moving about the exam room; he looks at a book for a few moments, and he then looks to the window for a short while before interrupting his parents. In addition to behavioral intervention, and assuming the parents agree to medication for their son, what medication is the most appropriate initial treatment?

Acute cystitis Analgesic abuse nephropathy Diabetic nephropathy Postinfectious glomerulonephritis Renal cell carcinoma Postinfectious glomerulonephritis

A 7-year-old female patient presents with a 1-day history of bloody urine. The parents note that the patient had a fever for a few days with a sore throat. She was given acetaminophen and about 3 days of some leftover amoxicillin; the fever and pharyngitis resolved. On physical exam, she appears interactive and in no apparent distress. She is well-nourished, non-obese, and perhaps mildly edematous. The abdomen is non-tender and non-distended. Temperature 99.0°F, BP 138/85 mm Hg, pulse 98 bpm, respiratory rate 20/min. Urinalysis is performed: Protein2+ GlucoseNegative KetonesNegative Blood4+ NitritesNegative Leukocyte esterase1+ Microscopic analysis confirms the presence of RBC casts and dysmorphic red blood cells (RBCs). Blood tests reveal elevated creatinine, elevated antistreptolysin O, and low serum complement. What is the most likely diagnosis for this patient?

Lithium

A 70-year-old female patient presents after collapsing at work. History includes a mood disorder that involves wild mood swings and reckless behavior. This mood disorder was diagnosed 1 year ago, and the patient has been taking the prescribed medication. She was recently hospitalized for acute gastroenteritis complicated by acute renal failure. Shortly afterward, she reports experiencing nausea, vomiting, fatigue, tremor, and hyperreflexia. Lab results show an elevation in BUN and creatinine and elevated serum drug levels, but the results are otherwise unremarkable. What drug is most likely responsible for her symptoms?

Non-contrast head CT scan Monitoring of intraventricular intracranial pressure Parenteral cefazolin MRI of the brain Lumbar puncture MRI of the brain

A 70-year-old man with hypertension, diabetes, obesity, and coronary artery disease presents with a 3-day history of fever, headache, nausea, vomiting, lethargy, and myalgias. His wife states that the symptoms began the day after a summer evening walk around a lake. Today, his condition has worsened; he has developed a diminished level of consciousness, behavioral changes, and abnormal movements. His physical exam reveals a lethargic man with photophobia, cranial nerve deficiencies, neck stiffness, and abnormal extremity movements. Based on your suspected diagnosis, what is the next step in the management of this patient?

Divalproex Haloperidol Risperidone Trazodone Trifluoperazine Divalproex

A 70-year-old patient presents to be evaluated for bizarre behavior. The daughter arrives with the patient and speaks with the provider alone; she describes the patient's behavior as consisting of mood swings, lavish trips, spending foolishly, and staying up at night. According to the daughter, bipolar disorder has been diagnosed in the patient in the past. Past medical history includes obesity, diabetes mellitus, and hypertension. What drug is most appropriate as chronic therapy for this patient?

Delusional disorder Schizophrenia Schizotypal personality disorder Substance-induced psychotic disorder Psychotic disorder due to a medical condition Schizophrenia

A 70-year-old woman has been refusing to leave her room at the nursing home facility where she resides. She says that people are following her, and she even refuses to go out with her daughter. She has a long history of mental illness; her ex-husband had her committed to a state hospital, where she had resided for over 30 years. On interview, it is difficult to obtain a history; her thinking is disordered and her speech is erratic. When asked why he committed her, she says that she believes her husband was trying to kill her. What is the most likely diagnosis?

Rotator cuff tear Impingement syndrome Adhesive capsulitis Glenohumeral dislocation Proximal humerus fracture Rotator cuff tear (elderly report a prodrome of chronic shoulder pain)

A 73-year-old man has had a history of pain in that shoulder for over 6 months that kept him from sleeping on the left side; the pain would wake him often. He presents with an inability to actively raise his left non-dominant arm, which began a month ago after his shoulder pain improved. There was no specific injury he can recall, but he felt a pop a month ago while taking out the trash. On inspection, you notice the back of the shoulder appears sunken compared to the other shoulder. What is the most likely diagnosis?

Increased cerebrospinal fluid pressure Urinary tract infection Frontal lobe dysfunction Normal pressure hydrocephalus Pelvic sphincter weakness Normal pressure hydrocephalus (wet, walbably, and wacky)

A 70-year-old woman is brought to your attention because of a slowly progressive gait disorder. She has also been experiencing forgetfulness and problems with micturition. About 1 year ago, she started having weakness and tiredness in her legs, followed by unsteadiness; her steps became shorter and shorter. She occasionally forgets where she put things and she has forgotten to turn off the oven on 2 occasions. Over the last month, she has started experiencing urinary urgency, the need for frequent urination, and the involuntary leaking of urine. She is worried about these symptoms. What is the most likely cause of her urinary problems?

Antimuscarinic drug (Tolterodine) Antibiotic (Sulfamethoxazole/trimethoprim) Acetylcholinesterase inhibitor (Donepezil) Ventriculoperitoneal shunt Kegel exercises Ventriculoperitoneal shunt

A 70-year-old woman is brought to your attention by her family because of the slowly progressive gait disorder, the impairment of mental function, and urinary incontinence. About 1 year ago, she started having weakness and tiredness in her legs, followed by unsteadiness; her steps became shorter and shorter, and she also experienced unexplained backward falls. She is becoming emotionally indifferent, inattentive, and her actions and thinking have become "dull." Over the past month, she has started having urinary urgency and involuntary leaking of urine. Besides multivitamins and local application of Timolol for glaucoma, she takes no other medications; there are no other symptoms. What is most likely the best method of treating the patient's urinary problems?

Bladder cancer

A 71-year-old Caucasian man has been "peeing red" for 3 days and presents after being urged by his spouse. He is not having pain with urination. Pertinent history includes a 100 pack-year smoking history, quitting 5 years ago after having a successful heart catheterization. He has a history of benign prostatic hyperplasia, for which he is taking tamsulosin 0.4 mg once daily. The patient denies any acute trauma, injury, urinary frequency, or urgency. What is the most likely diagnosis?

Serum creatinine test Prostate-specific antigen test Urine culture test Post-void residual urine test Blood urea nitrogen test Post-void residual urine test

A 71-year-old male farmer presents because he is experiencing a gradual increase of frequency of urination over the previous 4-5 months. He has difficulties starting to urinate, the stream is slow, and he frequently has a sensation of incomplete emptying. He has not had a physical examination in several years. He has no history of sexually transmitted diseases (STDs) or urinary tract infections. Upon examination, his abdomen and his prostate are non-tender. His prostate seems moderately enlarged (estimated at 35 g) but smooth and symmetrical. What diagnostic test do you recommend as the next step in this patient's management?

Change ibuprofen to hydrocodone. Initiate IV fluids. Decrease or stop levothyroxine. Increase levothyroxine. Increase oxybutynin Initiate IV fluids (hyptension, dizziness, dehydrated --> need fluids)

A 71-year-old woman presents with her daughter for dizziness. The daughter is worried about a potential fall; her mother recently had surgery for a right ankle fracture. The patient denies syncope but feels lightheaded when trying to stand. She lives alone and is in a walking boot with crutches. She has not had to use her narcotic pain medicine for 5 days, stating her ankle pain is controlled with ibuprofen alone. She is limiting fluid intake to minimize bathroom trips. Past medical history is remarkable for stable overactive bladder and hypothyroidism. Medications: oxybutynin, ibuprofen, levothyroxine. Only surgery is the recent ankle fracture repair. She denies allergies and use of tobacco, alcohol, and recreational drugs. On physical exam, the patient is wearing the boot and reports dizziness as she moves to the exam table. Exam is remarkable for mild tachycardia and decreased skin turgor, remainder is normal. BP 94/58, Pulse 110, Wt 117 lb, Ht 63". Large chart Dark urine incrase osmolaility What is the most appropriate next step in the management of her care?

Antibiotic therapy Steroids Supportive care Transfusion Antiviral treatment Antibiotic therapy (Bacterial) (Increased opening pressure and low glucose level)

A 72-year-old man presents with low-grade fever, nausea, confusion, and lethargy. His past medical history is significant for hypertension, hypercholesterolemia, and diabetes. He had sinusitis approximately 1 week ago; otherwise, he has been healthy. Laboratory workup shows a CSF with elevated opening pressure and low blood glucose levels. A Gram stain on the CSF shows gram-positive cocci. CBC shows an elevation of PMNs, but it is otherwise normal. What is the most appropriate treatment in this case?

Hyperglycemic hyperosmolar state

A 72-year-old woman presents with a 2-week history of fever, cough, and excessive diuresis. The woman has diabetes mellitus that is being treated with glimepiride (Amaryl). Her fluid and food intake have been poor during this time, as well. On physical examination, blood pressure is 98/58 mm Hg, pulse is 112/min, temperature is 100.6°F, and respirations are shallow and regular at 20/minute. On physical assessment, the patient is stuporous, skin and mucous membranes are dry, heart has a regular rate and rhythm without murmurs, and auscultation reveals rales in the left lung base. Her serum BG level is 602 and blood pH 7.35. She is diagnosed with pneumonia. Based on this information, what is this patient's most likely concomitant condition?

Increase the dose of prednisone to reduce symptomatology. Order human osteocalcin and bone alkaline phosphatase levels. Perform a quantitative computerized tomography of the spine. Prescribe estrogen replacement therapy. Order a dual-energy X-ray absorptiometry test Order a dual-energy X-ray absorptiometry test

A 72-year-old woman with a past medical history of COPD, deep venous thrombosis, breast cancer in remission, hyperthyroidism, and premature ovarian failure presents with sudden-onset severe lower back pain of 2 hours' duration. She states that the pain began when stepping onto the floor as she got out of bed. She denies any falls, prior injuries, genitourinary problems, or lower extremity numbness, tingling, paresthesias, or weakness. She takes a daily prednisone tablet for COPD. Her social history is notable for a sedentary lifestyle and a 40 pack-year smoking history. Her physical exam reveals a tall thin-framed woman with noticeable lid lag. Her thoracic vertebral body is exquisitely tender at T12, and she has limited mobility of all thoracolumbar planes of motion. Lateral and AP lumbar spine X-ray reveals osteopenia and collapse of vertebral body of T12. What is the best next step in managing this patient?

Ecchymosis over the deltoid Normal strength of all shoulder muscles Negative (-) painful arc sign Positive (+) drop arm test Paresthesia of the median, radial, and ulnar nerves Positive (+) drop arm test

A 73-year-old man has had shoulder pain for more than 6 months and it has kept him from sleeping on his left side and has often woken him up. He presents with an inability to actively raise his left arm that started 1 month ago, before which his shoulder pain had improved. There was no specific injury that he can recall, but he felt a pop in the shoulder a month ago while taking out the trash. What finding do you expect on physical exam?

Immediate subacromial steroid injection Immediate surgical repair Order CT of the left shoulder Refer for physical therapy Place in shoulder immobilizer Refer for physical therapy

A 73-year-old man presents with the inability to actively raise his left non-dominant arm to retrieve plates from the kitchen cabinet. He further describes the inability to retrieve any objects with his left hand/arm because of the limited range of motion. He is worried because he is the only driver in his household, and he does not want to lose his driving privileges. He has a longstanding history of chronic shoulder impingement syndrome. On exam, he is found to have a (+) positive drop arm test. What initial treatment is recommended for the most likely diagnosis?

Rectal cancer Prostatitis Acute colitis Rectal abscess Fecal impaction Fecal impaction

A 73-year-old man with hypothyroidism has been hospitalized with a spinal cord injury and is evaluated at bedside. He is unable to have bowel movements; he has only passed a stool twice in the past 10 days. He states that he has had involuntary passage of small loose or liquid stools during this time, associated with abdominopelvic discomfort. Dietary history is remarkable for a low-fiber diet that lacks raw fruits and vegetables. He takes oxycodone for chronic back pain. He denies any abdominal or pelvic pain, weight loss, hematochezia, melena, fever, chills, or urinary issues. Rectal exam reveals good sphincter tone, but a firm immovable mass is detected. Bedside pelvic radiograph shows colonic distension. What is the most likely diagnosis?

CT without contrast Electromyogram Ultrasound MRI Plain films MRI

A 73-year-old patient cannot actively raise the left non-dominant arm to reach the kitchen cabinet. This began 1 month ago after shoulder pain improved. The patient had a history of pain in that shoulder for over 6 months that kept them from sleeping on the left side, and the pain would often wake them. There was no specific injury they can recall, but the patient felt a pop 1 month ago while taking out the trash. What is the diagnostic study of choice if surgery is indicated?

Lithium Bromine Aspirin Acetazolamide Topiramate ASA

A 73-year-old patient presents to the emergency department after a suspected overdose. History is unobtainable; the patient is currently febrile. Blood pressure 125/76 mm Hg, respiratory rate 30 with increased breathing depth, heart rate 105. What drug did they take?

Mild cognitive impairment Intellectual disability Delirium Post-traumatic stress disorder Nyctalo Mild cognitive impairment

A 74-year-old man presents to discuss difficulty with orientation, memory, and word-finding. The orientation problems are worse at night. His problems began with mood and behavioral changes 7-8 years ago, after his brother passed away. He reports no trouble with activities of daily living. The patient has only an elementary school education, stating that he was "not smart enough to go to college." There is no family history of dementia. What is the most likely diagnosis?

Advise maintenance of current body weight. Apply topical estrogen cream. Perform anterior colporrhaphy. Recommend pelvic muscle exercises. Refer for dynamic MRI defecography Recommend pelvic muscle exercises

A 74-year-old multiparous woman with a history of breast cancer has been referred to a gynecologist due to a sensation of vaginal fullness and pressure, sacral back pain with standing, coital difficulty, lower abdominal discomfort, and urinary frequency and incontinence. She states that she feels "a bulge" in the lower frontal vaginal area. She denies any fever, chills, flank pain, history of infectious diseases, changes in weight, dysuria, or hematuria. The physical exam is remarkable for an obese body habitus and non-tender bulge located anterior within the vaginal introitus that becomes more pronounced upon Valsalva and standing. What is the most appropriate next step in the management of this patient?

Left optic tract Optic chiasm Left optic nerve Left parietal lobe Right optic tract Left parietal lobe

A 74-year-old woman presents for management of an ischemic stroke. She reports difficulty seeing objects on her right side. You perform confrontational visual field testing as part of your neurological examination and you discover she has a right inferior homonymous quadrantanopsia. This lesion localizes to what part of the body?

Add a diuretic to the patient's medication regimen. Add sodium polystyrene sulfonate sorbitol once daily. Discontinue lisinopril and initiate diuretic therapy. Instruct the patient to discontinue ibuprofen use. Instruct the patient to take 1 tsp of baking soda by mouth once daily Add a diuretic to the patient's medication regimen

A 75-year-old female patient presents for follow-up of osteoarthritis, diet-controlled type 2 diabetes, and hypertension. She has been taking lisinopril for 15 years. She takes ibuprofen about once a month for arthritis pain, but she has not used it recently. There are no new presenting problems, and physical exam is unremarkable. Blood pressure 147/90 mm Hg. Last urine albumin to creatinine ratio was 180 mg/g. In review of lab results, serum potassium is 5.8 mEq/dL and creatinine is 1.2 mg/dL. What is the most appropriate next step in management?

Discontinue lisinopril and initiate diuretic therapy. Advise the patient to take 1 tsp of baking soda by mouth once daily. Advise the patient to discontinue ibuprofen use. Add sodium polystyrene sulfonate sorbitol once daily. Add a diuretic to the patient's medication regimen Add a diuretic to the patient's medication regimen

A 75-year-old female patient presents for follow-up of osteoarthritis, diet-controlled type 2 diabetes, and hypertension. She has been taking lisinopril for 15 years. She takes ibuprofen about once a month for arthritis pain, but she has not used it recently. There are no new presenting problems, and physical exam is unremarkable. Blood pressure 147/90 mm Hg. Last urine albumin to creatinine ratio was 180 mg/g. In review of lab results, serum potassium is 5.8 mEq/dL and creatinine is 1.2 mg/dL. What is the most appropriate next step in management?

Foul urinary odor Lower back pain Lower extremity sensory loss Presence of a sinus tract Bowel incontinence Lower back pain (he has laminectomy = there will be discomfort)

A 75-year-old man presents to his pain management office for follow-up after a lumbar laminectomy 2 weeks ago. He states that he has had a fever for the past 6 days. He denies any headache, chest pain, shortness of breath, cough, abdominal pain, nausea, diarrhea, vomiting, or extremity pain. He has a past medical history of hypertension, hyperlipidemia, chronic lumbar stenosis (now status post laminectomy), and type 2 diabetes mellitus. He also has a history of intravenous drug use. He was referred for an MRI of the spine, which revealed destruction of the vertebral body of L4 with significant collapse. What is the most likely additional clinical manifestation in this patient?

Heparin Dobutamine Nicardipine Pyrimethamine Methotrexate Nicardipine (intracerebral hemorrhage)

A 75-year-old man with a past medical history significant for severe and uncontrolled hypertension is brought into the emergency room by his family due to a 30-minute history of change of mental status. The patient had just climbed stairs when he first developed a headache that has become progressive; it is associated with nausea, non-bilious vomiting, and unilateral upper and lower extremity numbness. Physical exam reveals an alert and oriented times zero patient. He is afebrile, with blood pressure 185/108 mm Hg, and there is nuchal rigidity. Neurological exam notes contralateral sensory loss, contralateral hemiparesis, gaze paresis, homonymous hemianopia, and miosis. A stat CT scan without contrast shows evidence of intracerebral hemorrhage. What medication would be most appropriate in this patient?

Paget's disease

A 75-year-old woman presents due to intense pruritis of the vulva and occasional bleeding. She is unsure if the bleeding is caused by her scratching. She has tried some OTC preparations to alleviate the itching without relief. She denies vaginal discharge or dysuria. On physical examination, you notice excoriations and some scattered lesions that look like eczema on the vulva; they do not scrape off. Also noted was inguinal lymphadenopathy. You decide to do a punch biopsy. The pathology report reveals large eosinophilic cells. What is the most likely diagnosis?

Gastric biopsy Fecal antigen test Urea breath test Culture of H. pylori Schilling test Urea breath test

A 75-year-old woman presents with heartburn and dyspepsia. She was diagnosed with osteoarthritis 4 years ago. For the past 18 months, she has been managing pain with naproxen. The gastroenterologist suggests that the patient be tested for Helicobacter pylori infection. What is the most sensitive and specific non-invasive method to diagnose this infection?

Elevated erythrocyte sedimentation rate Juxta-articular demineralization on x-ray Negatively birefringent crystals Plain radiograph subchondral sclerosis Synovial fluid leukocytosis Plain radiograph subchondral sclerosis (osteoarthritis)

A 75-year-old woman with a past medical history of hypertension, hyperlipidemia, and obesity presents with insidious but progressive bilateral knee pain for the past several months. She states that her pain is worsened with movement and relieved by rest. She reports "cracking" sounds in her knees upon ambulation and minimal stiffness lasting 10 minutes. Her physical exam is remarkable for bilateral knee crepitus, joint line tenderness, and an antalgic gait; there is no swelling, erythema, or warmth noted. What is an expected diagnostic test result in this patient?

Docetaxel Ketoconazole Leuprolide Proscar Rapaflo Leuprolide

A 76-year-old man with a past medical history of diabetes, hypertension, and hyperlipidemia is referred to a urologist for the evaluation of supranormal PSA readings associated with a reduction in urinary stream, back and hip pain, hematuria, and weight loss. He has not yet been managed for these symptoms. An in-office pelvic x-ray done reveals osteoblastic sclerotic areas of the pelvis and femurs bilaterally. What is the most appropriate pharmacotherapeutic agent for this patient?

EEG Stanford-Binet Intelligence Test Mini-mental status examination MRI of the brain TSH and T4 Mini-mental status examination (dementia test)

A 76-year-old woman presents because her children are concerned that she might have dementia. She states that she is doing reasonably well, except that she sometimes sleeps less deeply and wakes up more often than she did several years ago. According to her children, she is slower than before, and her memory has been getting worse over the last 3 years; she has difficulties recalling the specific date of an event (although she can describe the event itself). She also has a great deal of trouble with names, but she can easily recognize people. She always says, "It is on the tip of my tongue, but..." Aside from hypertension that is under control, she does not have any other health problems. She has been a widow for about 10 years. Her older brother was diagnosed with dementia. Physical examination today is within normal limits for the age, and neurological examination is non-focal. What should be your next diagnostic step?

Functional incontinence Diabetic cystopathy Overactive bladder Renal insufficiency Normotensive hydrocephalus Functional incontinence (been incontinent of urine) aware of need to urinate

A 77-year-old right-handed woman presents with a history of right-side hemiparesis and global aphasia. She has long-standing diabetes, for which she takes metformin; she also has a history of well-controlled hypertension. CT shows large left hemispheric infarction. On examination, she is alert, and you notice that she appears queasy. She has been incontinent of urine since admission. Serum analysis shows elevated glucose. Glomerular filtration rate shows mild renal insufficiency; dipstick urinalysis is glucose positive, and post-void residual volume is 80 mL. What is the most likely diagnosis?

Weight loss Low frustration tolerance Racing heart Stable mood Headache Low frustration tolerance (OCD)

A 9-year-old boy presents with episodes of severe impulsivity, lack of attention, poor listening skills, and obsessive and compulsive characteristics; symptoms have been evident to his mother and multiple teachers at school. There have been numerous episodes. The patient has had mild evidence of these signs since he was in kindergarten, but they have become substantially worse in the last 4 months. Considering the differential diagnosis, what is an associated feature that would support the most likely diagnosis?

Promethazine Metoclopramide Diclofenac Sumatriptan Cyclizine Sumatriptan

A consulting physician asks for a follow-up on a 35-year-old woman being treated for an acute migraine headache. Upon interview, she states that the physician gave her an injection about 20 minutes ago; she now has significant chest pain. What medication did the patient most likely receive?

Administration of growth hormone Cognitive behavioral therapy Dietary control Oxygen therapy Surgery Dietary control

A family friend calls you for medical advice. She just gave birth to her third child 1 week ago. She is worried after a nurse from the hospital called her. The nurse said the baby tested high on a newborn screening for phenylalanine and that she would set up an appointment at a specialty center for the family. Your friend could not remember the name of the disorder, but she was told that it could prevent the infant from normal neurological development if left untreated. The friend reports that her pregnancy was uncomplicated and that the infant appears healthy so far. No one in the infant's immediate family has any health problems. What will most likely be recommended treatment for the newborn's condition?

Rubella Aspirin Herbal tea Gestational diabetes Folic acid Gestational diabetes

A full-term female newborn has facial defects affecting the eyes, nose, and upper lips. She is the first child of non-consanguineous parents. The mother has a history of gestational diabetes, which began at the start of pregnancy. She contracted German measles a month before delivery. During the course of infection, her self-prescribed daily treatment was 3 tablets of aspirin, at least 6 cups of herbal tea, and a double dose of folic acid. Imaging studies showed prosencephaly. What risk factor is responsible for the condition of this newborn?

Glucose-6-phosphate dehydrogenase deficiency

A male neonate born 36 hours ago presents with a yellowish coloration of the whites of the eyes. The skin appears darker and more yellowish than the twin sister's skin. Pregnancy was unremarkable, and the 23-year-old mother had no infections or complications and took no medications during the pregnancy. Delivery was uneventful; the neonates were born on term with APGAR scores of 9 and 10. Peripheral smear reveals poikilocytes and red blood cells that have a "bitten" appearance. Bilirubin levels in the patient are high (13 mg/dL), with direct bilirubin 1 mg/dL. Coombs test is negative, and hemoglobin is low. What is the most likely diagnosis?

Klinefelter syndrome (XXY)

A mother brings her 16-year-old son to your medical office for a comprehensive history and physical examination. She tells you she is concerned about his immature physical development and insecure behavior. She thinks these characteristics are markedly different from her other children. His IQ is 70, and he is in special education for a language-based learning disability. On physical examination, he is tall and thin; he has sparse body hair and a high-pitched voice. Heart, lungs, abdomen, and neurologic exam are unremarkable. Pertinent positive findings include disproportionately long arms and legs, gynecomastia, as well as small testes and phallus. What is the most likely diagnosis?

Epispadias Cleft scrotum Cryptorchism Hypospadias Bifid penis Hypospadias

A mother brought her 2-month-old infant son to a pediatric clinic because, during micturition, urine ran from the opening at the bottom of the midline groove of the scrotum instead of from the tip of the penis. What is the most likely diagnosis?

Plan voiding cystourethrogram. Order antibiotics prophylaxis. Arrange circumcision. Address dysfunctional voiding. Perform radionucleotide cystography Plan voiding cystourethrogram

A mother presents her 2-year-old son for a follow-up of a febrile urinary tract infection (UTI). He just finished a 10-day course of trimethoprim/sulfamethoxazole (Bactrim) and is asymptomatic. He had a UTI with fever at age 15 months. His mother is concerned because is not toilet trained. Physical examination is unremarkable. He is not circumcised. Urinalysis and renal ultrasound findings are normal. What is the next step in the evaluation of this patient?

Gender dysphoria

A mother seeks medical attention for her 7-year-old son. For the last 8 months, he has not acted like his three older brothers. The mother indicates that her son has said on repeated occasions he wants to get rid of his male genitalia and he would prefer to be a girl. She has found him wearing his sister's clothing on numerous occasions. More history shows that he prefers to play with dolls and only spends time with female friends. Teachers relate that he turns down invitations from the boys in the class to join in sports activities. His male classmates are now teasing and embarrassing him in class, and it has begun to affect his schoolwork. What is the child's most likely diagnosis?

Petit mal (absence) seizures

A parent brings in their 6-year-old child whose teacher is concerned about episodes of abnormal behavior at school. The teacher has noticed the patient "staring off into space" frequently throughout the day; the teacher cannot get the patient's attention while staring. After the end of the "staring episode," the patient continues to behave as if nothing happened and they are not aware of their own behavior. The parent states that there are times when the patient does not seem to be paying attention to what the parent is saying. Shortly after these episodes, the patient engages in conversation without any problem, so the mother did not think the episodes were an issue. There is no concern about other abnormal behavior or discipline issues at home or school. What is the most likely diagnosis?

Left hemisection of spinal cord Bilateral posterior column lesion Anterior spinal artery lesion Syringomyelia in cervical spinal cord Right hemisection of spinal cord Right hemisection of spinal cord (Brown-Sequards syndrome)

A patient presents with a motor deficit on the right side of the body, decreased sensation and pain on the left side of the body, and diminished vibratory and position sense on the right side of the body. What type of spinal cord lesion is present?

Major depressive disorder

A patient presents with deteriorating work and feelings of worthlessness and hopelessness; symptoms have been worsening over the past month. The patient also gives history of excessive fatigue and loss of interest in pleasurable activities. They also have trouble eating and sleeping, and they are increasingly withdrawing from family and friends. These symptoms have been present for more than 2 months. What is the most likely diagnosis?

Extrauterine pregnancy Acute heart failure Acute liver failure Intraabdominal hemorrhage Ovarian fibroma Ovarian fibroma ( Ascities, pleural effusion, right flank pain)

A previously healthy 30-year-old woman G1P1 presents with amenorrhea, weight loss, shortness of breath, and increasing abdominal circumference. Menstrual irregularity started about 1 year ago, and her last menstruation was 3 months ago. She has lost around 5 kg over the last few months, but her waist has enlarged. Shortness of breath started a week ago, and it is worse when she is lying down; she now sleeps using at least two pillows. An examination of the lungs shows dullness to percussion, decreased tactile fremitus, and inaudible breath sounds bilaterally. Physical examination of the abdomen shows bulging of the flanks in the reclining position, and there is a difference in percussion in the flanks that shifts when she turns on her side. Pelvic examination shows a normal uterus and left adnexa; the right adnexum appears enlarged, smooth, and tender. Complete blood count and chemistry is normal, and a chest X-ray confirms the presence of pleural effusion on the right side. Fluid obtained from peritoneal cavity shows heterogeneously bloody content that clots; the leukocyte number is normal, and serum-ascites albumin gradient (SAAG) is 0.8 g/dL (low). The sample is negative of malignancy. What is the most likely cause of her symptoms?

NSAID analgesics Percutaneous nephrolithotomy Prednisone Shock wave lithotripsy Tamsulosin Tamsulosin

A previously healthy 35-year-old patient has had right flank pain radiating to the right groin for 3 hours. CT shows a 6 mm uric acid stone in the right proximal ureter. What treatment to facilitate stone passage is most appropriate?

Edema Serum protein Blood pressure Urine protein Lipids Urine protein

A previously healthy 8-year-old African American boy presented with a 3-day history of worsening fatigue and generalized edema. You saw him about a week ago when he had symptoms of a cold, for which you advised only supportive therapy. He appears alert and cooperative. His vitals are normal (temperature 37°C, pulse 90/min, respiratory rate 20/min, and blood pressure 100/70 mm Hg). Physical examination reveals the presence of generalized edema, and the rest of the examination is within normal limits. His laboratory values are below: The rest of his laboratory results are non-contributory. You plan to start therapy with prednisone. Protein 4+ What parameter would be most important to follow and evaluate the effect of therapy in order to prevent chronic kidney disease?

Flagellated protozoan

A recently married 22-year-old woman presents with a 7-day history of severe vulval and vaginal pruritus and purulent vaginal discharge. She was having dysuria and dyspareunia. Examination showed frothy yellowish mucopurulent vaginal discharge with an offensive odor. Vaginal mucosa appeared inflamed, and cervical erosion was also observed. Vaginal secretion was collected, and a microscopy of wet film done immediately. Oval pear-shaped organisms about the size of white blood cells with wobbling rotatory motility were observed among inflammatory cells. Gram stain was negative for Candida and clue cells. Based on the diagnosis, the patient and her husband were treated with metronidazole. What is a characteristic of the microbial agent causing her symptoms?

47,XXY

A young couple presents for advice about infertility. They have been married for 4 years and have never used contraceptive methods. They are both 27 years old, and the wife has an 8-year-old daughter from a previous relationship. Both are very eager to have a child; the husband is somewhat anxious about the results of 2 semen analyses that show azoospermia. He states that when he was 14 years old he was evaluated for delayed pubertal development; at that time, he had a chromosomal analysis that revealed that the problem is genetic. He was prescribed testosterone, which he used for only a few months. He is 6'8'' tall and has unusually long legs. His facial skin is smooth; there is no beard. There is palpable breast tissue bilaterally. His right fifth finger is curved towards the fourth finger. His pubic hair is substantially decreased for his age, and both testes are small (approximately 5 cm) and firm in consistency. What is his karyotype likely to show?

Sickle cell disease

An 11-month-old African American boy presents with an acute onset of anorexia, irritability, unexplained bruising, and jaundice. On examination, you note pale conjunctivae, icteric sclerae, and splenomegaly. Laboratory studies reveal decreased hemoglobin and hematocrit and a significantly elevated reticulocyte count. Hemoglobin electrophoresis reveals the presence of hemoglobin S. What is the most likely diagnosis?

Oral NSAIDs (Osgood Schlatter disease)

An 11-year-old boy comes in to see you due to knee pain. He notes that he started having right knee pain about 3 months ago that has increasingly become more apparent. The patient plays basketball in the winter and baseball in the spring for the last 2 years. When asked to point to where the pain is, the patient points to an enlarged area just inferior to the kneecap. Aggravating factors include participating in sports, especially when he is doing excessive amounts of running and jumping. Alleviating factors include resting the knee. Physical examination reveals a prominence and soft tissue swelling over the right tibial tubercle and tenderness to palpation of the right patellar tendon. The remainder of the musculoskeletal exam is unremarkable. Based on the most likely diagnosis, what is the most appropriate pharmaceutical regimen?

Decreased serum vitamin D Elevated serum gastrin Elevated serum phenylalanine Elevated sweat chloride Presence of Bence Jones protein in the urine Elevated serum phenylalanine

An 11-year-old boy presents with increasing cognitive trouble in school. His parents state that he has had more trouble with grades and schoolwork since starting middle school. He also seems slightly more defiant. His history is significant for a metabolic disorder; it was diagnosed at birth, but it has been controlled with diet. He is interviewed while his parents are out of the room; mild cognitive impairment is noted. He tells you that, since he started middle school, he has been "cheating" on his diet. What laboratory finding would be expected in this patient?

Chromosomal analysis for fragile X syndrome Reassessment of language in 3 months Computerized tomography scan of the brain without contrast Audiology consultation for a hearing evaluation Testing for pervasive developmental delay Audiology consultation for a hearing evaluation

An 18-month-old boy is brought in for a well-child visit. His mother states that he does not say any words, not even "mama" or "dada." He does have monosyllabic babble and points to objects that he wants. The remainder of the child's development is within normal limits, including gross and fine motor skills. He was born full-term via normal spontaneous vaginal delivery with no complications. He has always been healthy. He is currently not on any medications; he has no known drug allergies and all of his vaccinations are up to date. On observation, the child plays with the mother and makes good eye contact with her. The physical exam of the child is normal. The head circumference is normal. There is no cleft lip or palate, and the neurologic exam is within normal limits. What is the most appropriate next step in the management of this patient?

Smith fracture Torus fracture Salter-Harris IV fracture Colles' fracture Die-punch fracture Colles' fracture (looks like a dinner fork)

An 18-year-old man presents 30 minutes after falling on his outstretched arm while skateboarding. He is guarding his left forearm near his wrist with his right hand, and he has his left arm against his body for support. There is a deformity similar to the image with edema and ecchymosis at the wrist. He is neurovascularly intact and the skin is closed. His left elbow and shoulder exam are normal. With this type of deformity, what fracture is most likely?

Alcoholic hepatitis Crigler-Najjar syndrome Dubin-Johnson syndrome Gilbert's syndrome Infectious hepatitis Gilberts Syndrome

An 18-year-old man presents for a screening physical exam to join his college freshman lacrosse team. He reports no medical problems, and he does not take any medications. Physical exam is unremarkable. His immunizations are current, and he denies sexual activity or smoking. Review of routine labs reveals an elevation in unconjugated bilirubin. His total bilirubin level 4 mg/dL. Liver enzymes, serum electrolytes, complete blood count, and conjugated bilirubin level are within normal limits. What is the most likely diagnosis?

Begin physical therapy 3 times a week for 6 weeks. Begin a xanthine oxidase inhibitor and indomethacin. Perform a right knee radiograph. Prescribe a trial of ibuprofen and observe for 2 weeks. Refer the patient for a bone scintigraphy scan Perform a right knee radiograph.

An 18-year-old man with no significant past medical history presents with pain and swelling over the upper right knee for 5 months. The pain was initially insidious, dull, and achy. Over the past several weeks, however, it has gradually become progressively more severe and unremitting, often waking the patient at night. He also notes increased swelling, warmth, and erythema. He denies a history of injuries, accidents, trauma, surgeries, or sexual encounters. His physical exam reveals a noticeable limp, reduced right knee range of motion, and localized tenderness and swelling to the distal anterior femur. What is the next step in the management of this patient?

Isosthenuria Ureteral obstruction Hydronephrosis Diabetes Increased GFR Increased GFR --> increases with pregrenacy

An 18-year-old pregnant woman presents with nocturia and increased urinary frequency. She is at 18 weeks gestation and she has no past medical history. She has no suprapubic fullness; her fundal height is appropriate for her weeks of gestation. Diagnostic findings are as below: UrineNo cells, no blood, no protein; trace glucose, specific gravity 1.020Fingerstick glucose85 mg/dLRenal ultrasoundBilateral mild hydronephrosis, no noted ureteral abnormalities; bladder not distended post-voidSerum creatinine0.5 mg/dL What is the most likely explanation for her urinary frequency and nocturia?

Cognitive stimulation therapy Gestalt therapy Behavioral therapy Psychoanalytic therapy Existential therapy Behavioral therapy

An 18-year-old woman has been afraid of dogs ever since she was bitten as a child. You begin therapy by having the client do relaxation exercises while looking at the word "dog." Once the client is comfortable with this process, you have her do relaxation exercises while looking at a picture of a dog. The next step is to have the client do relaxation exercises while watching a dog from a distance. You then have the client do deep breathing exercises while sitting in the same room as a small dog. The final step is to have the client sit in the same room as a big dog while performing deep breathing exercises. What type of therapy are you using with the client?

Acute compartment syndrome

An 18-year-old woman presents to the emergency department with her mother due to pain in her right leg after a car accident. She points to her lower leg and describes the pain as severe. Upon physical exam, there is extreme pain with passive movement of right leg with diminished sensation. When asking the patient to stand on the leg, she reports weakness and extreme tenderness. Upon use of the Stryker IC pressure monitor system, the patient's pressure was 35 mm Hg. What is the most likely diagnosis?

Endometriosis Ventricular septal defect Von Willebrand disease Hemolytic uremic syndrome Hemophilia A Von Willebrand disease

An 18-year-old woman presents with chronic fatigue and menorrhagia. Menorrhagia has been present since her first menstrual cycle, but it has recently become worse; she sometimes uses 20 tampons per day. About a year ago, she started using contraceptive pills but is now considering stopping using them because of migraine-like headaches. Her headaches are sometimes so severe that she has to take aspirin or other painkillers several times a day. The rest of her past medical history is unremarkable. Physical examination reveals pale skin and mucosa, pulse rate of 100 beats per minute, and a systolic ejection murmur 1/3 intensity over the precordium. Laboratory findings include white blood count 9 K, hemoglobin 10, platelet count 250 K, normal prothrombin time, slightly prolonged partial thromboplastin time, and normal fibrinogen. Her bleeding time is prolonged. Blood smear shows microcytic hypochromic anemia. What is the most likely diagnosis?

Tolterodine Desmopressin Imipramine Oxybutynin Corticosteroid Desmopressin (suppresses urine: anti-diuretic)

An 8-year-old boy is evaluated for persistent bedwetting. He has never been continent, averaging 2-3 episodes of bedwetting per week. His urological evaluation revealed a normal bladder and urethra and no neurological problems. Lately, his problem has been a source of much embarrassment; he is unable to attend camp or sleepovers due to fear of wetting his bed. He has tried multiple interventions, including lifestyle changes, alarm systems, and reward systems. His physical exam shows no abnormalities. His parents are keen on a rapid resolution to his problems, and they insist treatment be initiated. What is the best therapy?

Pseudodementia Normal aging Alzheimer's disease Pick's disease Parkinson's disease Normal aging

An 82-year-old woman presents with her anxious daughter who lives with her. The daughter has noticed recent "lapses in memory" and feels that her mother has become forgetful lately. Her lapses in memory usually relate to people's names and recollection of past events and recent conversations. According to her, she is otherwise healthy; she takes calcium and vitamin D for osteoporosis, aspirin for her heart, and vitamin B complex. She has no history of trauma, strokes, or CNS infections. Her lapses in memory mostly affect short-term memory. She is able to carry out activities of daily living and is well oriented to time, place, and person. Her husband passed away 18 months ago, and she sometimes finds it difficult to sleep when she remembers his death. She feels "lonely" and "desperate" at times. Her BP is 130/80 mm Hg, and other vitals are normal. Neurological, cardiac, respiratory, and abdominal exam are normal. Her MMSE score is 28/30. What is the most likely diagnosis?

Obtain consent from his wife to perform surgery.

An 88-year-old man with abdominal pain presents to the emergency room with his wife. A mini-mental status exam reveals pronounced forgetfulness and confusion. The patient is discovered to have acute appendicitis requiring immediate surgery. He is unable to understand the situation and cannot provide informed consent. What further action should the physician take?

Father reports similar symptoms into adulthood

An 8-year-old boy presents for evaluation of problems at school and at home. His parents report that he does not pay attention in class, he is frequently in trouble for being disruptive, and he often forgets to do his schoolwork. He has had similar problems since starting school (in kindergarten), but they are worsening. The teacher reports the patient often seems distracted. He rarely sits still at his desk, fidgets often, and blurts out comments without waiting his turn. Physical examination is remarkable for increased motor activity but is otherwise normal. What additional aspect of the history would fit best with this patient's most likely condition?

Initiate stimulant medication

An 8-year-old boy presents for evaluation of problems at school and at home. His parents report that he does not pay attention in class, he is frequently in trouble for being disruptive, and he often forgets to do his schoolwork. He has had similar problems since starting school (in kindergarten), but they are worsening. The teacher reports the patient often seems distracted. He rarely sits still at his desk, fidgets often, and blurts out comments without waiting his turn. Physical examination is remarkable for increased motor activity but is otherwise normal. What is the best intervention for this patient's condition?

Attention-deficit/hyperactivity disorder

An 8-year-old boy presents for evaluation of problems at school and at home. His parents report that he does not pay attention in class, he is frequently in trouble for being disruptive, and he often forgets to do his schoolwork. He has had similar problems since starting school (in kindergarten), but they are worsening. The teacher reports the patient often seems distracted. He rarely sits still at his desk, fidgets often, and blurts out comments without waiting his turn. Physical examination is remarkable for increased motor activity but is otherwise normal. What is the most likely diagnosis?

Pseudomonas

An 8-year-old boy sustained a puncture wound to his right foot by stepping on a nail through his sneaker 4 days ago. His mother said the wound bled profusely but the nail did not go completely through his foot. They washed the wound at home with soap and water, wrapped it in a bandage, and did not seek further care. This morning, he complained that it was very painful and his mother noted that his foot looked red and swollen. On exam, his temperature is 99°F; pulse is 114 BPM, and his BP is 104/68 mm Hg. The plantar surface of his right foot has a small 2 mm scabbed entry wound that is surrounded by a 5-6 cm area that is erythematous, swollen, and quite tender. There is a scant amount of thin seropurulent material from the entry wound on examination. What bacteria is most likely the cause of the suspected complication the patient is experiencing?

Separation anxiety disorder Panic disorder Agoraphobia Social anxiety disorder Adjustment disorder Separation anxiety disorder (cant sleep alone and is clingy to parents)

An 8-year-old girl is presented because she recently started refusing to go to school. When her mother tries to explain that she must go to school, the girl begins to report headaches and stomachaches. She has no trouble going out with her parents to public places. She does not want to sleep alone and is clingy around both of her parents. These symptoms have been present for 6 weeks. She has been acting out towards her teachers in the classroom. Physical and laboratory findings, including drug tests, are within reference ranges. What is the most likely diagnosis?

Selegiline (Eldepryl) Vitamin E (Alpha-E) Risperdal (Risperidone) Donepezil (Aricept) Haldol (Haloperidol) Donepezil (Aricept)

An 80-year-old woman is diagnosed with dementia of the Alzheimer's type as a result of comprehensive testing. You believe she is in the very early stages of the disease and you want to try a medication to possibly slow disease progression. What drug would you be most likely to try first in this patient?

Claudication Gout Osteomyelitis Pes anserine bursitis Stress fracture Osteomyelitis

An 89-year-old Caucasian man is brought in by his daughter due to pain in his left shin. He bumped his leg on a coffee table about 3 weeks ago; he developed some mild discomfort, bruising, and a small gash in the skin. It seemed as if he was healing well, but his condition has worsened over the past few days. Now he is moaning due to pain, and he says it hurts to walk on the leg. He describes the pain as "horrible" and an 8/10. The daughter reports the pain keeps him up at night and is unresponsive to ibuprofen and narcotic pain pills. He denies fever and chills. On physical exam, the patient is in obvious pain and is assisted to the exam table with limited weight bearing on his left leg. The lower extremities are examined; significant findings include healing and a scabbed lesion of approximately 3 cm in length across mid-tibia, with surrounding erythema and edema. Tenderness is elicited along the shin, extending well past the area of erythema. Homan's sign is negative. Distal pulses, temperature, coloration, knee range of motion, and lower extremity reflexes are symmetric and normal. Right lower extremity is normal. What is the most likely diagnosis?

Principle of benign paternalism Principle of autonomy Utilitarian principle Principle of justice Principle of beneficence Principle of autonomy

An 89-year-old man presents with pneumonia. You suspect lung cancer, which would require surgery and possibly chemotherapy. Upon hearing the news regarding his diagnosis, the patient becomes tearful and sad; he confides his thoughts of suicide. You notice the patient seems confused at times, not knowing the place and people who surround him. When the patient is asked to sign the informed consent for a scheduled thoracotomy, he vehemently refuses any and all treatments for his condition, stating that he is going to die anyway. What is the ethical principle for obtaining informed consent in this case?

Osteoarthritis Gouty arthritis Fibromyalgia Avascular necrosis Septic arthritis Osteoarthritis

An afebrile 76-year-old man has a history of pain and stiffness in his knees bilaterally and in his right hip. The pain is worse after an activity or first thing in the morning. Otherwise, he is healthy and has no other complaints. Upon physical exam, the patient has limited range of motion, joint line tenderness of knees, and mild swelling in the painful joints without warmth. 2 of 18 "tender points" are tender to palpation. A synovial fluid analysis was conducted and it was negative for urate crystals. X-rays of the knees and right hip do not show sclerosis of the bone. Additionally, a Gram stain was conducted and was negative for any bacteria. What is the most likely diagnosis?

Adrenocorticotropic hormone Androgens Cortisol Follicle-stimulating hormone Adrenocorticotropic hormone

An obese 45-year-old woman is requesting medical weight loss therapy. She reports excessive weight gain of (37 kg) over the past 3 years. A detailed history also reveals easy bruising, oligomenorrhea, and increased hair growth on various parts of her body. A thorough examination shows hypertension (BP 180/110), truncal obesity with a buffalo hump, and moon face, along with hirsutism and pigmentation, with purple abdominal and lower leg striae. Lab reports confirm the most likely diagnosis. Based on the most likely diagnosis, high serum levels of what substance are causing hyperpigmentation?

Metabolic acidosis Respiratory acidosis Metabolic alkalosis Respiratory alkalosis Mixed pattern Metabolic acidosis

An otherwise healthy 50-year-old man is brought to your office, located in a small town in the foothills of the Appalachian mountains, by his daughter. The daughter reports that her father has been more lethargic than usual, and she states that she has seen him go into the backwoods and return obviously inebriated. You suspect that he is drinking moonshine from a homemade still and that it is leaching poisonous metals/substances into the distillate. The patient is morose and has difficulty answering questions. The rest of the physical exam uncovers no other abnormalities. The blood gas analysis shows a pH of 7.3, a pCO2 of 30 mm Hg, and a bicarbonate of 15 mEq/L. Further laboratory testing revealed sodium of 140 mEq/L and a chloride of 100 mEq/L. What is the most likely diagnosis?

Serum calcitonin levels

As part of a routine checkup, a 40-year-old man is evaluated by his family practitioner. He smokes 1/2 a pack of cigarettes a day, and he has an occasional glass of wine. For the past 5 years, he has been on SSRIs for mild depression. On general physical examination, his BP is 110/70, and his BMI is 25. The physician notes slight enlargement of his thyroid gland with a solitary left upper lobe nodule. Careful examination reveals cervical lymphadenopathy. An FNAC of the nodule shows large amounts of amyloid stroma and disorganized spindle-shaped cells with large vesicular nuclei. What laboratory test would have helped identify the tumor at an early stage?

Urologist for radical nephrectomy

Case A 62-year-old man presents with persistent hematuria. He denies any dysuria, urgency, or frequency. The first episode of hematuria was discovered 4 months ago on a routine urinalysis for a workplace physical. A repeat urinalysis 6 weeks later again shows hematuria, confirmed by microscopic evaluation. The remainder of his urinalysis is within normal limits. His past medical history is remarkable for COPD and obesity. He has smoked 1.5 ppd x 45 years. He uses inhaled medications for his COPD and has NKDA. His physical exam is significant for a temperature of 100.5°F. Abdominal exam elicits mild tenderness in the left upper quadrant, but no masses are palpable (although his exam is limited by his obesity). Chest and abdominal CTs with contrast show a normal chest and a left renal mass enhanced by radiocontrast, suggestive of renal cell carcinoma (RCC). No metastatic disease was noted on imaging. A renal biopsy is pending. Assuming the biopsy supports the diagnosis of renal cell carcinoma, what referral would be most appropriate for this patient?

Anal fissure

Case A 28-year-old man presents with rectal bleeding. The patient has noticed blood with bowel movements 3 times. The blood is described as bright red in color and small in amount. He also complains of rectal pain, especially with passing hard stools. He has tried some over-the-counter hemorrhoid creams without relief. The patient admits episodic constipation. He denies dark, tarry stools, easy bruising, and prior episodes of rectal bleeding. He has not noticed blood in his urine or with brushing his teeth. He denies nausea, vomiting, diarrhea, fevers, and weight loss. He has no known medical conditions. Family history is negative for gastrointestinal disorders. Social history reveals he is in a heterosexual relationship, and he denies anal intercourse. On physical exam, his abdomen is normal. The anus has no visible protrusions or rash, but there is a very small erythematous and tender area that appears like a "paper cut" or crack in the skin. The patient experiences pain with digital rectal exam (DRE). No masses are noted in the rectal vault. What is the most likely diagnosis?

Neurohypophysis

Case A 45-year-old woman is constantly thirsty and consumes large amounts of water. She has a history of sarcoidosis. In addition, her urine output is in the range of 7 liters per day; she wakes up several times a night to urinate. Upon administration of aqueous vasopressin, her urine osmolarity rises above the value of her plasma osmolarity. What structure is most involved in the pathophysiology of this patient's condition?

Surgical referral - Hernia surgery

Case A 48-year-old man presents with a 2-day history of left-sided groin and scrotal pain. He has had similar pain episodically for several months, but it has recently become much worse after a weekend of helping his brother move furniture. He admits that he is not in good physical shape, and he thinks he may have pulled a groin muscle. He is in a monogamous relationship with his wife of 17 years. He has never had any testicular or scrotal conditions, and he has a negative surgical history. He denies fever and urinary symptoms. He has no allergies and takes no other medications. On physical exam, the patient has normal sexual development, with no edema, warmth or erythema present in the scrotum. No skin lesions are present. On palpation, there is mild tenderness on the left scrotum. However, with Valsalva, a small bulge is palpable in the left scrotum, and the patient's reported pain level increases. When he lies supine, the bulge is no longer palpable. What intervention is most appropriate for this patient's suspected condition?

Rotavirus

In the winter, an 11-month-old male infant presents with a 2-day history of vomiting, diarrhea, and fever. He has not had routine medical care since birth. Mother reports no significant past medical history. His temperature is 102°F. Clinically, he appears dehydrated; his white blood cell count is 5400 cells/mm3 with a normal differential. His stool and urine are negative for white blood cells. What is the most likely cause of gastroenteritis in this child?

Classic galactosemia Congenital hypothyroidism Phenylketonuria Sickle cell disease Tyrosinemia (of the newborn) Phenylketonuria

Mandatory newborn screening has been completed on a Caucasian infant who was born 36 hours ago. The birth was relatively benign without any complications; the newborn was born at exactly 40 weeks gestation. An abnormal result found in the screening tests has resulted in you ordering required counseling for the parents to initiate lifelong modification in their child's diet to decrease the incidence of abnormal intellectual development. What disease state was most likely found during the screening?

Tyrosine Fructose Phenylalanine Thyroid hormone Leucine Phenylalanine

Mandatory newborn screening has been conducted on a neonate born 36 hours ago. The abnormality found in the screening has resulted in counseling on initiating lifelong modification to the newborn's diet. It is explained to the mother this action must be taken in order to help decrease the incidence of severe intellectual disability, hyperactivity, and even seizures. Considering the most likely diagnosis of this patient, what must be limited in order to prevent these consequences?

Autism spectrum disorder MMR adverse effect Transcortical mixed aphasia Conduct disorder Tourette's syndrome Autism spectrum disorde

Parents bring their 3-year-old son for evaluation of a recently diagnosed intellectual disability. The test for intellectual disability was performed primarily because of delayed speech. The mother states that her child started to babble at about 9 months of age and then learned a few words such as "dada" and "boo" at 18 months. Despite the efforts of his parents to stimulate his language (reading to him, singing, exposing him to sounds, teaching him to mimic their speech, etc.), his speech has remained far behind other children his age. He is not interested in playing with the other children, always looks serious, and behaves more independently than other children. His prenatal and past medical history is unremarkable; he was always healthy, and his immunizations are up to date, including MMR. While you are taking the anamnesis, he does not seem interested in the conversation. Instead, he started spinning himself, an activity that he enjoyed so much that he did not respond when parents tried to stop him. When asked, his parents left the room, but the child did not pay attention to them leaving. Then he saw fancy colored wooden sticks and started sorting them out by colors because nobody could interrupt him. Developmental screening today is impossible because he simply ignores you, so you decide to schedule a follow-up evaluation. What is your initial diagnosis?

X-rays of the lower extremities Reassurance that this is normal for age Orthopedic shoes Long leg braces Lab tests for markers of bone turnover Reassurance that this is normal for age (patient will grow out of it)

Parents bring their 4-year-old daughter in because of knock-knee. She is otherwise healthy, and her height is in the 50th percentile for age. On examination, she has about 10 degrees of valgus. What should you recommend?

Nephrotic syndrome

The effect of steroid therapy is evaluated in an 8-year-old Caucasian boy being treated for fatigue and generalized edema following a "bad cold." His vitals are T 37°C, pulse 90/min, RR 20/min, and BP 110/70. Physical exam reveals the presence of mild periorbital edema and marked peripheral edema in hands and feet with the remainder of the exam within normal limits. Lab values include Dipstick urine protein 3+; urine protein 50 mg/m2/hr (<40 mg/m2/hr) Specific gravity 1030 (1008-1020) Urine protein/creatinine ratio 2.0/mg creatinine (<0.2/mg creatinine) Serum albumin 3.9 (5.9-8.0 mg/dL) Cholesterol 250 (112-247 mg/dL) Remainder of laboratory values including BUN and plasma creatinine are within normal limits. What is the most likely diagnosis?

Schizoaffective disorder Schizoid personality disorder Schizophrenia Schizophreniform disorder Schizotypal personality disorder Schizophrenia

Police officers found a 27-year-old man walking aimlessly and shouting the names of former presidents. Urine toxicology is negative, and the man appears to be oriented with respect to person, place, and time. He has had 5 similar admissions over the past year. Attempts to interview the patient are fruitless; he is easily derailed from his train of thought. A phone call to a friend listed in the chart provides the additional information that the man is homeless and unable to care for himself. This patient's signs and symptoms are characteristic of what pathology?

Phenylalanine

State mandatory newborn screening has been conducted on an neonate born approximately 36 hours ago. An abnormality found in the screening has resulted in counseling of the parents on initiating lifelong modification to the newborn's diet. It is explained to the parents that this action must be taken to help decrease the likelihood of the infant developing severe intellectual disability, hyperactivity, and seizures. Considering the most likely diagnosis, what must be limited in this patient's diet to prevent these complications?

Cryptorchidism

Upon inspection of an 18-year-old man's scrotum, you note that the left side is underdeveloped and a testis is not palpable. There is no scrotal tenderness, swelling, or nodularity. Considering this is not an acute finding, what is the most likely diagnosis?

The effectiveness is related to its action as a serotonin antagonist. The most common side effect is irritability. The recommended oral dose is 6 mg. It is indicated for both treatment and prophylaxis of migraine headache. If it is effective in the initial therapy, it is often effective in aborting a recurrence of symptoms. If it is effective in the initial therapy, it is often effective in aborting a recurrence of symptoms.

What is the most accurate statement concerning the treatment of migraine headache with sumatriptan?

Closure of the wound Debridement Hemostasis History of wound mechanism Tetanus immunization Hemostasis (Large amounts of blood pooling or notable pulsations warrant urgent exploration for arterial damage, which could need emergent clamping and ligation.)

While covering the ED, you attend to a 12-year-old boy who cut his hand while trying to slice a bagel. His mother wrapped his hand with a dishrag and drove him to the ED. On exam, his vitals are stable. He has remained alert and cooperative, but the rag has soaked through and his fingers feel numb. On removal of the rag, there is a fairly deep-looking straight-edged transverse laceration running the length of his palm, with overlying clotted blood that begins to actively bleed. In overall wound management, what is the first priority in evaluating this patient?

Yes; his seizure disorder is well-controlled

You are evaluating a 14-year boy for a pre-participation sports physical. He has been conditionally accepted as a wide receiver on his high school's football team. He was diagnosed with generalized tonic-clonic seizures at age 6. He is well-controlled on valproic acid, having had only 2 seizures in the past 3 years, associated with an intercurrent illness. There is no history of status epilepticus, head trauma, or other neurologic abnormalities, and he maintains a B+ average in school. Past medical history is otherwise unremarkable. Growth and development have been normal and immunizations are current. Vital signs are normal. Examination is unremarkable. Would you clear him for football?

"Serious medical risks exist in connection with the use of this substance.

You are evaluating a 17-year-old Caucasian boy on his high school's cycling team. He admits that he has been injecting himself with erythropoietin (EPO)—provided to him by a recent graduate from the team—once every 2 weeks for the past 3 months. While he is convinced that he has achieved the desired increase in endurance needed to win consistently, he is concerned by stories he has heard regarding the side effects of this drug. Past medical history is unremarkable and there is no history of tobacco, alcohol, or recreational drug use. Growth and development have been normal and immunizations are current. Vital signs are normal. The examination is unremarkable. How would you advise him?

Klinefelter syndrome

You are evaluating a 26-year-old man; he is suspected of being infertile. His past medical history is unremarkable. On examination, you note he is 6'4"; he has mild gynecomastia, sparse body hair, and small soft testes. What is the most likely diagnosis?

Ultrasound of the right lower leg CBC MRI of the right hip with and without contrast CXR Urinalysis MRI of the right hip with and without contrast

You are evaluating a 34-year-old African American man for a 5-week history of increasing right groin pain. He denies any injury or history of similar pain. The pain is worse with movement and has progressed to the point that the patient has severe pain with bearing weight. He denies fever, chills, urinary symptoms, or any other issue at this time. He has a past medical history of sickle cell disease and hypothyroidism. Physical exam reveals tenderness upon palpation of the groin with increased pain on both active and passive range of motion of the hip. Homan's sign is negative. Distal pulses and sensation are intact and normal. What diagnostics study should be ordered at this time?

Ecchymoses Jaundice Large mass Lymphadenopathy Petechiae Lymphadenopathy

You are following up on laboratory results for your supervising physician while they are out of town. A bone marrow biopsy result for a 62-year-old man is noted. Bone Marrow aspiration (biopsy)Infiltration with small lymphocytes, B-cell. Coexpression of CD19, CD20, and CD5 On further chart review, you see that this patient presented to your clinic for review of abnormal health fair screening tests for his employment. The patient reported that he felt well in general, but he added that he could not exercise as vigorously as he could in the past; he attributed that to his aging. The table shows his complete blood count. ***Large Chart*** If you were to review this patient's physical exam from his prior visit, what finding would you expect to see in his chart?

Chemoimmunotherapy

You are following up on laboratory results for your supervising physician while they are out of town. A bone marrow biopsy result for a 62-year-old man is noted. Bone Marrow aspiration (biopsy)Infiltration with small lymphocytes, B-cell. Coexpression of CD19, CD20, and CD5 Upon further chart review, you see that this patient presented to your clinic for review of abnormal health fair screening tests for his employment. The patient reported that he felt well in general, but he added that he could not exercise as vigorously as he could in the past; he attributed that to his aging. The table shows his complete blood count. ***Large Chart*** What pharmaceutical treatment should primarily be used to treat his condition, especially as it progresses?

Major depression Anorexia nervosa Constitutional development delay Hypergonadotropic hypogonadism Hypogonadotropic hypogonadism Hypogonadotropic hypogonadism

You are performing an annual physical examination of a 14-year-old girl. Over the last 3 years, she has been treated for the depression related to her parents' divorce process. She complains of frequent constipation followed by loose stools and is treated for dermatitis herpetiformis. She participates in swimming competitions and spends about 18-20 hours per week training. However, her mother is worried because she has not grown enough and still has not gotten her period (her mother had her period when she was 12 years old). The rest of her personal and family history is not contributing. Your patient's BMI is 15 (percentile 3%); she is in Tanner stage 2 (the same as last year); and her bone age is 12.5 years. The rest of the physical examination is normal, and complete blood count results are within normal limits. What is the most likely diagnosis?

Gluten-free diet Ferrous sulfate Emotion-focused therapy Low-carbohydrate diet Growth hormone Gluten-free diet

You are performing an annual physical examination on a 14-year-old girl. In the last couple of years, she started having constipation "every other day or so," followed by loose stools. She still feels depressed because of her parents' recent divorce. She has no problems at school, her grades are good, and she participates in sports. Her mother is worried because she is so slim despite excellent appetite: that she has not grown enough. She still has not gotten her period. Her mother had her first period when she was 13. The rest of personal and family history is non-contributory. The patient's height is 5 ft, weight 79 lb (BMI 15.46; 3rd percentile); she is in Tanner stage 2 (the same as last year, according to her records); and her bone age is 12.5 years. The rest of physical examination is normal. Laboratory shows Hct of 31% and MCV of 73, low insulin-like growth factor (IGF), low FSH, positive anti-tissue transglutaminase antibodies. The rest of laboratory results are within normal limits, including TSH and prolactin. What will restore normal puberty, growth, and weight in your patient?

Agnosia Diplopia and blurred vision Headaches Memory loss Polyuria and polydipsia Polyuria and polydipsia

You are reviewing incoming test reports while your supervising physician (SP) is out of town. There are two reports for a 46-year-old man shown in the table. TestFindingsVasopressin challenge testPositive responseMRI brainMild thickening of the pituitary stalk, no masses or lesions Your SP left a note planning to initiate desmopressin after the test results confirmed the suspected diagnosis. What symptom would you expect to find in reviewing the history portion of this patient's chart?

Bladder cancer Hodgkin's disease Retinoblastoma Bone and soft tissue sarcoma Familial polyposis coli Bone and soft tissue sarcome (radiation is a risk)

You have inherited three adolescent patients who had different early childhood malignancies that were treated in different ways according to the type and grade of cancer. They have been in remission now for over 5 years. In reviewing their charts, you consider the need to monitor for late effects of childhood cancer. What secondary malignancy is commonly associated with radiation therapy?


Conjuntos de estudio relacionados

Ch 1 Independent Contractor or Employee

View Set

Chapter 6: The Revolution Within

View Set

Diversity Strategy Terms - Pricing

View Set

properties of probability distributions

View Set

Psychology: Relationships - Factors affecting attraction

View Set

Chapter 17 Dosage Calculations and medication administration

View Set